ROSH REVIEW Orthopedics

Pataasin ang iyong marka sa homework at exams ngayon gamit ang Quizwiz!

One Step Further Question: What is the classic radiographic finding associated with ankylosing spondylitis?

Answer: Symmetrical squaring of the margins of the vertebral bodies that over time develops into a "bamboo spine.

One Step Further Question: High ankle sprains involve a partial tear of which ligament?

Answer: Syndesmosis.

One Step Further Question: List some diseases that are associated with Raynaud's phenomenon

Answer: Systemic lupus erythematosus, mixed connective tissue disorder, rheumatoid arthritis, Sjogren's, dermatopolymyositis, vasculitis and primary pulmonary hypertension.

One Step Further Question: Which nerve is most often injured with shoulder dislocations?

Answer: The axillary nerve.

One Step Further Question: What nerve is most commonly damaged in tibiofemoral dislocations?

Answer: The common peroneal (fibular) nerve.

One Step Further Question: How is Finkelstein's test performed?

Answer: The examining physician grasps the patient's thumb and ulnar deviates the hand sharply.

One Step Further Question: Examination of which structure is important in any suspected ankle fracture or injury?

Answer: The fibular head. External rotation forces at the ankle can cause a Maisonneuve fracture, a compilation of fibular head fracture, tear of the ankle's medial collateral ligament and disruption of the tibiofibular syndesmosis.

One Step Further Question: What nerve innervates the flexor digitorum superficialis?

Answer: The median nerve.

One Step Further Question: Which joint is most commonly affected in gout?

Answer: The metatarsal-phalangeal joint of the great toe.

One Step Further Question: Osteonecrosis is more likely with fractures of the scaphoid in the proximal or distal pole?

Answer: The more proximal the fracture, the greater likelihood of osteonecrosis.

One Step Further Question: When should the nail be removed in a patient with a subungal hematoma?

Answer: The nail may be removed if it is split, avulsed or a laceration extends to the skin.

One Step Further Question: What is a common description of the relief felt by spinal stenosis patients when bending forward?

Answer: The shopping cart sign

One Step Further Question: What is a special test used to elicit radicular nerve pain?

Answer: The straight leg raise test.

One Step Further Question: What is the prognosis for greater trochanteric pain syndrome?

Answer: The syndrome is generally self-limiting and resolves spontaneously.

One Step Further Question: What is the treatment for Slipped Capital Femoral Epiphysis (SCFE)?

Answer: The treatment for SCFE is prompt operative stabilization.

One Step Further Question: What are the typical laboratory abnormalities found in patients with myofascial pain syndrome?

Answer: There are no specific lab tests to confirm or refute a diagnosis of myofascial pain syndrome.

One Step Further Question: In what age group are supracondylar fractures common?

Answer: They typically occur in children between 5 and 10 years of age.

One Step Further Question: What is the recommend splint for a scaphoid fracture?

Answer: Thumb spica splint.

One Step Further Question: List some modifiable risk factors for osteoporotic fracture?

Answer: Tobacco and alcohol use, body weight <127 pounds, regular use of corticosteroids or anticonvulsants, poor balance, poor eyesight, recurrent falls and low calcium intake.

One Step Further Question: What is the role of topical acyclovir in herpetic whitlow?

Answer: Topical acyclovir may reduce the length of symptoms.

One Step Further Question: What physical exam technique helps to differentiate ganglion cysts from other nodular lesions of the hand and wrist?

Answer: Transillumination.

One Step Further Question: True or false: Metacarpal fractures are more common in adults than children?

Answer: True.

One Step Further Question: True or false: Patients with fibromyalgia have no evidence of joint or muscle inflammation on physical exam?

Answer: True.

One Step Further Question: What is the most common symptom (aside from back pain) seen in cauda equina syndrome?

Answer: Urinary retention.

One Step Further Question: In which population does Sjogren's most commonly occur?

Answer: Women in their late-forties to mid-fifties.

One Step Further Question: What vascular structure is at risk of injury with a proximal fibular fracture?

Answer: Anterior tibial artery.

One Step Further Question: What are the common etiologies of spinal subarachnoid hemorrhage?

Answer: Arteriovenous malformation, tumors and anticoagulation.

One Step Further Question: What other medication should be given to patients with giant cell arteritis?

Answer: Aspirin.

One Step Further Question: How long should a scaphoid fracture be immobilized?

Answer: At least 6 - 12 weeks.

One Step Further Question: Which endocrinopathies predispose adolescents to Slipped Capital Femoral Epiphysis?

Answer: Hypothyroidism and growth hormone deficiency.

One Step Further Question: What type of immune complex is associated with Henoch-Schönlein Purpura?

Answer: IgA. Rapid Review

One Step Further Question: What are two relative contraindications to performing an arthrocentesis?

Answer: Overlying cellulitis and coagulopathy.

One Step Further Question: Using the 1990 criteria, how many trigger points must a patient have to meet criteria for the diagnosis of fibromyalgia?

Answer: At least eleven of eighteen symmetrical trigger points.

One Step Further Question: What class of antibiotics is associated with tendon rupture?

Answer: Fluoroquinolones. "FluoroquinoLONES hurt attachments to your BONES."

One Step Further Question: What is a Jones fracture?

Answer: Fracture of the proximal fifth metatarsal diaphysis.

One Step Further Question: What set of guidelines can be used to determine if a patient with an acute ankle injury requires radiographs?

Answer: Ottawa ankle rules.

One Step Further Question: What is intersection syndrome?

Answer: An overuse tendinopathy manifesting with pain to the radial side of the wrist 4-8 cm proximal to the site of de Quervain's disease.

One Step Further Question: What bedside test is helpful in ruling out vascular claudication?

Answer: Ankle-brachial index.

Spinal Injuries

A lumbar disk abnormality (A), such as herniation, typically causes unilateral radicular pain and symptoms, but it is almost exclusively associated with normal rectal tone and perineal sensation. Spinal cord tumor, or other spinal cord neuronal cell body disease affecting the gray matter (C), would cause upper motor neuron symptoms. Hyperreflexia, positive Babinski testing and spasticity are not present in this patient. Spinal cord neuronal axon or tract disease affecting the white matter (D), as in multiple sclerosis, usually presents earlier in age, and with upper extremity and cranial nerve neurologic findings, both of which are absent in this case A normal adult spinal cord ends as the conus medullaris at the L1-L2 level. The remaining L2-S4 spinal nerve roots that occupy the subdural space from the conus to the filum terminale are called the cauda equina. The cauda equina is a collection of post-ganglionic nerves (lumbosacral nerve roots), so injury to it presents as a lower motor neuron injury pattern with flaccid tone, paresis or paralysis, and hyporeflexia (or areflexia). Upper motor neuron symptoms such as hyperreflexia and clonus, spasticity and upgoing toes (positive Babinski test) are absent. The clinical presentation of cauda equina syndrome includes back pain, perianal numbness, loss of rectal tone, bowel and bladder (urinary retention followed by incontinence) abnormalities, leg weakness, loss of bulbocavernosus reflex, and loss of deep tendon reflexes. Cauda equina syndrome most commonly results from a herniated disk, but can also occur from trauma, an infection (epidural abscess), or a hematoma (epidural hematoma).

A 58-year-old man presents with bilateral buttock and shoulder pain. Other than hypertension, he has no significant medical history. He states that he has felt "stiff" and "tired" for the past few weeks. He denies any specific injury, but does report being more active in the garage working on his classic automobile. He also denies back pain. Examination reveals no skin or digit abnormalities. Ophthalmologic examination is within normal limits. The gluteal and periscapular muscles are tender to palpation, but there is no discernable trigger point, warmth or induration. Which of the following laboratory abnormalities is most likely present in this patient? Antinuclear antibody positivity Erythrocyte sedimentation rate > 50 mm/h HLA-B27 positivity Sideroblastic microcytic anemia

A positive ANA (A) suggests a mixed-connective tissue disease, rheumatoid arthritis, systemic lupus erythematosus, scleroderma, polymyositis or Sjogren's syndrome, all of which typically display skin and/or digit abnormalities. A positive HLA-B27 (C) suggests a presence of the seronegative spondyloarthropathies: ankylosing spondylitis (spine and sacroiliac pain), reactive arthritis (conjunctivitis, urethritis, arthritis), psoriatic arthritis (skin and nail findings) and enteropathic arthropathy (history of Crohn's or ulcerative colitis). Sideroblastic microcytic anemia (D) does not occur in PMR. If present, the type that accompanies PMR is a mild normochromic normocytic anemia.

One Step Further Question: What is CREST Syndrome?

Answer: Another mixed connective tissue disorder but it is considered more of a constellation of symptoms: Calcinosis, Raynaud's phenomenon, Esophageal dysmotility, Sclerodactyly and Telangiectasias.

One Step Further Question: Which two anatomic compartments are the most affected in acute compartment syndrome?

Answer: Anterior compartment of the leg and the volar compartment of the forearm.

One Step Further Question: What is the most common injury associated with traumatic hemarthrosis of the knee joint?

Answer: Anterior cruciate ligament (ACL) rupture.

One Step Further Question: Which anatomical structures are involved in the "unhappy triad"?

Answer: Anterior cruciate ligament, medial meniscus and medial collateral ligaments.

One Step Further Question: If a patient has ankle instability, what physical exam tests should be positive?

Answer: Anterior drawer test and talar tilt test

One Step Further Question: What nerve is responsible for pincer grip between the thumb and index finger?

Answer: Anterior interosseous branch of the median nerve.

One Step Further Question: What is the most commonly sprained ankle ligament?

Answer: Anterior talofibular ligament.

One Step Further Question: Which ligament is most commonly damaged in an inversion injury of the ankle?

Answer: Anterior talofibular ligament.

One Step Further Question: What is the normal pressure of a tissue compartment?

Answer: < 10 mmHg.

One Step Further Question: What is Ewing sarcoma?

Answer: A marrow malignancy, common in the second decade, that causes lysis and osteomyelitis.

One Step Further Question: What is a Maisonneuve fracture?

Answer: A spiral fracture of the proximal 1/3 of the fibula associated with a tear of the distal tibiofibular syndesmsosis and the interosseous membrane.

One Step Further Question: What is the most common cause of an acute hemarthroses after a sports-related knee injury?

Answer: ACL tear.

One Step Further Question: What is the most common cause of an acute hemarthrosis after a sports-related knee injury?

Answer: ACL tear.

One Step Further Question: What antibody is classically associated with scleroderma?

Answer: ANA positive with anticentromere antibodies (limited), and anti-SCL-70 (progressive).

One Step Further Question: Which abnormal reflex is most indicative of intraspinal pathology?

Answer: Abdominal.

One Step Further Question: When is emergent orthopedic consultation required for carpal tunnel syndrome?

Answer: Acute onset of symptoms over hours suggests the presence of acute median nerve compression from fracture or infection and requires emergent decompression.

One Step Further Question: What is first line therapy for spinal stenosis?

Answer: Adequate pain management, physical therapy, exercise, and weight loss

One Step Further Question: What medication is used to prevent attacks of recurrent gouty arthritis?

Answer: Allopurinol.

One Step Further Question: What drug commonly used for maintenance therapy of gout is avoided during an acute gout attack?

Answer: Allopurinol. Except in diuretic-induced gout.

One Step Further Question: Is estrogen replacement therapy (ERT) indicated in primary prevention of osteoporosis in women?

Answer: Although once believed to be beneficial, ERT therapy is no longer indicated for primary prevention of osteoporosis.

One Step Further Question: What antibiotic is first-line therapy for human bite infection prophylaxis?

Answer: Amoxicillin-clavulanate.

One Step Further Question: What is a typical radiographic finding that supports a diagnosis of a Lisfranc dislocation-fracture?

Answer: An AP view which reveals lateral shift of the second metatarsal off the middle cuneiform.

One Step Further Question: What is a felon?

Answer: An infection of the pulp of the distal finger or thumb.

One Step Further Question: In what age group are physeal injuries most common?

Answer: Boys aged 12-15 and girls aged 9-12.

One Step Further Question: What are the indications for non-operative management of a complete ACL tear?

Answer: Bracing and lifestyle modifications should be considered in adult patients with a complete tear if their daily activities do not involve jumping, cutting, or heavy manual labor.

One Step Further Question: Which condition commonly coexists with polymyalgia rheumatica?

Answer: Giant cell arteritis, the most common elderly primary vasculitis, occurs in 10% of patients with PMR. The main symptoms are jaw claudication, temporal artery pain or beading and diplopia in patients >50 years of age.

One Step Further Question: What is the most common type of major joint dislocation?

Answer: Glenohumeral joint (shoulder).

One Step Further Question: If MRI is not available, what other radiographic modality may identify cord compression?

Answer: CT Myelography.

One Step Further Question: In patients with chronic or unresponsive paronychia, what organisms are likely involved?

Answer: Candida albicans and atypical mycobacteria.

One Step Further Question: Tarsal tunnel syndrome is analogous to which other nerve entrapment pathology?

Answer: Carpel tunnel syndrome.

One Step Further Question: Which type of foot anomaly is plantar fasciitis common in?

Answer: Cavus feet.

One Step Further Question: What is the treatment of choice for disseminated Neisseria gonorrhoeae septic arthritis?

Answer: Ceftriaxone.

One Step Further Question: Developmental dysplasia of the hip occurs with increased incidence if an infant also has which neck condition?

Answer: Congenital torticollis ("wry-neck", most commonly caused by sternocleidomastoid fibrosis).

One Step Further Question: What is the first step in treatment of chemical contamination of the eyes?

Answer: Copious irrigation with normal saline should be utilized to flush the eye for 15-30 minutes.

One Step Further Question: What is the treatment of multiple myeloma?

Answer: Corticosteroids, chemotherapeutics (melphalan, bortezomib, doxorubicin), interferons, and bisphosphonates (pamidronate).

One Step Further Question: What genitourinary infection may cause both septic and reactive arthritis?

Answer: Gonorrhea

One Step Further Question: What genetic factor is linked to reactive arthritis?

Answer: HLA-B27.

One Step Further Question: What are some dangerous side effects of long-term methotrexate use?

Answer: Hepatotoxicity, stomatitis, leukopenia, bone marrow suppression, interstitial pneumonitis, renal toxicity, neurotoxicity.

One Step Further Question: What is the name for herpes simplex infection of the distal finger?

Answer: Herpetic whitlow.

One Step Further Question: Avascular necrosis is a complication of which traumatic hip injuries?

Answer: Hip dislocation or femoral neck fracture.

One Step Further Question: Which endocrinopathies are associated with pseudogout?

Answer: Hyperparathyroidism and hypothyroidism.

One Step Further Question: Which pathological disorder is associated with pretibial myxedema?

Answer: Hyperthyroidism, specifically Grave's disease.

One Step Further Question: What is cauda equina syndrome?

Answer: Damage to the cauda equina causes loss of function of the lumbar plexus (nerve roots) of the spinal canal below the termination (conus medullaris) of the spinal cord.

One Step Further Question: What is the innervation of the extensor pollicis brevis and abductor pollicis longus?

Answer: Deep branch of the radial nerve.

One Step Further Question: What are the nodular deposits seen in gout?

Answer: Deposits of uric acid crystals, or tophi, which can be seen in subcutaneous tissues, tendons, cartilage, and bone.

One Step Further Question: What physical exam finding is a contraindication to manual reduction of a tibiofemoral dislocation?

Answer: Dimple sign.

One Step Further Question: In rheumatoid arthritis of the hands, which joints are generally spared?

Answer: Distal interphalangeal joints

One Step Further Question: What is the treatment of cauda equina syndrome?

Answer: Emergent decompression (a surgical emergency).

One Step Further Question: What is the Neer test?

Answer: Exam of the shoulder where the examiner performs maximal passive forward flexion with internal rotation while stabilizing the patient's scapula with the other hand.

One Step Further Question: What is the MRI finding which confirms a diagnosis of hip avascular necrosis?

Answer: Femoral head hypointensity (T1) and double-line signs (T2).

One Step Further Question: When occurring in males, which underlying condition is commonly associated with symptoms of fibromyalgia?

Answer: Fibromyalgia may be a complication of sleep apnea, particularly in male patients.

One Step Further Question: As compared to Dupuytren's contracture, where is the pathology located in stenosing tenosynovitis (trigger finger)?

Answer: Fibrotic nodules occurring in the digital flexor tendons. Dupuytren's contracture (B) refers to a thickened and fibrous palmar fascia which is due to fibrous proliferation. It occurs commonly in men over age 40 years, but it also is common in those of Northern European descent, diabetics, alcoholics and epileptics.

One Step Further Question: Where is the most common site for gout?

Answer: First MTP joint (podagra).

One Step Further Question: What is the treatment for patellofemoral syndrome?

Answer: First-line treatment for patellofemoral syndrome is rest or activity modifications, NSAIDs, and physical therapy for quadriceps strengthening (specifically strengthening of the vastus medialis obliquus).

One Step Further Question: What is the treatment for a SLAP tear?

Answer: First-line treatment is usually non-operative management with nonsteroidal anti-inflammatory drugs and physical therapy, with consideration for surgery if the patient fails conservative treatment.

One Step Further Question: What is the motor function of the common peroneal nerve?

Answer: Flexion of the leg at the knee through innervation of the short head of the biceps femoris muscle

One Step Further Question: What are the superior and inferior boundaries of the carpal tunnel?

Answer: Flexor retinaculum (superior) and the carpal bones (inferior).

One Step Further Question: What is the first-line treatment for an acute attack of gout?

Answer: Indomethacin. Gout is characterized by painful joint inflammation, most commonly in the first metatarsophalangeal joint, resulting from precipitation of urate crystals in a joint space. The cultures are sterile in gout and "rat-bite" erosions may be seen on joint radiographs. Diagnosis is confirmed by identification of negatively birefringent, needle-like monosodium urate crystals in synovial fluid aspiration. Rheumatoid arthritis is characterized by symmetric, inflammatory joint pain that is worse in the morning and is associated with warmth, erythema and swelling of the joints. Synovial fluid analyses is not needed to diagnose rheumatoid arthritis, instead it is based on a timeline of clinical signs and symptoms as well as radiographic changes and serum rheumatoid factors.

One Step Further Question: What is the classic description of ankylosing spondylitis?

Answer: Insidious onset of bilateral buttock pain (sacroiliitis), thoracolumbar pain (enthesitis) and lumbar morning stiffness that lessens with exercise most commonly seen in young men.

One Step Further Question: What is another term for xerophthalmia?

Answer: Keratoconjunctivitis sicca.

One Step Further Question: What are the classic muscle biopsy results associated with polymyositis?

Answer: Large nuclei, fiber size variation, atrophy and the simultaneous presence of necrotic and normal type I and type II fibers.

One Step Further Question: What are the unique laboratory findings of pleural effusions associated with rheumatoid arthritis?

Answer: Low glucose, low pH and high LDH.

One Step Further Question: Intake of which food group correlates with lower urate levels?

Answer: Low-fat dairy products.

One Step Further Question: What are the two most common primary sources of spinal metastatic disease?

Answer: Lung and breast cancer.

One Step Further Question: What are the histological findings of Sjögren's syndrome?

Answer: Lymphocytic foci of tightly aggregated lymphocytes.

One Step Further Question: Besides repeat X-rays, what imaging modality can confirm a scaphoid fracture?

Answer: MRI.

One Step Further Question: What imaging modality is the gold standard for diagnosing an anterior cruciate ligament injury?

Answer: MRI.

One Step Further Question: If there is severe tenderness and swelling at the medial malleolus, which fracture must be suspected?

Answer: Maisonneuve fracture defined by fracture of the medial malleolus, disruption of the syndesmosis and fracture of the proximal fibula.

One Step Further Question: What concomitant knee injury should be evaluated for in patients with pes anserine bursitis?

Answer: Medial collateral ligament injury (strain or tear), due to its insertion site near the bursa.

One Step Further Question: Which nerve provides innervation to the ventral surface of the thumb, index, and long finger?

Answer: Median nerve.

One Step Further Question: What is the differential diagnosis for non-traumatic subungual hematoma?

Answer: Melanoma and Kaposi sarcoma.

One Step Further Question: Is polyarteritis nodosa more common in men or women?

Answer: Men.

One Step Further Question: What antibiotic can cause a disulfiram effect?

Answer: Metronidazole.

One Step Further Question: Which neurologic entity is commonly associated with rheumatologic diseases like polyarteritis nodosa?

Answer: Mononeuritis multiplex, a painful, asymmetrical sensorimotor peripheral neuropathy with numbness and weakness.

One Step Further Question: At which level do most lumbar disc herniations occur?

Answer: More than 95% of disk herniations occur at L4 or L5.

One Step Further Question: What is the treatment for gout in the acute phase?

Answer: Most commonly indomethacin, but any NSAID can be used.

One Step Further Question: What is the etiology of systemic lupus erythematous?

Answer: Multifactorial including genetics, environmental factors, hormones, race, medications and immunology

One Step Further Question: What are complications of flexor tenosynovitis?

Answer: Necrosis and proximal spread.

One Step Further Question: What birefringence is acute gout associated with?

Answer: Negative birefringence.

One Step Further Question: What are indications for nonoperative management of fingertip amputations?

Answer: No bone or tendon exposed and less than 2 cm of skin loss

One Step Further Question: Does the serum CK level predict the risk of acute renal failure?

Answer: No. CK level correlates well with degree of muscle injury but does not predict acute renal failure.

One Step Further Question: Heberden nodes refer to what physical examination finding?

Answer: Nodules on the distal interphalangeal joints of the hands associated with osteoarthritis.

One Step Further Question: What is the first-line pharmacologic treatment of ankylosing spondylitis?

Answer: Nonsteroidal anti-inflammatory medications.

One Step Further Question: What is the most common radiographic finding for a Salter-Harris Type V fracture?

Answer: Norma

One Step Further Question: How reliable are ESR and CRP in ruling out a septic joint?

Answer: Not very. ESR is normal in up to 30% of patients and CRP is normal in up to 12% of patients.

One Step Further Question: Which wrist ganglion is typically not aspirated?

Answer: One arising on the volar radial wrist, as its proximity to the radial artery makes aspiration risky.

One Step Further Question: Which exam is recommended every 6-12 months in those on hydroxychloroquine?

Answer: Ophthalmologic exam, due to this medication's potential adverse events of retinal deposition and retinopathy.

One Step Further Question: What signs and symptoms are consistent with a L5 radiculopathy?

Answer: Pain that radiates down the lateral aspect of the leg into the foot, weakness with foot dorsiflexion, and decreased sensation between the first and second toes.

One Step Further Question: What is a reverse straight leg test?

Answer: Pain with passive extension of the hip while the patient is prone. It is indicative of an L3 or L4 radiculopathy.

One Step Further Question: Surgical removal of what gland can cause a pseudogout flare?

Answer: Parathyroid gland.

One Step Further Question: What age groups are at higher risk for a malignant cause of their back pain?

Answer: Patients under 18 years or over 50 years of age.

One Step Further Question: In which patients should NSAIDs be avoided?

Answer: Patients with acid peptic ulcer disease, renal insufficiency, diabetes, CHF and a history of allergic reaction to NSAIDs.

One Step Further Question: Where are the subcutaneous nodules of acute rheumatic fever typically found?

Answer: Pea-sized and nontender, they are typically found over extensor surfaces of the wrists, elbows, and knees.

One Step Further Question: Which nutritional deficiency also presents with a malar rash?

Answer: Pellagra (vitamin B3 deficiency).

One Step Further Question: What disease is caused by niacin deficiency?

Answer: Pellagra.

One Step Further Question: Which medication provides systemic stimulation for patients with Sjögren's syndrome?

Answer: Pilocarpine.

One Step Further Question: What finding is seen on synovial fluid analysis in pseudogout?

Answer: Positively birefringent, rhomboid-shaped crystals.

One Step Further Question: Urate crystals come from the breakdown of foods that are rich in what?

Answer: Purines.

One Step Further Question: What are two common complications of untreated recurrent patellar instability?

Answer: Quadricep weakness and patellar arthrosis.

One Step Further Question: Which two arteries comprise the dual blood supply to the hand?

Answer: Radial and ulnar arteries.

One Step Further Question: What X-ray findings would suggest a rigid flatfoot secondary to a tarsal coalition?

Answer: Radiographs of the foot may show a bony bridge between the calcaneus and navicular or calcaneus and talus.

One Step Further Question: What is the difference between Raynaud's disease and Raynaud's phenomenon?

Answer: Raynaud's phenomenon is the term used for Raynaud's disease when there is a specific causative etiology. These included scleroderma, rheumatoid arthritis and systemic lupus erythematosus.

One Step Further Question: In CSF analysis, what does xanthochromia indicate?

Answer: Red blood cells in the subarachnoid space.

One Step Further Question: Why should fluoroquinolones be withheld in the treatment of patients with pneumonia and a suspicion for TB?

Answer: Respiratory fluoroquinolones are among the medications that may be used in the treatment of TB and may alter the results of sputum samples for acid-fast bacilli.

One Step Further Question: Pseudogout is often confused with which disease process secondary to its multi-joint presentation?

Answer: Rheumatoid arthritis, both have multiple joint involvement with symmetric distribution.

One Step Further Question: Which grading system is used to assess skeletal maturity?

Answer: Risser classification system.

One Step Further Question: What critical diagnoses should be considered in patients with syncope and back pain?

Answer: Ruptured abdominal aortic aneurysm, aortic dissection, pulmonary embolism and ruptured gastric/duodenal ulcer.

One Step Further Question: The term "Madonna fingers" refers to what physical finding in scleroderma?

Answer: Sclerosis and eventual tapering of the distal fingertips.

One Step Further Question: What other bacterial agents are commonly associated with reactive arthritis?

Answer: Shigella, Salmonella, Yersinia and Campylobacter organisms.

One Step Further Question: Which other condition mimics Osgood-Schlatter disease?

Answer: Sinding-Larsen-Johansson syndrome, a similar pathology in the inferior patellar pole of boys aged 9-11 years.

One Step Further Question: What is a relative contraindication for surgery in patients with unresolved Osgood-Schlatter disease?

Answer: Skeletal immaturity.

One Step Further Question: Name some neuromuscular causes of childhood secondary scoliosis?

Answer: Spina bifida, cerebral palsy, tethered cord syndrome, muscular dystrophy and spinal muscular atrophy.

One Step Further Question: Neonatal septic arthritis is commonly caused by which organisms?

Answer: Staphylococcus aureus and group-B Streptococcus. Haemophilus influenzae should be considered in childhood septic arthritis.

One Step Further Question: What is the most common cause of septic arthritis?

Answer: Staphylococcus aureus is the most common cause of septic arthritis in adults.

One Step Further Question: What is the most common infectious agent in acute osteomyelitis?

Answer: Staphylococcus aureus.

One Step Further Question: What is the most common type of bursitis?

Answer: Subacromial (subdeltoid) bursitis.

One Step Further Question: Which patellar pole is the most common site of osseous nonunion in bipartite patella?

Answer: Superolateral.

One Step Further Question: The greater tuberosity is the insertion point of which 3 rotator cuff muscles?

Answer: Supraspinatus, infraspinatus and teres minor.

What medication should be first line treatment in moderate musculoskeletal back pain? Acetaminophen Cyclobenzaprine Diazepam Oxycodone

Correct Answer ( A ) Explanation: The first choice for patients with mild to moderate back pain should be acetaminophen or a nonsteroidal anti-inflammatory drug (NSAID) like ibuprofen or naprosyn. In patients with severe pain, opiate medications may be necessary. However, it is important to note that no studies have demonstrated the superiority of one pain medication (or class of medication) over another in the treatment of back pain. Patients with milder symptoms should initially be treated with acetaminophen or an NSAID. These medications are well tolerated in short courses with minor side effects. NSAIDs have not been shown to be superior to acetaminophen.

A 20-year-old man presents complaining of a painful swollen finger. On examination, you note the findings seen in the image above. Which of the following is true regarding this condition and its treatment? An 11-blade scalpel or 18-gauge needle should be inserted into the eponychium and paronychium parallel to the nail until pus begins to drain Incision should be made on the palmar aspect of the finger pulp and then the wound should be packed with gauze It is caused by herpes simplex virus and therefore incision and drainage should be avoided to prevent spread, secondary infection and delayed healing The nail must always be removed to allow appropriate abscess drainage

Correct Answer ( A ) Explanation: A paronychia is an infection of the soft tissues surrounding of the nail root of a finger or toe. Initially a mild cellulitis develops that ultimately progresses to an abscess under the eponychium. Standard therapy includes incision and drainage, packing and possibly oral antibiotic therapy. Drainage is performed by insertion of an 11-blade scalpel or 18-gauge needle into the eponychium and parinychium parallel to the nail until pus begins to drain. The abscess cavity should be opened completely with hemostats or small scissors. The cavity is then packed and the patient is discharged with 48-hour follow-up. If not treated early, a subungual abscess can form and purulent matter accumulates under the nail itself.

Which of the following is a common cause of non-traumatic avascular necrosis of the hip? Cholecystitis Chronic corticosteroid use Chronic marijuana use Iron deficiency anemia

Correct Answer ( B ) Explanation: Although a specific causative disorder is not identified in 20% of the cases, known atraumatic causes include chronic corticosteroid therapy, chronic alcoholism, hemoglobinopathy (e.g., sickle cell anemia), and chronic pancreatitis. When a patient has an increasingly painful hip, buttock, thigh, or knee and no history of recent trauma, avascular necrosis (AVN) of the femoral head should be considered. AVN is bilateral in 40 to 80% of patients. It is common in relatively young patients, the mean age at diagnosis is 38 years. AVN also is an emerging complication associated with human immunodeficiency virus (HIV) infection. It is unclear whether the virus itself or the treatments are the pathogenic agents.

A 25-year-old woman comes to the office with diffuse pain, multiple tender points and generalized fatigue. A full work up revealed no connective tissue disease but you suspect fibromyalgia. Which of the following can be used for treatment of fibromyalgia? Aerobic exercise High dose opioids Physical therapy Trigger point injections

Correct Answer ( A ) Explanation: Aerobic exercise increases cardiovascular fitness and reduces pain and other fibromyalgia symptoms. Fibromyalgia is characterized by high levels of pain, sleep disturbance, and fatigue combined with a general increase in medical symptoms including problems of memory or thinking and often psychological distress. Many issues that surround fibromyalgia are not scientific ones—it is widely agreed that pain and suffering are real; instead, the primary issues are often social, political, and financial. Fibromyalgia lies at the end of a continuum of polysymptomatic distress rather than being a discrete disorder. Fibromyalgia may be diagnosed with American College of Rheumatology (ACR) 2010 or 1990 criteria, but clinical care does not require a diagnosis. The ACR 2010 criteria should result in changes in the sex ratio of patients with fibromyalgia because men have higher pain thresholds and are therefore less likely to be diagnosed as having fibromyalgia than women when the 1990 criteria including tender points are used.

In which setting is allopurinol contraindicated for the treatment of gout? Acute attack of gout Alcoholic patient Hyperuricemia greater than 8.0 In combination with colchicine

Correct Answer ( A ) Explanation: Allopurinol therapy should never be initiated until an acute attack has subsided. Allopurinol is a xanthine oxidase inhibitor that decreases uric acid production but also produces a more soluble metabolite. Allopurinol is therefore effective regardless of the cause of the hyperuricemia. Gout is caused by deposition of uric acid crystals in the synovium, bursae, tendon sheaths, skin, heart valves, and kidneys, which can lead to arthritis, tophi, renal stones, and gouty nephropathy. Typically, patients present with acute, exquisitely tender, monoarticular arthritis. The joint is usually warm and erythematous. The first metatarsophalangeal joint (MTP) joint is most commonly affected. Detecting negatively birefringent, needle-like crystals in the synovial fluid from an arthrocentesis confirms the diagnosis. First line treatment are NSAIDs. Indomethacin has traditionally been the NSAID of choice, but any NSAID can be used with similar efficacy.

Which of the following wounds is a potential candidate for primary closure? Facial laceration from a dog bite Laceration on sole of the foot after stepping on an unknown object in a stream Laceration over the metacarpal-phalangeal joint sustained during a fistfight Puncture wounds on the leg as a result of a dog bite

Correct Answer ( A ) Explanation: Although dog bite wounds are high risk for infectious complications, those that are cosmetically deforming are often closed primarily, especially on the face. Dog bites are high risk for infection from organisms commonly found in the dog's mouth, which include S. aureus, P. multocida, and S. viridans. Although controversy exists over management of animal bite wounds, generally it is agreed that wounds that are cosmetically deforming (including those on the face) should be repaired primarily. All bite wounds must be thoroughly irrigated and cleansed. Wounds on the extremities and that are not as cosmetically concerning are generally left open or loosely approximated.

A 29-year-old man presents with progressive back pain and stiffness that started 2 months ago. The pain is worse at night and when he wakes up in the morning and improves with exercise. In the last week he has also noted pain and stiffness in the hips and ankles. Neurological examination is unremarkable. What is the likely diagnosis? Ankylosing spondylitis Lumbar radiculopathy Rheumatoid arthritis Spinal stenosis

Correct Answer ( A ) Explanation: Ankylosing spondylitis (AS) is a chronic inflammatory condition primarily affecting the spine and pelvis. There is a strong association between the presence of human leukocyte antigen (HLA) B-27 and AS. Patients often present in their 20s to 30s with a 3:1 male predilection. Symptoms consistent with AS include an insidious onset of pain and stiffness (often present for 3 months or more) that is worse at night and the morning, but improves with mild activity. Pain typically affects the lower spine and hips, but can involve peripheral joints as well, most notably the ankles and knees. Another classic finding is pain, stiffness, and tenderness at the insertion sites of ligaments and tendons to the bone (enthesitis). This is most commonly seen at the achilles tendon attachment to the calcaneus and the plantar fascia. Diagnosis is made based on history and physical findings, presence of HLA B-27, and radiographs consistent with disease (e.g. erosions or fusion of sacroiliac joints). First-line treatment is nonsteroidal anti-inflammatory medications.

What test is helpful in ruling out the diagnosis of Systemic lupus erythematosus? Anti-nuclear antibody Anti-Sm antibody C-reactive protein (CRP) Erythrocyte sedimentation rate (ESR)

Correct Answer ( A ) Explanation: Antinuclear antibody (ANA) is highly sensitive as it is present in nearly all patients with systemic lupus erythematosus (SLE). SLE is a multisystem, autoimmune disorder. The disease results in organ inflammation, dysfunction and eventually destruction. Patients can present with nephritis, thrombosis, carditis, pneumonitis, pulmonary hypertension, cerebrovascular accident or infection resulting from chronic immunomodulatory medications. The workup of patients with signs or symptoms concerning for SLE involves a number of serum tests all of which are imperfect. However, nearly all patients with the disease will have antinuclear antibody present on testing. The absence of this antibody virtually rules the disease out. Unfortunately, it is also present in up to 50% of patients without SLE and a positive test does not clinch the diagnosis. Anti-Sm antibody (B) is a more specific antibody for SLE but it is not present in all patients with the disease. CRP (C) does not typically rise in response to increased disease activity and may be normal in 10-15% of patients. ESR (D) is frequently elevated in SLE but may be normal in up to 30% of patients.

A 29-year-old woman presents to your clinic for fatigue and myalgias that have been worsening over the past few months. In addition to daily headaches, she feels chronic aching and stiffness in her back, arms, and legs. On exam she has distinct tender points at her buttocks, trapezius, lateral elbow epicondyles, and medial aspects of her knees. Strength testing is 5/5 throughout. An erythrocyte sedimentation rate, rheumatoid factor, TSH, and serum phosphate level all return within normal limits. Which of the following conditions does she most likely have? Fibromyalgia Oncogenic osteomalacia Polymyalgia rheumatica Polymyositis

Correct Answer ( A ) Explanation: Fibromyalgia is characterized by chronic, widespread musculoskeletal pain and stiffness with tenderness at discrete "trigger points" on physical exam. Points include the trapezius, medial fat pads of the knees, and lateral epicondyles of the elbows. A review of systems will usually show fatigue, frequent headaches, sleep disorders, and subjective numbness; co-existing depression and irritable bowel symptoms are also common. Fibromyalgia is a diagnosis of exclusion, and requires ruling out other rheumatologic conditions including systemic lupus erythematous and rheumatoid arthritis, as well as underlying hypothyroidism. Treatment is multifaceted. Patient education should emphasize that the disease is not progressive; exercise programs and cognitive behavioral therapy may be beneficial. Many medications (including amitriptyline, duloxetine, gabapentin, and muscle relaxants) show modest symptom relief; corticosteroids and opioids should be avoided as these have not be proven effective. Polymyalgia rheumatica (C) causes musculoskeletal pain that is concentrated in the shoulder and pelvic girdle. It is most common in patients over 50, and is usually associated with an anemia of chronic disease and an elevated ESR. Polymyositis (D) is more likely to produce proximal muscle weakness, which this patients does not have. Additionally, though a quarter of patients with polymyositis may have concurrent muscle pain and tenderness, this is rarely the chief complaint.

A 27-year-old man presents after a fall. He struck his leg on a table and sustained a fracture of the fibular neck. Which of the following nerves is commonly injured in this type of injury? Common peroneal Femoral Posterior tibial Saphenous

Correct Answer ( A ) Explanation: Fractures of the fibula usually occur as a result of direct blunt trauma or varus stress to the knee. Isolated fractures of the fibula are frequently inconsequential because the fibula is almost completely a non-weightbearing bone. The exception to this is the significant injury of a Maisonneuve fracture when the proximal fibular fracture is associated with disruption of the medial portion of the ankle joint. The common peroneal nerve passes around the neck of the fibula and its function must be evaluated in the setting of a proximal fibula fracture. The common peroneal nerve is a branch of the sciatic nerve and divides into the deep and superficial branches. These nerves provide sensation to the anterior and lateral portion of the leg and to the top of the feet. The motor innervation is for dorsiflexion of the foot. As a result, a patient with a peroneal nerve injury will develop a foot drop.

A 58-year-old woman presents to your office with complaints of headache, a temperature of 101.1°F, anorexia, and morning stiffness in her hips and shoulders. She indicates that this morning her vision was blurry, whereas yesterday it was normal. Which of the following is the most appropriate next step in management? Administration of 60 mg of prednisone and referral to ophthalmology Administration of 800 mg of ibuprofen and referral to neurology Administration of 800 mg of ibuprofen and referral to rheumatology Supportive care only

Correct Answer ( A ) Explanation: Giant cell arteritis is a vascular syndrome characterized by inflammation of medium and large-sized arteries. Also called temporal arteritis, this vasculitis involves arteries in multiple locations including the temporal artery. Clinical manifestations include headache, fever, polymyalgia rheumatica, systemic inflammation, anemia, and elevated erythrocyte sedimentation rate. Risk factors include age greater than 50 years, female sex and Scandinavian background. The primary goals of treatment are to reduce systemic symptoms and prevent vision loss, therefore immediate administration of high-dose prednisone with referral to ophthalmology is indicated. Diagnosis is confirmed by temporal artery biopsy, however treatment should be administered as soon as symptoms occur and not wait for biopsy results.

An overweight 38-year-old man presents with severe left foot pain that began acutely 2 days ago. He denies recent injury and has no history of surgery to the ankle. He has a past medical history of hypertension. His current medications include lisinopril-hydrochlorothiazide. His vital signs are 99°F, BP 148/86, and HR 92. He works as a train conductor. His job requires a lot of jumping on and off the train. Otherwise, he rarely exercises and has a poor diet. He has had 5 sexual partners in the last 12 months, and uses barrier-protection. He had gonorrhea once in his 20s but has no penile discharge currently. He has not had any recent skin infections. He has never used intravenous drugs. He is not having issues with any other joints. He has a family history positive for osteoarthritis in his maternal grandmother and paternal grandfather, but there is no history of rheumatoid disorders. Physical exam reveals a warm great toe that is moderately swollen, erythematous, and extremely tender to the touch at the metatarsalphalangeal joint. What is the most likely diagnosis? Gout Navicular fracture Osteoarthritis Septic arthritis

Correct Answer ( A ) Explanation: Gout is crystal-induced arthropathy caused by needle-like urate crystals. These crystals are formed most commonly in the metatarsalphalangeal joint of the big toe, ankle, wrist, and knee. The crystals develop as purines break down into uric acid. The crystals develop in peripheral joints because these are cooler areas of the body and the crystals are less soluble at lower temperatures. Most purines come from dietary intake of purine-rich foods (e.g., liver, kidney, anchovies, asparagus, consommé, herring, meat gravies and broths, mushrooms, mussels, sardines, sweetbreads). Certain comorbid conditions are associated with a higher incidence of gout, including hypertension, diabetes mellitus, renal insufficiency, hypertriglyceridemia, hypercholesterolemia, obesity, and anemia. Use of diuretics also increases the risk of attacks. Attacks are usually monoarticular, and begin abruptly. Attacks usually reach maximum intensity in 8-12 hours. The affected joint becomes swollen, warm, erythematous and exquisitely tender. Gout is confirmed by identifying uniquely polarized (birefringent) crystals from the joint aspirate. Navicular fractures (B) primarily occur from sports related activities, such as running, or marching. The navicular helps maintain the medial longitudinal arch of the foot, and is important for absorbing the heel strike as well as enabling the push-off during walking. In this type of injury, the pain would be mid-foot rather than in the toe, and it would be unlikely to be accompanied by significant erythema or warmth.

Lateral epicondylitis involves inflammation at the attachment of which of the following muscles? Extensor carpi radialis brevis Flexor carpi radialis Pronator teres Triceps

Correct Answer ( A ) Explanation: Lateral epicondylitis, commonly referred to as tennis elbow, is an inflammatory condition involving the lateral epicondyle of the humerus. The lateral epicondyle is the site of origin for the wrist and digit extensors and the forearm supinators. Muscles that originate at the lateral epicondyle include the extensor carpi radialis brevis, extensor digitorum, extensor digiti minimi, and the extensor carpi ulnaris. Lateral epicondylitis is often the result of repetitive supination of the forearm. Patients present with focal tenderness over the lateral epicondyle as well as proximal wrist extensor muscle mass pain with resisted wrist extension with the elbow in full extension, and pain with passive wrist flexion with the elbow in full extension. Treatment includes nonsteroidal anti-inflammatory medications and activity modification. A counterforce brace placed distal to the elbow and physical therapy is recommended as well.

An elderly man presents with 5 months of bruising, back pain, progressive leg weakness and distal leg paresthesias. Examination reveals flaccid weakness and areflexia in the legs. Laboratory testing reveals hypercalcemia, thrombocytopenia, anemia and elevated serum creatinine. A lumbar radiograph shows an L4 vertebral body fracture. Which of the following is the most likely diagnosis? Multiple myeloma Myelomeningocele Myelopathy Myositis ossificans

Correct Answer ( A ) Explanation: Multiple myeloma is a malignant proliferation of plasma cells which produces excessive monoclonal paraproteins. Symptoms include bone pain, pathologic fractures, recurrent infections, spinal cord compression, malaise, weakness, anemia, bleeding, renal failure and hypercalcemia. Patients may be asymptomatic, and diagnosis may be incidental. Presentation often includes bone pain (commonly associated with atraumatic pathologic axial spine fractures) and abnormal laboratory results (especially abnormal serum/urine protein electrophoresis, anemia and hypercalcemia). Associated findings include macular detachment, retinal hemorrhage or cotton-wool spots, ecchymoses and pallor, cardiomegaly, hepatosplenomegaly and carpal tunnel syndrome. Bone and PET scanning, as well as plain radiography and MRI, are routinely used in the diagnostic work-up. Myelomeningocele (B), a form of spina bifida, is a congenital disorder. This neural tube defect is marked by failure of closure of the posterior spinal elements. It results in open protrusion of the meninges and spinal cord. Myelopathy (C) is a general term for spinal cord injury. It has several etiologies, of which, cervical spine injury is quite common. Myelopathy is the constellation of upper motor neuron signs (spastic weakness, spasticity, hyperreflexia, positive Babinski and Hoffman's signs), paresthesias and numbness and genitourinary dysfunction. Myositis ossificans (D), also known as heterotopic ossification, involves soft tissue calcification, typically about a joint which has suffered trauma, immobilization or surgery. The quadriceps and gluteals are commonly affected sites.

You diagnose a patient with myofascial pain syndrome. In an effort to decrease her overall pain levels, which of the following medications is most appropriate to prescribe? Cyclobenzaprine Haloperidol Oxycontin Pregabalin

Correct Answer ( A ) Explanation: Myofascial pain can essentially be defined as the presence of trigger points, focal distinct painful spots located in palpable taut bands of muscles. It is a common musculoskeletal cause of pain, and can be associated with local and distal sensory abnormalities as well as autonomic, sleep and mood disorders. Treatment begins with aggressive physical therapy, which includes specific treatments of myofascial release, ultrasound, spray-and-stretch techniques, posture rebalancing and specific muscle stretch-strengthen exercises. Further treatment options include cognitive-behavior therapy and trigger point injection therapy. Medications also play a role. Muscle relaxants, such as cyclobenzaprine, provide pain relief. NSAIDs, only when used in conjunction with other active treatment, are also beneficial for pain relief.

A 12-year-old boy is brought to his pediatrician's office for right knee pain that started three months ago after starting basketball practices. On physical exam, the boy is tender to palpation over the anterior tibial tubercle. There is no erythema or warmth to the area. A lateral radiograph of the knee reveals elevation of the tibial tubercle away from the shaft. Based on the likely diagnosis, which of the following is the most appropriate management? Acetaminophen and continued activity as tolerated Activity avoidance until pain resolution Knee immobilization and crutches Referral to orthopedics for glucocorticoid injection

Correct Answer ( A ) Explanation: Osgood-Schlatter disease is best managed with acetaminophen and continued activity as tolerated. Osgood-Schlatter disease is a traction apophysitis of the proximal tibial tubercle at the point of patellar tendon insertion. Osgood-Schlatter disease is caused by repetitive quadriceps contraction, which results in excessive patellar tendon traction on the skeletally immature tibial tendon. Osgood-Schlatter disease is most commonly seen in 9-14 year old children who have undergone a rapid growth spurt. It is more common in children and adolescents who are participant in sports. Osgood-Schlatter disease is more common in athletes who participate in sports that involve running, cutting, and jumping. Anterior knee pain that gradually worsens over time is the most common presenting complaint. Physical exam findings typically include tenderness and soft tissue or bony prominence of the tibial tubercle. The pain can be exacerbated by having the patient extend the affected knee against resistance or squatting with the knee in full flexion. Erythema and warmth are not typical findings and warrant further evaluation for more serious conditions, such as osteomyelitis or malignancy. Radiographs are not necessary in patients with clinical findings characteristic of Osgood-Schlatter disease, but may be used to exclude other conditions. Radiographic findings findings may include anterior soft tissue swelling or tibial tubercle abnormalities. Because Osgood-Schlatter disease is usually benign and self-limited, conservative measures are the mainstay of treatment. Conservative treatment includes pain control, continuation of activity, and physical therapy. Non-steroidal anti-inflammatory drugs (NSAIDs), acetaminophen, and ice may help control the pain. Patients should continue to participate in activity as tolerated. Surgical treatment is reserved for patients with persistent pain despite conservative management. The course of Osgood-Schlatter disease usually lasts 6-18 months. Osgood-Schlatter disease rarely causes permanent impairment or disability.

A 55-year-old man presents with new and insidious onset groin pain that began 5 months ago. His medical history includes sickle cell anemia. He currently abuses alcohol. Examination reveals painful hip rotation and decreased range of motion. There is no midline pelvic pain with resisted hip adduction. Pelvic radiographs reveal mild subchondral sclerosis about the femoral head, but no joint space narrowing or osteophytosis. Which of the following is the most likely diagnosis? Avascular osteonecrosis Legg-Calve-Perthes disease Osteitis pubis Osteoarthritis

Correct Answer ( A ) Explanation: Osteonecrosis of the femoral head is usually caused by a traumatic disruption of its blood supply or poor circulation due to other diseases. It most commonly occurs bilaterally and in those aged 40-60 years. Risk factors include a history of hip dislocation or femoral fracture, rheumatoid arthritis, systemic lupus erythematosus, Crohn's disease, chronic steroid or alcohol use, myeloproliferative disorders and sickle cell disease. Insidious onset of deep groin, hip or buttock pain predominates, however, acute presentations occur with acute collapse of the necrotic femoral head. Examination typically reveals antalgic gait and decreased and painful hip flexion, rotation and abduction. Radiographs early in the disease may be normal, or may reveal the earliest radiographical sign of this disease, subchondral femoral head sclerosis. MRI is necessary when the diagnosis is strongly suspected and the initial radiographs are normal or only positive for subchondral femoral head sclerosis. Pre-collapse treatment includes risk factor modification, physical therapy and pain management. Hip arthroplasty is typically reserved for function limiting head collapse. Secondary osteoarthritis is a common disease complication.

A 30-year-old man presents with asymmetric myalgias and arthralgias. He also complains of difficulty climbing stairs. You note fever, hip, and shoulder muscle weakness but no atrophy, scattered extremity numbness, and tender palpable purpura. However, there is no facial or truncal rash. Laboratory testing reveals a low hematocrit, a high creatinine kinase, a negative antinuclear antibody titer, and an elevated erythrocyte sedimentation rate. Which of the following is the most likely diagnosis? Polyarteritis nodosa Polymyalgia rheumatica Polymyositis Pseudogout

Correct Answer ( A ) Explanation: Polyarteritis nodosa is a systemic vasculitis of small to medium vessels. It represents an autoimmune inflammatory disorder of unknown origin. It results in transmural fibrinoid necrosis. It typically affects younger males. Symptoms include proximal myalgia and weakness (similar to polymyalgia rheumatica, and commonly expressed as difficulty climbing stairs), arthralgias, tender subcutaneous palpable nodules, abdominal pain with diarrhea or GI bleeding and glomerular ischemia with hypertension, renal failure, and hematuria. Compared to other vasculitides, polyarteritis nodosa does not affect the pulmonary vasculature, however it is accompanied by systemic inflammatory signs such as low grade fever, malaise, unintentional weight loss, and night sweats. Treatment includes prednisone and cyclophosphamide.

A 60-year-old woman presents with neck and chest rash. Review of systems is significant for proximal weakness and myalgias of 3 months duration. Examination of the skin reveals a violet-hue on both eyelids, reddened macules on the neck, shoulders and chest and thick, scaly skin on the dorsal surfaces of the metacarpophalangeal and proximal interphalangeal joints. However, there are no tender subcutaneous nodules. You send the patient for electromyography and nerve conduction testing. You would expect findings most consistent with which of the following pathologies? Myopathy Neuropathy Retinopathy Vasculopathy

Correct Answer ( A ) Explanation: Polymyositis is an idiopathic inflammatory myopathy. It causes proximal myalgias and weakness, similar to polyarteritis nodosa and polymyalgia rheumatica, which can present as pelvic and shoulder girdle functional impairments. It most often presents in females over age 50 years. In addition, pharyngolaryngeal weakness occurs, and manifests as dyspnea, dysphagia and dysphonia. Three classic skin findings commonly coexist with polymyositis. A heliotrope rash (symmetric violet-erythema, located periorbitally, especially about the eyelids), a cloak-like or cape rash (erythematous confluent macules on the neck, shoulders and chest), and Gottron's papules (scaly, violaceous thickened knuckle skin, sometimes confused with psoriasis). When these are present, the syndrome can be designated polymyositis-dermatomyositis (dermatomyositis). Lab testing usually reveals a positive ANA and high creatinine kinase. The classic EMG findings are those of myopathy: positive sharp waves, fibrillations, complex repetitive discharges and small amplitude, short duration, polyphasic motor units with an early recruitment pattern. Treatment options include prednisone, methotrexate and cyclophosphamide.

A 44-year-old man presents with insidious and atraumatic proximal lower leg pain. It is more tender during rest than exercise. It commonly occurs at night, and doesn't seem to improve with oral analgesics. Knee testing is negative for meniscal or ligamentous injury. A screening radiograph reveals no fracture, but does show a non-speckled calcified lesion within the proximal tibia. A bone scan shows no spine or pelvic lytic lesions. Which of the following is the most likely diagnosis? Chondrosarcoma Enchondroma Multiple myeloma Osteoid osteoma

Correct Answer ( A ) Explanation: Primary malignant bone tumors are rare and benign bone tumors are common. However, metastatic bone tumors are very common, and skeletal metastasis must be strongly considered in any patient over 40 years of age with any bony lesion. The most common bone tumors in those over 40 years of age are metastases, chondrosarcoma, fibrosarcoma and multiple myeloma. Constant deep achy pain, especially in the nighttime hours and not relieved by rest, is the main symptom of bone malignancy. Large benign tumors may present in a similar fashion. Mass is a less common presenting complaint, but may suggest a benign condition especially if it is nontender. Chondrosarcoma and fibrosarcoma are common bone malignancies in adult metaphyseal bones. Osteosarcoma also has a metaphyseal predilection, however, it occurs with lytic and blastic lesions in the second decade, with chondrosarcoma having calcified lesions and fibrosarcoma having lytic lesions. Radiographic examination is necessary. CT is best used in evaluating benign bone lesions. MRI is better for evaluating malignant bone lesions. Bone scans are used to detect other sites of skeletal lesions. A chest radiograph and CT are usually ordered if a malignant bone tumor is suspected. Routine laboratory testing is of limited use, however, in those over 40 years of age, consider urinalysis, urine and serum protein electrophoresis and PSA testing. Benign tumors may be observed or excised, while malignant tumors are excised and treated with chemotherapy and radiotherapy Enchondroma (B) is a benign cartilaginous tumor. It appears as speckled calcifications within the phalanges and metacarpals of young adults. Multiple myeloma (C) most commonly occurs in those over 40 years of age. Lytic lesions of the spine and pelvis are key findings. Osteoid osteoma (D) is a childhood and teenage benign bone lesion, commonly associated with night pain that is relieved with NSAIDs.

A 44-year-old woman presents with 3-month history of Raynaud's phenomenon and complains of puffy hands and fingers and of arthralgias. On exam you notice skin thickening proximal to the metacarpalphalangeal joints and fingertip-pitting scars. On chest X-ray there are findings suggestive of chronic pulmonary interstitial changes. Which of the following is the most likely diagnosis? Churg-Strauss syndrome Diffuse Fasciitis with eosinophilia Granulomatosis with polyangiitis Scleroderma (systemic sclerosis)

Correct Answer ( D ) Explanation: Scleroderma or hardening of the skin, is an autoimmune disorder now recognized to involve almost every organ system and is therefore more appropriately referred to as systemic sclerosis. Systemic sclerosis is characterized by a progressive fibrosis of the skin, blood vessels, lungs, kidneys, heart, and GI tract. The degree of skin involvement (diffuse or limited) is useful for prognosis. Diffuse cutaneous systemic sclerosis (DCSS) involves almost the entire body, including the trunk, face, neck, and extremities. Cardiopulmonary disease is the leading cause of death in DCSS. Limited cutaneous systemic sclerosis has a more benign course and involves the face, neck, and distal extremities below the knee and elbow. Systemic sclerosis affects women up to eight times more often than men; peak incidence is in the 40s and 50s. The major American College of Rheumatology (ACR) classification criterion for systemic sclerosis is skin thickening proximal to the MCP or MTP joints. The three minor ACR criteria are involvement distal to the MCP joints (sclerodactyly), fingertip-pitting scars or loss of subcutaneous tissue, and chronic pulmonary interstitial changes. The vast majority (>90%) of systemic sclerosis patients have Raynaud's phenomenon, which is the initial complaint in most patients, although some might complain first of puffy hands and fingers and of arthralgias. The syndrome is also characterized by CREST syndrome (Calcinosis, Raynaud phenomenon, Esophageal dysmotility, Sclerodactyly, and Telangiectasias. Churg-Strauss syndrome (A) is a rare systemic vasculitis involving the lungs, skin, peripheral nerves, GI tract, and kidneys. It is seen in patients with asthma and allergies, most often in middle age, and in men. Asthma and allergy symptoms become exacerbated, followed by a peripheral eosinophilia and vasculitis. Urinalysis shows red blood cell casts and proteinuria; chest radiographs show diffuse interstitial disease or multiple nodules. Diffuse fasciitis with eosinophilia (DFE) (B), also known as eosinophilic fasciitis or Shulman's syndrome, is a relatively rare disorder that involves the trunk and limbs but spares the hands and face. There is no organ involvement or Raynaud's phenomenon associated with DFE. Diagnosis is made by deep biopsy involving muscle, subcutaneous tissue, and skin. DFE can be associated with hematologic malignancies. Monitoring for these is therefore prudent if the cause is unknown. Granulomatosis with polyangiitis (C) (formerly known as Wegener's granulomatosis) is a small-vessel, necrotizing, granulomatous vasculitis involving the upper and lower respiratory tracts and the kidneys. These three locations are not always involved, however, and other areas, such as skin (purpura, nodules, ulcers) and eyes (uveitis, episcleritis), might be affected. Wegener's granulomatosis can occur at any age but most often occurs between ages 30 and 50. Patients might present with epistaxis, sinus pain, nasal or oral ulcers, and nasal septal perforation. Cavitary lesions can be seen on radiographs.

Which of the following is used to diagnose and monitor scoliosis? Cobb angle Hoffman reflex Modified Schober test Pelvic tilt

Correct Answer ( A ) Explanation: Scoliosis is lateral curvature of the thoracic or lumbar spine > 10 degrees. It is usually idiopathic, but can be secondary to neuromuscular disease (cerebral palsy, spinal muscular atrophy, myelomeningocele), vertebral disease (tumor, infection), neurofibromatosis, or Marfan syndrome. Symptoms usually develop in early adolescence. Females are more likely to have progressive disease requiring treatment. Adult onset scoliosis is commonly caused by spinal degenerative disk disease, facet spondylosis and spondylolisthesis. On exam, use a forward bending test to assess for vertebral and rib rotation. The diagnosis is made radiographically with weight-bearing PA and lateral full-length spinal views to measure the Cobb angle. The Cobb angle is used to quantify the magnitude of spinal deformities. To measure the Cobb angle, first decide which vertebrae are the end-vertebrae of the curve deformity (vertebrae at the upper and lower limits of the curve) and then Cobb angle formed by the intersection of two lines: one parallel to the endplate of the superior end vertebra and the other parallel to the endplate of the inferior end vertebra. The angle is plotted and scoliosis is defined as a lateral spinal curvature with a Cobb angle of 10° or more. The Hoffman reflex (B) involves assessing for thumb or second digit flexion when the nail of the third digit is flicked. A positive response suggests upper motor neuron disease such as cervical spinal cord injury. The modified Schober test (C) is used to quantify lumbar spine flexion and extension in the sagittal plane. It is not used to monitor worsening coronal spine curve as in progressive scoliosis. Pelvic tilt (D) refers to the height difference between the iliac crests. It is primarily used to evaluate leg-length discrepancy.

A 67-year-old man with a history of gout presents with atraumatic left knee pain. Physical examination reveals an effusion with overlying warmth and erythema. There is pain with passive range of motion. He reports a history of gout in this joint in the past. What is the appropriate next step? Arthrocentesis of the knee Indomethacin for acute gout Measurement of serum uric acid Radiograph of the knee

Correct Answer ( A ) Explanation: Septic arthritis is a bacterial or fungal infection of a joint typically spread hematogenously unless there is direct bacterial contamination. The synovium is highly vascular and lacks a basement membrane making it susceptible to bacterial seeding. Certain conditions predispose individuals to septic arthritis including diabetes, sickle cell disease, immunocompromise, alcoholism or pre-existing joint disease like rheumatoid arthritis or gout. Fever is present in less than half of cases of septic arthritis so with clinical suspicion an arthrocentesis is indicated. The knee is the most common joint affected and patients have pain (especially on passive range of motion) and decreased range of motion often accompanied by warmth, erythema and fever. This patient may have an acute gouty flare, but the clinician must exclude an infection. On joint fluid analysis, the white blood cell count of a septic joint is typically > 50,000.

In which of the following spondyloarthropathies is uveitis the most common extra-articular manifestation? Ankylosing spondylitis Fibromyalgia Psoriatic arthritis Reactive arthritis

Correct Answer ( A ) Explanation: Seronegative spondyloarthropathies share the characteristics of sacroiliac involvement, peripheral inflammatory arthropathy, an absence of rheumatoid factor, ligamentous and tendinous changes, and a genetic component related to the HLA-B27 marker. Patients with ankylosing spondylitis typically have back discomfort, with radiographic evidence of sacroiliitis. Uveitis is the most common extra-articular manifestation. Aortic root disease may also occur. Fibromyalgia (B) is characterized by idiopathic widespread pain with no associated extra-articular manifestations. Psoriatic arthritis (C) occurs in up to 20% of patients with psoriasis. Other than psoriasis, patients exhibit asymmetrical oligoarthropathy or symmetrical polyarthropathy, spondylitis, and arthritis mutilans. Reactive arthritis (Reiter's) (D) most commonly occurs in young men after an episode of urethritis or dysentery. It is characterized by asymmetrical polyarthritis, conjunctivitis that can progress to uveitis, and painless oral mucosal lesions that develop into painful ulcers.

Which of the following is the most common type of shoulder dislocation? Anterior Inferior Posterior Superior

Correct Answer ( A ) Explanation: Shoulder dislocations are very common, due in part to the inherent instability of the shoulder joint. Anterior dislocations are most common and make up 95-97% of all shoulder dislocations. The mechanism of injury is generally due to a blow to the arm that is abducted, externally rotated, and extended. Sports activities such as blocking a basketball shot, motor vehicle accidents and a fall on an outstretched arm can all lead to anterior shoulder dislocation. Treatment involves reduction of the shoulder, immobilization and referral to an orthopedic specialist within one week of the injury

A 16-year-old girl presents to the clinic with complaints of left knee pain, instability and swelling after sustaining a twisting injury to the knee ten days ago during a soccer game. Which of the following physical exam findings would be most consistent with an anterior cruciate ligament tear? Positive Lachman test Positive McMurray test Positive posterior drawer Posterior sag sign

Correct Answer ( A ) Explanation: The Lachman test is performed with the patient supine. The knee is flexed to 20-30 degrees and an anterior force is placed on the tibia. The examiner looks for laxity or increased tibial translation compared to the contralateral knee. The Lachman test is described as grade 1 if there is 0 - 5 mm of anterior tibial translation, grade 2 if there is 6 - 10 mm of translation, or grade 3 if there is greater than 10 mm of translation. There is also a modifier for the presence or lack of an endpoint (i.e., A = good endpoint; B = no endpoint). Other physical exam tests that can be used to detect the presence of an anterior cruciate ligament (ACL) tear include the anterior drawer test and the pivot shift test. To perform the anterior drawer test, the knee is flexed to 90 degrees and an anterior force is placed on the tibia. Laxity or increased anterior tibial translation is a positive test. The pivot shift test is performed with the patient supine. The knee is extended and internally rotated. The examiner places a valgus stress on the knee and moves the knee from extension to flexion. The examiner will feel a clunk in the knee at approximately 20 - 30 degrees of flexion in an ACL-deficient patient as the tibia reduces from its subluxated position Both the posterior drawer test (C) and posterior sag sign (D) are used to assess the posterior cruciate ligament (PCL). For the posterior drawer test, the knee is flexed to 90 degrees and a posterior force is placed on the tibia. Laxity or posterior tibial translation is a positive test. To look for a posterior sag sign, the patient is examined in the supine position. The hip is flexed to 45 degrees and the knee is flexed to 90 degrees with the patient's foot on the exam table. Increased posterior translation of the tibia on the femur compared to the contralateral side, as a result of gravity, indicates a PCL injury.

Which of the following tests is the most accurate physical examination maneuver to identity an injury to the anterior cruciate ligament? Lachman's test McMurray's test Thompson test Valgus stress

Correct Answer ( A ) Explanation: The anterior cruciate ligament (ACL) is one of the four major ligaments of the knee (anterior cruciate, posterior cruciate, medial collateral, and lateral collateral). It provides anterior stability to the knee joint and is the most commonly injured ligament of the knee. The Lachman's test is the most accurate test for the ACL. In this test, the knee is flexed to 20 or 30 degrees and the thigh is stabilized. The tibia is then pulled anteriorly to look for abnormal excursion.

A 42-year-old man complains of difficulty washing his face and combing his hair with his right hand. On examination a nodule, band, and slight contracture are noted in the palm proximal to the fourth finger. This patient's symptoms are associated with which one of the following? ADiabetes mellitus BHyperparathyroidism CHyperthyroidism DHypothyroidism

Correct Answer ( A ) Explanation: The patient has Dupuytren's disease (Dupuytren's contracture), which is most common in men over 40 years of age. It is a progressive condition that causes the fibrous fascia of the palmar surface to shorten and thicken. Patients present with one or more small, pitted nodules on the palm, which slowly progress to contracture of the fingers. Patients usually have difficulty with tasks such as face washing, hair combing, and putting their hands in their pockets. Grade 1 disease initially can be managed expectantly, but injecting the nodule with a steroid injection can be helpful. Surgery is recommended if function is impaired, contracture is progressing, or severe deformity is disabling. There is a strong association between diabetes and Dupuytren's disease. Studies have found a 3 to 33 percent prevalence of Dupuytren's in patients with diabetes; however, these patients tend to have a mild form of Dupuytren's with slow progression. Greater alcohol intake per week is associated with increased risk of Dupuytren's disease, but most patients with the disease are not alcoholics. Smoking also increases the risk of the disease. Although the reason for this association is unclear, microvascular changes in smokers may play a role.

A 53-year-old man presents with a swollen, painful left knee. He denies trauma or fever. Examination reveals decreased range of motion of the left knee with swelling and warmth. What test is likely to be diagnostic? Arthrocentesis and fluid analysis Serum uric acid level Serum white blood cell count X-ray of the knee

Correct Answer ( A ) Explanation: The patient presents with atraumatic, monoarticular arthritis with warmth and swelling and must have an arthrocentesis to rule out a septic joint. Patients with septic arthritis will typically present with fever, joint pain, swelling and effusion in a single joint. Additionally, patients frequently have warmth of the joint and decreased range of motion. Involvement of multiple joints raises the possibility of meningococcal disease and sepsis. Most septic joints are caused by gram-positive bacteria (75-90%) with Staphylococcus aureus being the most common. Although there are a number of serum lab tests that are often requested in the workup of a possible septic joint (WBC, ESR, CRP) systematic reviews show little evidence that these tests can rule in or out the disease. The only way to obtain a definitive diagnosis is with arthrocentesis and fluid analysis. A synovial fluid WBC count is traditionally considered the most useful test. A WBC >50,000 cells is highly suggestive of a septic joint. Synovial lactate level >5.6 mmol/L has been found to be highly predictive of a septic joint, as well. Treatment of a septic arthritis involves antibiotics and orthopedic consultation for possible open drainage.

A 52-year-old man with a history of chronic alcoholism and hypertension presents to the Emergency Department with weakness of the right wrist that he first noticed when he woke up this morning. On examination, he is unable to extend the wrist against resistance. Which area of his hand is likely to have decreased sensation? Dorsal first web space Thenar eminence Volar tip of the index finger Volar tip of the pinky finger

Correct Answer ( A ) Explanation: The radial nerve, formed from C6-C8 nerve roots, courses down the medial upper arm before it wraps around the mid-humerus and continues more posteriorly. It is this area, the spiral groove, that is prone to compression injury and subsequent neuropraxia. Prolonged external compression can occur during sleep if the patient's head rests on the upper arm or the arm is hung over the edge of the bed or a bench. Sedation or intoxication increases the risk of this type of injury as the patient will not wake up despite paresthesias in the arm. Patients with a radial nerve palsy (also known as Saturday night palsy) will present with a wrist drop and be unable to extend the wrist against resistance. The fingers will be held in flexion at the MCP joint and the thumb is adducted. The radial nerve supplies sensation to the dorsum of the hand from the thumb to approximately the fourth metacarpal. The dorsal first web space is entirely within the radial nerve distribution and is the best location to confirm radial nerve involvement. Treatment includes splinting of the wrist and physical therapy. Prognosis is good for recovery of function in 3-4 months.

Which of the following is the most concerning complication of this injury? Avascular necrosis Infection Malunion Osteoarthritis

Correct Answer ( A ) Explanation: The radiograph demonstrates a fracture at the waist or middle third of the scaphoid. The blood supply to the scaphoid penetrates the cortex on the dorsal surface near the tubercle waist area (distal aspect of scaphoid). Therefore, there is no direct blood supply to the proximal portion of the bone. Because of this tenuous blood supply, scaphoid fractures have a tendency to develop avascular necrosis (AVN). The more proximal the fracture, the greater the likelihood of developing AVN.

The shoulder is most vulnerable to an anterior glenohumeral dislocation when in which of the following positions? Abduction and external rotation Abduction and internal rotation Adduction and external rotation Adduction and internal rotation

Correct Answer ( A ) Explanation: The shoulder is most vulnerable when abducted and externally rotated. A fall or tackle with the arm in this position can cause an anterior shoulder dislocation. Posterior shoulder dislocations are less common and are associated with grand mal seizures and electric shock.

Which of the following correctly describes a positive straight leg raise? Pain in the back which radiates past the knee when raising the symptomatic leg 45 degrees Pain in the back which radiates past the opposite affected knee when raising the asymptomatic leg 45 degrees Pain in the back without radiation to the leg when raising the asymptomatic leg 45 degrees Pain in the back without radiation to the leg when raising the symptomatic leg 45 degrees

Correct Answer ( A ) Explanation: The straight leg test is an easy test to assess for the presence of an L5 or S1 radiculopathy. While supine with the legs both extended, the symptomatic leg is passively raised keeping the knee straight. The presence of pain in the back which radiates past the knee when the leg is elevated 30 to 70 degrees is a positive test. The straight leg test has a sensitivity of 91%, but a specificity of only 26%.

A 7-year-old boy accompanied by his mother presents to your office, complaining of a rash and bilateral knee and ankle pain. He also notes some mild crampy abdominal pain associated with nausea but denies vomiting, diarrhea, and fever. On exam you note the rash seen above. He has swelling around his patellar joints bilaterally, and the area is very painful. There are no palpable abdominal masses. His lungs are clear and the remainder of the physical examination is unremarkable. What complication is this patient at greatest risk for? Intussusception Joint damage Pneumonia Thrombocytopenia

Correct Answer ( A ) Explanation: This patient has Henoch-Schönlein Purpura (HSP). Henoch-Schönlein Purpura is the most common systemic vasculitis of childhood and is characterized by a rash (palpable purpura), arthritis, abdominal pain, and renal involvement. The exact etiology is unknown, but it is often associated with an antecedent bacterial or viral infection. It affects boys more commonly than girls and the median age is 5-years-old. The most striking feature of HSP is the characteristic rash of palpable purpura that typically occurs on the lower extremities and buttocks and is present in all cases of HSP. The rash may blanch with pressure. Gastrointestinal involvement is typically seen during the acute phase of illness and may precede the rash. It is thought to be due to small vessel involvement and usually presents as gastrointestinal colic but may lead to ischemia (heme positive stools) and intussusception. When intussusception occurs, it is usually ileoileal, unlike the more common ileocolic, and is more difficult to diagnose and reduce by barium or air contrast enema. Renal involvement will usually present early and can range from microscopic hematuria to nephrotic syndrome. Additionally, up to 15% of males can have scrotal involvement with epididymitis, orchitis, torsion, and scrotal bleeding. The musculoskeletal involvement is periarticular and does not involve the actual joint. It tends to affect the lower extremities, especially the ankles and knees. It can be very painful but does not leave any permanent joint damage. Treatment is supportive.

A 19-year-old man with HIV presents with finger pain. Physical examination reveals the lesion shown above. What management is indicated? Acyclovir Cephalexin Incision and drainage Ritonovir

Correct Answer ( A ) Explanation: This patient has herpetic whitlow which is caused by the herpes virus. This is a self-limited condition. Immunocompetent individuals only require supportive care with analgesics and should cover the lesion to prevent autoinoculation. Immunocompromised individuals, as in the patient above, should be started on the antiviral, acyclovir. The lesion may be caused by HSV 1 or 2. Typically, the infection involves a single finger. It begins as pain or pruritus followed by the appearance of clear vesicles. These vesicles coalesce and can form an ulcer after 2 weeks. The diagnosis is made clinically but can be confirmed by a viral culture or Tzank smear from a scraping of an unroofed vesicle. The lesion resolves spontaneously in 3-4 weeks but recurrence occurs in 20% of patients. The main goals of treatment are to prevent oral inoculation or transmission and provide symptomatic relief.

A 67-year-old woman presents with low back pain. She states that she has had pain for over 10 years but it has gotten worse recently. She also complains of pain shooting down her leg into her foot when she walks, which improves with bending forward. Which of the following disorders is affecting this patient? Spinal stenosis Spinal subarachnoid hemorrhage Transverse myelitis Vascular claudication

Correct Answer ( A ) Explanation: This patient's presentation is most consistent with spinal stenosis; a narrowing of the spinal canal leading to radicular symptoms on exertion. Spinal stenosis is most commonly seen in older patients (age >55 years). Typically, patients present with subacute back pain and lower extremity radiculopathies. The radicular pain occurs with walking and is relieved with rest thus mimicking claudication symptoms. Bending forward at the waist causes an increase in spinal canal diameter and reduces tension on the spinal cord leading to decrease in pain. Relief of pain with forward flexion at the waist is a unique finding in spinal stenosis. Sitting may also relieve the pain. Walking uphill may be painless (patient is flexed forward) while walking downhill (back extended) increases pain. Physical examination reveals lumbar radiculopathy (frequently of multiple levels) and increased pain with extension. Spinal subarachnoid hemorrhage (B) typically results from an A-V malformation or tumor. Patients will have a sudden onset of severe pain as opposed to the chronic or subacute nature of spinal stenosis. Transverse myelitis (C) is an inflammatory process of the spinal cord that presents with combined motor and sensory deficits that may be at multiple levels.

Which of the following structures is most at risk of direct damage with the condition shown above? Common peroneal nerve Deep femoral artery Posterior tibial artery Saphenous nerve

Correct Answer ( A ) Explanation: Tibiofemoral dislocation is a limb-threatening emergency caused by damage to multiple ligamentous structures in the knee joint. Tibiofemoral dislocations and their reduction threaten multiple neurovascular structures that run through and around the popliteal fossa. As such, examination should include a thorough neurovascular check, including an ankle-brachial index and detailed motor and sensory exams. Tibiofemoral dislocations, once recognized, should be immediately reduced to avoid permanent limb-threatening neurovascular damage. Serial neurovascular checks should follow, as damage to the popliteal artery, which runs through the popliteal space, and common peroneal (fibular) nerve, which winds around the fibular head, are common.

A 44-year-old man presents to the emergency department with back pain. On exam, the man appears uncomfortable and is tender to palpation over the right paraspinal muscles. Which of the following characteristics would most likely necessitate further evaluation? Back pain lasting more than ten days History of chronic corticosteroid use History of peptic ulcer disease Pain radiating down the posterior thigh during straight leg test

Correct Answer ( B ) Explanation: A history of chronic corticosteroid use is a risk factor for vertebral compression fractures and would most likely require further evaluation. Low back pain a very common complaint seen in primary care setting and in the emergency department. Risk factors for back pain include smoking, obesity, increasing age, female gender, sedentary work, and strenuous physical work. The majority of low back pain cases are due to musculoskeletal causes. Rare, but serious systemic etiologies of back pain include cauda equina syndrome, metastatic cancer, epidural abscess, vertebral compression fracture, and vertebral osteomyelitis. In order to help rule out serious etiologies, patients who present with low back pain should be questioned about constitutional symptoms, history of malignancy, neurologic symptoms, recent bacterial infections, history of injection drugs, history of corticosteroid use, and recent history of back procedures. Physical exam should include inspection of the back, palpation of the spine, straight leg raise, and neurologic exam. Laboratory studies are not typically required for the majority of patients who present with low back pain. If underlying infection or malignancy is suspected, the erythrocyte sedimentation rate (ESR) or C-reactive protein (CRP) may be useful. Routine plain radiographs are not recommended for patients with back pain lasting less than four weeks. Plain radiographs are indicated in patients with risk factors for compression fractures. A magnetic resonance image (MRI) is indicated for patients with any neurologic deficit and in patients with a high clinical suspicion for epidural abscess. First-line pharmacologic treatment for subacute and chronic low back pain is acetaminophen or nonsteroidal anti-inflammatory drugs (NSAIDs). Second line treatment options include short-term opioids, short-term muscle relaxants, and antidepressants. Physical therapy is also beneficial in patients with subacute and chronic low back pain. Spinal fusion is the most common surgery for chronic nonspecific back pain with degenerative changes.

A 77-year-old woman presents with 2 days of bowel incontinence, leg pain, and difficulty ambulating. Lower extremity examination reveals bilateral sensory deficits of the lumbosacral dermatomes, bilateral hyporeflexia, negative Babinski testing, and weak rectal tone. There is no lower extremity spasticity. Cranial nerve and upper extremity examination is normal. These symptoms are most likely the result of injury to which of the following structures? Lumbar disk Lumbosacral nerve roots Spinal cord gray matter Spinal cord white matter

Correct Answer ( B ) Explanation: A normal adult spinal cord ends as the conus medullaris at the L1-L2 level. The remaining L2-S4 spinal nerve roots that occupy the subdural space from the conus to the filum terminale are called the cauda equina. The cauda equina is a collection of post-ganglionic nerves (lumbosacral nerve roots), so injury to it presents as a lower motor neuron injury pattern with flaccid tone, paresis or paralysis, and hyporeflexia (or areflexia). Upper motor neuron symptoms such as hyperreflexia and clonus, spasticity and upgoing toes (positive Babinski test) are absent. The clinical presentation of cauda equina syndrome includes back pain, perianal numbness, loss of rectal tone, bowel and bladder (urinary retention followed by incontinence) abnormalities, leg weakness, loss of bulbocavernosus reflex, and loss of deep tendon reflexes. Cauda equina syndrome most commonly results from a herniated disk, but can also occur from trauma, an infection (epidural abscess), or a hematoma (epidural hematoma). A lumbar disk abnormality (A), such as herniation, typically causes unilateral radicular pain and symptoms, but it is almost exclusively associated with normal rectal tone and perineal sensation. Spinal cord tumor, or other spinal cord neuronal cell body disease affecting the gray matter (C), would cause upper motor neuron symptoms. Hyperreflexia, positive Babinski testing and spasticity are not present in this patient. Spinal cord neuronal axon or tract disease affecting the white matter (D), as in multiple sclerosis, usually presents earlier in age, and with upper extremity and cranial nerve neurologic findings, both of which are absent in this case

A 36-year-old woman presents to your clinic with a complaint of low back pain that began 4 days ago. She was moving some heavy furniture and felt her back "give out". Physical exam reveals muscle spasm in the lumbar paraspinal muscles and decreased range of motion with back flexion, extension, and rotation. Which of the following is the most appropriate therapy? Amitriptyline Naproxen Oxycodone Prednisone

Correct Answer ( B ) Explanation: Acute low back pain is one of the most common reasons that people seek out medical care in the United States. The majority of patients with low back pain who seek out treatment in the first 3 days after onset will recover within 2 weeks. Most cases of mechanical low back pain are due to muscular strain or sprain. Other causes include degenerative changes of the spine, herniated disks, compression fractures related to osteoporosis, and spinal stenosis. Diagnosis is by history and physical exam. Imaging should be reserved for patients with clinical findings that suggest systemic disease or trauma. Serious systemic causes of low back pain include cauda equina syndrome, spinal infection and metastatic cancer. Cauda equina syndrome is generally related to a metastatic tumor within the vertebra. Clinical manifestations include pain, weakness and both bladder and bowel dysfunction. Patients with spinal epidural abscess generally present with fever, malaise and a history of intravenous drug use or recent spinal injection. In cancer patients, one of the most common sites of metastasis is to the bone. Patients with back pain should be asked about any history of cancer. Once serious etiologies have been ruled out, first-line treatment for acute low back pain is with non-steroidal anti-inflammatory agents (NSAIDs), such as naproxen or ibuprofen.

Which of the following is a minor criteria for acute rheumatic fever? Carditis Fever Rash Subcutaneous nodules

Correct Answer ( B ) Explanation: Acute rheumatic fever is characterized by diffuse inflammation of the heart following an infection with group A beta-hemolytic Streptococcal infection. It most often affects children from 4-9 years old, and is still one of the leading causes of valvular disease in the world, though the incidence has dramatically decreased in the U.S. due to proper treatment of Strep infections. The pancarditis seen in acute rheumatic fever is thought to be an abnormal immunologic response, and may result in fibrosis and obliteration of the pericardial sac. Cellular infiltrates and fibrin will also often form verrucous lesions on the leafs of valves, causing valvulitis and eventually valvular disease. The mitral valve, followed by the aortic valve, are the most commonly affected. Valvulitis of the tricuspid or pulmonic valve is relatively uncommon. A new regurgitation murmur should alert a provider to the presence of valvulitis, which may eventually lead to restricted leaflet mobility and stenosis. There is no definitive laboratory test to diagnose acute rheumatic fever, so the diagnosis is made based on guidelines that include symptoms and physical exam findings. The Revised Jones Criteria establishes the major and minor criteria of acute rheumatic fever, with a diagnosis requiring two major, or one major and two minor criteria. Major criteria include carditis, polyarthritis, chorea, erythema marginatum, and subcutaneous nodules. Minor criteria include fever, arthralgia, and previous rheumatic fever or known rheumatic heart disease.

Which of the following fractures requires orthopedic referral? AFirst distal phalanx fracture without intra-articular involvement BFirst metatarsal fracture with 1 mm displacement CMidshaft fifth metatarsal midshaft with 1 mm displacement DSecond metatarsal fracture with 2 mm displacement

Correct Answer ( B ) Explanation: All first metatarsal fractures, including first metatarsal fracture with 1 mm displacement, should receive an orthopedic referral. Metatarsal fractures are the second most common type of foot fractures in adults, behind toe fractures. Direct blows and twisting injuries are the most common mechanisms of metatarsal fracture. Greater force is typically needed to fracture the first metatarsal due to its larger size. Risk factors for metatarsal fractures include osteoporosis, diabetes, and decreased physical activity. Patients with metatarsal fractures usually present with acute onset of foot pain and difficulty walking. Physical exam is often notable swelling and ecchymosis. Point tenderness is common, but swelling may make localization difficult. The tarso-metatarsal joint should be palpated to evaluate for Lisfranc injury. Three-view plain radiographs are recommended as initial diagnostic imaging. All open fractures require emergent orthopedic referral. In non-athletes, indications for orthopedic referral include multiple metatarsal fractures, intra-articular fractures, first metatarsal fracture, or second to fifth metatarsal fracture displaced greater than 3-4 mm. Initial management for minimally displaced involves a posterior splint, non-weight-bearing status, elevation, and ice. Healing typically occurs within 6 weeks in the majority of adults with metatarsal fractures

A 14-year-old girl sprained her ankle. She rates her pain 5/10. On examination, she has moderate tenderness and swelling with decreased range of motion secondary to pain. Although quite painful, she is able to ambulate. What is the grade of this ankle sprain? Grade I Grade II Grade III Grade IV

Correct Answer ( B ) Explanation: Ankle sprains are caused by partial or complete tearing of one or more ligaments that support the ankle joint. Injuries are most often due to an inversion mechanism, causing injury to the lateral ligaments. The anterior talofibular ligament (ATFL) is the first ligament to be injured, followed by the calcaneofibular ligament (CFL), and finally, in the most severe lateral sprains, the posterior talofibular ligament (PTFL). A syndesmotic ankle sprain (high ankle sprain) involves the ligaments that connect the tibia and fibula. Ankle sprains are graded I, II or III based on exam findings and functional loss. The patient in the above scenario has a Grade II ankle sprain (incomplete tear) characterized by moderate pain, swelling, tenderness and ecchymosis with mild to moderate joint instability and some loss of range and function. Typically these are painful with weight bearing.

You suspect a new patient has systemic lupus erythematous (SLE) based on symptomatology and physical exam. In addition to an ANA, what laboratory tests should be ordered to help confirm the diagnosis of SLE? Anti-Ro/SSA and anti-La/SSB Anti-smith antibody and anti-double-stranded DNA Rheumatoid factor and anti-CCP Sedimentation rate and CRP

Correct Answer ( B ) Explanation: Antibodies to an RNA-protein complex called Sm (anti-Smith antibody) and to double-stranded DNA (dsDNA) are highly associated with systemic lupus erythematous (SLE), both being 95% specific. ANA, anti-double-stranded DNA, and antiphospholipid antibody are all markers for SLE, but these test results may be misleading if not considered in clinical context. Between 2% - 5% of patients with SLE are ANA negative, whereas 5% of the normal population, and up to 20% of healthy young women are ANA positive. SLE is most common in women of reproductive age (15-40 years). The female/male ratio is approximately 2:1 before puberty and 4:1 after puberty.

A 55-year-old man with a history of peptic ulcer disease presents to your office with a complaint of right knee pain that started last night. On physical exam his knee is erythematous, warm and exquisitely tender to palpation. Synovial fluid analysis reveals the presence of positively birefringent calcium pyrophosphate dihydrate crystals. Which of the following is the most appropriate therapy? Allopurinol Colchicine Indomethacin Vancomycin

Correct Answer ( B ) Explanation: Calcium pyrophosphate crystal deposition disease, also referred to as pseudogout, is a common crystal-induced arthropathy that generally affects the large joints. Pseudogout has a similar clinical presentation to gout, but the etiology is different. Pseudogout may be idiopathic, especially in the elderly. It may also be caused by trauma, hyperparathyroidism, hemochromatosis, and medications that cause hypomagnesemia such as loop diuretics or proton pump inhibitors used in peptic ulcer disease. Patients present with acute onset of severe pain, inflammation and edema in the knees, ankles, elbows or wrists. Pseudogout is generally monoarticular, but may present in multiple joints as well. Diagnosis is by synovial fluid analysis, so arthrocentesis is necessary for patients with monoarticular arthritis. Initial treatment for pseudogout is with nonsteroidal anti-inflammatory drugs (NSAIDs) or colchicine. Workup of the underlying metabolic problem causing the pseudogout attack will help to prevent future flare-ups.

Which of the following is true regarding the management of a subungual hematoma? Antibiotics should be prescribed to patients after trephination of uncomplicated subungual hematomas Complete nail removal may be necessary if the nail bed has been significantly damaged Trephination is contraindicated when the subungual hematoma is associated with a distal phalanx fracture Trephination is indicated only when the hematoma involves greater than 50% of the nail bed

Correct Answer ( B ) Explanation: Complete nail removal may be necessary if the damage to the nail bed is significant. A subungual hematoma is the result of a laceration to the nail bed. If the nail is stable and firmly adherent to the nail bed and the surrounding tissues are not significantly damaged, there is no reason to remove the nail and repair the underlying nailbed laceration. The nail itself will keep the nail bed laceration well approximated. Nail trephination in this scenario often results in a good cosmetic endpoint. Nail trephination is preformed under sterile technique with adherence to universal precautions. Local anesthesia is generally unnecessary for uncomplicated hematoma trephination. More complex injuries, including those in which nail removal is necessary, will require a digital block for patient comfort. Most commonly hot cautery (with a heated paperclip or disposable cautery device) is used. The device is inserted perpendicular to the nail in the center of the hematoma. A pop will be felt as the device passes through the nail and at this point the device should be removed to avoid damage to the nail bed. Blood will come out of the hole and the nail will return to a normal color. Gentle downward pressure applied by the patient will help blood escape. Occasionally large hematomas require more than one trephination site for adequate drainage. After the procedure, the area should be cleaned and dressed with a dry dressing. Splints are applied as indicated. The patient is instructed to keep the digit dry for 2 days. Alternative trephination techniques include the use of an 18-gauge needle twisted into the nail. Also a small 29-gauge needle can be inserted parallel and under the nail, past the hematoma site. This is useful for hard to drain second-fifth toe subungual hematomas. Subungual hematoma is often the result of slamming a finger in a door, striking a finger with an object or dropping an object onto a toe. This can result in a distal phalanx fracture as well. This is not a contraindication to nail trephination (C). There is little risk of infection after trephination of an uncomplicated subungual hematoma therefore antibiotics are not prescribed (A).

A mother is concerned with her newborn baby boy. She believes he doesn't move his right hip like he moves his left hip. You begin an examination with flexing both hips and knees. You come down to the level of the examination table and look at the heights of the patellae. You notice the right patellae sits much lower than the left. Since you are concerned with developmental dysplasia of the hip, you refer the mother to an orthopedist. In your consultation request, you state which of the following physical examination tests is positive? Barlow Galeazzi Ortolani Yergason

Correct Answer ( B ) Explanation: Developmental dysplasia of the hip (DDH) refers to hip instability caused by inadequate pressure of the femoral head against the acetabulum that leads to a shallow socket. This is associated with an increased risk of future hip dislocation and gait abnormalities. Risk factors include a positive family history, breech birth, and female gender. There are several examination tests used to screen for this condition. The Galeazzi (or Allis) test involves flexing the infant's hips and knees, then looking level at the knee heights. If unequal, DDH and congenital short femur may be present in the lower of the two knees. Early diagnosis of hip instability, and treatment with closed reduction, harnessing (Pavlik) or casting, usually results in a favorable prognosis. Later diagnosis may require surgery to correct gait abnormalities. The Barlow and Ortolani tests are used to test for infantile hip instability. The Barlow test (A) begins with hip flexion and adduction. Lateral force is then applied. The test is positive if this lateral force causes hip dislocation. The Ortolani (C) test begins with hip abduction. A dislocated hip will have less abduction than a properly-seated hip. As further abduction is attempted, a posteriorly-placed examiner's finger pushes anteriorly to lift the head back into the acetabulum, leading to a palpable "clunk" and improved passive abduction, which results in a positive test result. Yergason's test (D) is used in evaluating adolescent or adult biceps tendonitis, not congenital hip instability.

A factory worker sustains a crush injury to his lower left leg. He has 9/10 sharp pain, distal pallor and paresthesias. The skin is intact but swollen. Minimal passive plantar flexion reproduces significant pain. Which of the following measurements are necessary to confirm a diagnosis? Ankle-brachial index and wedge pressure Diastolic blood pressure and intracompartmental pressure Mean arterial pressure and jugular venous distension Stroke volume and heart rate

Correct Answer ( B ) Explanation: Extremity compartments are bound by fascial planes and contain muscles, nerves and arteries. Acute compartment syndrome occurs when the vascular supply is unable to properly perfuse these structures. Cases are typically acute and traumatic, especially when long bones, such as the femur and tibia, fracture, or when there is a crush injury. Pathologically, injury causes intracompartmental pressure to build, which decreases venous outflow, which backs-up arterial inflow, which ultimately leads to ischemic necrosis in as little as 4 hours. Pain out of proportion to injury, distal paresthesias, distal paresis or paralysis and distal pallor are common presenting symptoms. A key physical finding is intolerable passive stretching of muscles about an injury site. Since necrosis is imminent if left untreated, suspected compartment syndrome needs immediate diagnosis and treatment. Compartment syndrome is confirmed when the diastolic blood pressure minus the intracompartmental pressure is ≤ 30 mm Hg or absolute compartment pressure > 30-40 mm Hg. Once confirmed, surgical fasciotomy of the compartment is essential for favorable prognosis. The wound is left open. Delayed closure or skin grafting is performed after edema has resolved

A track-and-field athlete presents to orthopedic clinic with insidious onset anterior knee pain. Examination reveals tenderness over the lateral patella. Which of the following radiographic findings suggests bipartite patella over patellar fracture? Narrowed patellofemoral joint space Rounded patellar fragment Sharply demarcated patellar fragment Soft tissue swelling indicative of hemarthrosis

Correct Answer ( B ) Explanation: Failure of secondary ossification in one or two of the patellar poles is called bipartite or tripartite patellae. The missing bone is replaced by thick fibrous tissue. This is mainly an asymptomatic condition, however, it may be a cause of anterior knee pain. Symptom onset is insidious and may occur after repeated knee stretching seen in jumping sports, or symptoms can arise after frank trauma. In either case, disruption of the fibrous band is causative. During examination, tenderness over the unfused fragment is frequently encountered. Knee effusion or hemarthrosis is usually absent. Its presence, however, suggests patellar fracture. Furthermore, the radiograph can differentiate a disrupted bipartite patella from a patellar fracture, in that a bipartite fragment has rounded edges, and a patellar fracture has sharp fragment edges. Activity modification, which may include knee immobilization and crutches, is the main treatment. This gives the fibrous disruption time to heal and strengthen. A narrowed patellofemoral joint space (A) suggests a decrease in patellar cartilage, as in patellofemoral syndrome or chondromalacia patellae. It does not help differentiate bipartite patella from patellar fracture. A patellar fragment with sharp edges (C), or one associated with soft tissue swelling indicative of hemarthrosis (D), suggests the acute bony disruption of patellar fracture and not the fibrous disruption of bipartite patella.

Which structure is compressed by a far lateral unilateral disc herniation between L4 and L5 lumbar vertebrae? L3 nerve root L4 nerve root L5 nerve root No nerve roots as this is below the level of the nerve roots

Correct Answer ( B ) Explanation: Far lateral unilateral disc herniation between L4 and L5 usually compresses the L4 root as it crosses the disc before exiting at the L4 intervertebral foramen. Pain may be localized around the medial side of the leg. Numbness may be present over the anteromedial aspect of the leg. The anterior tibial muscle may be weak, as evidenced by the inability to heel walk. The quadriceps and hip adductor group, both innervated from L2, L3, and L4, also may be weak and, in extended ruptures, atrophic. Reflex testing may reveal a diminished or absent patellar tendon reflex (L2, L3, and L4) or anterior tibial tendon reflex (L4). Sensory testing may show diminished sensibility over the L4 dermatome, the isolated portion of which is the medial leg and the autonomous zone of which is at the level of the medial malleolus. The nerve roots are still present at this level (D). Although the spinal cord terminates near lumbar nerves L1 and L2, the spinal nerves continue as a bundle of nerves called the cauda equina.

A 48-year-old woman presents to your office with a complaint of diffuse pain. She tells you that over the past few years her pain has worsened and she also experiences daily fatigue, difficulty concentrating, and headaches. Which of the following is most likely to be found on physical exam? Decreased grip strength Multiple points of tenderness to palpation Pitting edema of lower extremities Thinning hair

Correct Answer ( B ) Explanation: Fibromyalgia is a syndrome that causes chronic, widespread pain and tenderness. The etiology of fibromyalgia is unknown and the pathogenesis has not been proven, however it is believed to be related to a dysfunction in central pain processing. In addition to complaints of pain, patients often present with fatigue, cognitive difficulties, and multiple somatic and psychiatric symptoms. Fibromyalgia is a diagnosis of exclusion and patients should be evaluated for other disorders before making the diagnosis. Physical exam findings are generally within normal limits other than multiple points of tenderness to palpation, also referred to as trigger points. Laboratory test results are within normal limits. There is no cure for fibromyalgia and management involves a multi-disciplinary approach including education, lifestyle modifications and medication. Prognosis for patients with fibromyalgia varies, with better outcomes seen when patients seek help, engage in more physical exercise, pace their activities and in those that have less guarding during physical exam.

In addition to evidence of a preceding streptococcal infection, which of the following combination of symptoms meets the criteria for the diagnosis of acute rheumatic fever? Arthralgias, fever, and elevated erythrocyte sedimentation rate Carditis and polyarthritis Erythema marginatum and arthralgias Polyarthritis and fever

Correct Answer ( B ) Explanation: The Jones criteria were established over seventy years ago and remain the basis for diagnosing acute rheumatic fever. Diagnosis requires evidence of a preceding streptococcal infection (e.g. positive throat culture, positive rapid streptococcal antigen test or elevated streptococcal antibody titer) in addition to two major criteria or one major and two minor criteria. Carditis, polyarthritis, chorea, erythema marginatum and subcutaneous nodules are considered major manifestations of disease. Minor manifestations include fever, arthralgias, elevated erythrocyte sedimentation rate or C-reactive protein, or a prolonged PR interval on ECG.

A tax accountant presents with wrist pain. For the past year, she has had an intermittent mass that appears on the dorsum of her wrist. It comes and goes, but never seems to be the same size. Recently, with the increased workload of tax season, it has persisted for the past 3 weeks. Examination reveals a nonpulsatile, mildly tender mass on the lateral wrist dorsum. There is no surrounding erythema or edema. It transilluminates when a penlight is placed next to it. Percussion of this mass does not produce distal paresthesias. Which of the following is the most likely diagnosis? Aneurysm Ganglion Lipoma Neuroma

Correct Answer ( B ) Explanation: Ganglia are the most common soft tissue tumors of the hand. They are cystic structures which arise from a torn or degenerated joint capsule or tendon synovial sheath. Basically, they are one-way-valved "outpocketings" of synovium, connected to the capsule proper and filled with synovial fluid. The most common locations are the wrist dorsum, the volar radial wrist and the base of the finger at the proximal annular ligament of the flexor tendon sheath. They may arise and resolve spontaneously, but are most common during an increase in repetitive activity. A painful mass is a common complaint. In some cases, the ganglion is large enough to compress nearby nerves, leading to median or ulnar sensorimotor deficits. Ganglia are usually more painful and more prominent with flexor or grasping activity. Radiographs should be obtained in all cases to rule out bony pathology. Treatment includes immobilization with orthoses, aspiration or surgical excision.

A 35-year-old woman presents to your office with a complaint of wrist pain. Physical exam findings include a flesh-colored, smooth, firm, rounded swelling on the dorsal aspect of her wrist that transilluminates. Which of the following is the most likely diagnosis? Epidermoid cyst Ganglion cyst Lipoma Tophus

Correct Answer ( B ) Explanation: Ganglion cysts are benign lesions and are one of the most common soft tissue swellings of the wrist and hand. They occur most commonly in the second to fourth decade of life with a slight predominance in women. Patients may present with an obvious swelling or may have joint pain as the primary complaint without an obvious etiology. Diagnosis is generally by physical exam with the lesion being palpable and able to transilluminate. Ultrasound and magnetic resonance imaging may also help with diagnosis. Initial treatment is with nonsurgical measures including observation or needle aspiration. Surgical intervention may be indicated if conservative measures are not effective and patients continue to experience pain or limited range of motion.

Which of the following describes a grade 3 ankle sprain? Complete ligamentous rupture with concomitant distal fibular fracture Complete ligamentous rupture with considerable swelling, pain, and significant laxity Partial tear with mild laxity and moderate pain, tenderness, and instability Partial tear without laxity and only mild edema

Correct Answer ( B ) Explanation: Grade 3 sprain is a complete rupture resulting in considerable swelling, increased pain, significant laxity, and often an unstable joint. Ankle sprain involves stretching or tearing of the ligaments of the ankle. There are three grades of ankle sprain as determined by the extent of ligamentous injury. This injury is a common cause of morbidity in the general population, and the ankle is the most commonly injured joint complex among athletes. Patients who play sports experience approximately one ankle sprain for every 1000 person-days of competition. It is estimated that more than 23,000 ankle sprains require medical care in the United States per day. Eighty-five percent of all ankle sprains occur on the lateral aspect of the ankle, involving the anterior talofibular ligament and calcaneofibular ligament

Which of the following tuberculosis medications is commonly associated with lupus-like syndrome? Ethambutol Isoniazid Levofloxacin Rifampin

Correct Answer ( B ) Explanation: Isoniazid is one of a number of drugs that is associated with the development of a lupus-like syndrome. Drug-induced lupus is SLE-like but self-resolving. The illness is characterized by arthralgias, rash, serositis and myalgias. It can be caused by a number of medications but the most common are hydralazine, isoniazid, procainamide, methyldopa, chlorpromazine, quinidine, minocycline and phenytoin. Patients with drug-induced lupus rarely have major organ involvement. Cessation of the offending medication leads to resolution.

Which of the following is an indication for plain radiographs of the lumbosacral spine in the setting of low back pain? Age 45 years History of prostate cancer Pain present for three weeks Pain with radiation to the lateral foot

Correct Answer ( B ) Explanation: Low back pain is a frequent presenting complaint in the Emergency Department and the majority of these patients have uncomplicated musculoskeletal pain. Plain radiographs contribute little to the workup and management of these patients in the absence of findings that would be concerning for possible fracture, malignancy, infection, or cauda equina syndrome (the so-called "red flag" diagnoses). Indications for imaging include age < 18 years and > 50 years, history of fever, weight loss, malignancy (e.g. history of prostate cancer), injection drug use, immunocompromised state, trauma, progressive neurologic deficits, and symptoms lasting > 4-6 weeks.

A 45-year-old man presents complaining of three days of fever and myalgias, as well as sores on both of his shins. Physical exam reveals tender, nodular lesions surrounded by a starburst pattern of erythema. What is his most likely diagnosis? Nodular vasculitis Polyarteritis nodosa Schonlein-Henoch purpura Wegener granulomatosis

Correct Answer ( B ) Explanation: Polyarteritis nodosa is an autoimmune disease that causes necrotizing vasculitis of small and medium-sized muscular arteries as well as of renal and visceral arteries. It often presents with cutaneous manifestations as well as constitutional symptoms such as fever, asthma, and myalgias. If left untreated it progresses from cutaneous involvement to cause cardiovascular complications, bowel infarction and perforation, as well as renal failure. These complications lead to a high morbidity and mortality if systemic steroids are not used to delay the progression of the disease. Diagnosis is made via a deep wedge biopsy of suspicious nodules on the skin. The nodules associated with polyarteritis nodosa follow the course of the involved arteries, forming painful violaceous plaques that are surrounded by livedo reticularis. This is often described as "starburst" livedo, and is pathognomonic for the disease. This, in addition to the classic location of the nodules on the lower legs, makes polyarteritis nodosa the most likely diagnosis for this patient.

A 42-year-old woman with a history of rheumatoid arthritis presents to her primary care provider's office complaining of dry eyes and recurrent dental caries. She is constantly using artificial tears and frequently drinking liquid to help swallow food. On physical exam, her buccal mucosa is dry and tacky. Which of the following diagnostic studies is most likely to be abnormal in this woman? Schilling test Schirmer test Sims-Huhner test Sweat electrolytes test

Correct Answer ( B ) Explanation: The Schirmer test would most likely be abnormal in a patient with Sjögren's syndrome. Sjögren's syndrome is an autoimmune disorder characterized by decreased lacrimal and salivary gland function. Sjögren's syndrome is most common seen in women between 40-60 years of age. Sjögren's syndrome is frequently associated with other rheumatologic conditions such as rheumatoid arthritis, scleroderma, polymyositis, Hashimoto's thyroiditis, and systemic lupus erythematosus. Patients with Sjögren's syndrome often present with signs and symptoms related to xerophthalmia and xerostomia including ocular itching, burning, or gritty-sensation, difficulty swallowing dry foods without water, loss of taste and smell, dental caries, and periodontal disease. Women may also experience dyspareunia due to vaginal dryness. Physical exam may reveal dental caries, loss of filiform papillae, absence of salivary pooling, or parotid gland enlargement. Lymphoma is one of the most serious complications of Sjögren's syndrome and should be suspected in patients with a hard, nodular gland texture, persistent gland swelling, or systemic features. The diagnosis of Sjögren's syndrome is based on history, physical exam findings, and abnormal diagnostic studies, such as the Schirmer test, anti-Ro and Anti-SS antibodies. Treatment for Sjögren's syndrome is focused on preventing oral dryness, dental caries, and corneal damage. These measures include sipping water, using sugar-free salivary stimulants, artificial saliva, artificial tears, topical ocular cyclosporine, and muscarinic agonist. Patients should avoid anticholinergic drugs if possible. All patients with ocular symptoms should be referred to an ophthalmologist. And all patient should undergo regular preventive dental care. The Schilling test (A) is used to evaluate vitamin B12 absorption. The Schilling test involves measuring urinary B12 levels after the ingestion of radioactive vitamin B12. The Schilling test is not commonly used because it is expensive and complicated to perform. The Schilling test would have abnormal finding in pernicious anemia, malabsorption syndromes, liver disorders, and Crohn disease. The Sims-Huhner test (C) is used in the evaluation of infertility. The Sims-Huhner test is performed by examining postcoital cervical mucous to measure the ability of sperm to penetrate the mucous and maintain mobility. Normally 6-20 active sperm should be seen in each microscopic field. Decreased number of sperm or inactive sperm indicate that the cervical environment is unsuitable. The sweat electrolyte test (D) is used in the diagnosis of cystic fibrosis. The test is performed by inducing sweat and then collecting the sweat for sodium and chloride analysis. Almost all patients with cystic fibrosis have sweat sodium and chloride levels two to five times greater than normal values.

You are caring for a patient with rheumatoid arthritis. A rheumatologist refills this patient's medications, but does not order the proper laboratory testing used to monitor for adverse side effects. You decide to begin such monitoring. Which of the following is most commonly serially monitored in patients on disease-modifying antirheumatic drugs (DMARDs)? Chest radiograph Complete blood count Rheumatoid factor Thyroid function test

Correct Answer ( B ) Explanation: Rheumatoid arthritis (RA) is treated with a variety of medications, from salicylates to disease-modifying anti-rheumatic drugs (DMARDs). Several, if not all, of these medications carry significant side effects and toxicities. As such, routine monitoring of potential side effects or pathology is recommended and necessary. Common side effects of these medications include nausea, vomiting, diarrhea and rash. However, one of the most common DMARD adverse events is bone marrow suppression, especially with sulfasalazine, methotrexate, gold preparations, leflunomide, cyclosporine and azathioprine. Therefore, it is important to monitor marrow health with serial complete blood counts. Hepatitis and hepatic toxicity can occur with chronic methotrexate and leflunomide. As such, clinicians should obtain liver function tests at baseline and periodically throughout treatment. Leflunomide and cyclosporine are associated with hypertension. As such, routine monitoring of blood pressure is important in patients on these medications. Gold preparations are associated with significant kidney damage, namely proteinuria. It is recommended to check serial urinalyses, even on a weekly basis, in patients taking gold sodium thiomalate. Checking a PPD test for tuberculosis is also recommended.

A 65-year-old man presents to your office with complaints of swelling, stiffness and mild pain to the proximal interphalangeal joints in his 3rd and 4th fingers for the past few months. He is an avid golfer and has been noticing that it is affecting his grip on his golf clubs. His hands are much stiffer in the morning, but seem to loosen with movement. He takes ibuprofen before and after golf sessions, but it has not resolved the issue. For patients with this condition, which drug class has been shown to reduce permanent joint damage with early initiation of treatment? Analgesics (e.g. acetaminophen) Disease-modifying antirheumatic drugs (DMARDs) Nonsteroidal anti-inflammatory drugs (NSAIDs) Oral corticosteroids

Correct Answer ( B ) Explanation: Rheumatoid arthritis is a chronic systemic inflammatory disease with an unknown cause. It is characterized by persistent symmetric polyarthritis of the hands and feet, bone erosion, cartilage destruction and joint structure loss. It commonly affects women more than men. Smoking is a known risk factor. Patients present with a prodrome of constitutional symptoms such as fever, malaise, arthralgias, and weakness that starts insidiously before developing joint inflammation and swelling. The joints commonly involved include the metacarpals, wrists, proximal interphalangeal joints, metatarsals, and ankles, knees, and shoulders. Arthritis is symmetric and described as boggy, swollen, tender, erythematous, and stiff. Patients complain of pain or limitation with motion, and may exhibit joint deformities or nodules and ulnar deviation at the metacarpal joint in advanced disease. People with rheumatoid arthritis describe morning stiffness that gets better after initiating movement, and improves throughout the day. Diagnosis is made using a combination of modalities including laboratory studies, imaging, and clinical judgment. Helpful lab tests include rheumatoid factor assay (sensitive, but not specific), anti-cyclic citrullinated peptide antibodies (most specific for RA), erythrocyte sedimentation rate, C-reactive protein level, complete blood count, and antinuclear antibody assay. Radiography of affect joint spaces may show osteopenia, symmetrically narrowed joint space, and erosions. MRI, ultrasound, and joint aspiration can also provide useful information. Prompt initiation of disease-modifying antirheumatic drugs reduces permanent joint damage.Methotrexate is first line. NSAIDs and corticosteroids are often used in conjunction with disease-modifying antirheumatic drugs for pain and symptom control. Non-pharmacologic treatment such as heat and cold therapies, orthotics and splints, therapeutic exercise, occupational therapy, etc. is also helpful for symptom management.

A 73-year-old woman with a past medical history of osteoporosis, degenerative disc disease, and knee pain presents with a chief complaint of worsening dyspnea and recent falls. Upon physical examination you notice an excessive curve of her thoracic spine along with forward-head posture. What is the most likely diagnosis? Spinal hyperlordosis Spinal kyphosis Spinal scoliosis Spinal stenosis

Correct Answer ( B ) Explanation: Spinal kyphosis is the excessive convex curvature of the thoracic spine that tends to progress with age and can lead to complications such as diminished respiratory function and frequent falls, if not corrected. Patients with kyphosis have an excessive rounding of the back that creates a hunchback appearance. Their heads appear to be displaced forward, and often patients can't fully look upward. Patients may complain of back pain and stiffness. Morbidities associated with kyphosis include impaired respiratory function with restrictive respirations, poor physical function, and frequent falls that lead to increased rate of fractures. Diagnosis is made by clinical examination as well as imaging to measure the extent of spinal curvature. Treatment includes symptom management, rehabilitation to retrain the muscles, and surgical realignment.

A 58-year-old man presents with low back pain. Which of the following historical or physical examination findings is most consistent with a diagnosis of spinal stenosis? Cobb angle of greater than 15 degrees Decreased symptoms on forward flexion Pain relief when walking downhill Positive straight leg test

Correct Answer ( B ) Explanation: Spinal stenosis is a progressive degeneration of disc and facet joints causing a narrowing of the canal and compression of nerve roots. This often leads to neurogenic claudication and radicular symptoms which are often described as unilateral or bilateral buttock pain. In addition to this described pain, spinal stenosis may lead to sensory changes, decreased deep tendon reflexes, and mild weakness. Diagnostic imaging, along which exam findings, are helpful in establishing a diagnosis. X-rays of the lumbar spine may show narrowing of the intervertebral discs or old burst fracture, while an MRI will demonstrate stenosis. Management consists of physical therapy to improve core strength, anti-inflammatory agents, and surgical decompression for progressive disease. It may also be helpful to make certain changes to a patient's posture as specific movements or positions can either aggravate or relieve spinal stenosis symptoms. For patients with spinal stenosis, rest and forward flexion relieve compression of the nerve roots leading to decreased symptoms. Patients with spinal stenosis often feel worse when walking downhill (C) because it forces them to extend their backs, placing more compression on their spinal canal and nerve roots. The straight leg test stretches the lower lumbar roots, causing pain for patients with herniated lumbar discs. Therefore, a positive straight leg test (D) indicates a diagnosis of sciatica caused by a herniated lumbar disc, not spinal stenosis.

A woman with systemic sclerosis complains of painful blue fingers. It only seems to affect her during the cold winter months, especially if she doesn't wear gloves. She doesn't complain of arthralgias, myalgias or pruritus. Her most recent chest radiograph shows bibasilar fibrosis. Her last four serum creatinine results were normal. Which of the following medications do you recommend? Lisinopril Nifedipine Prednisone UVA-1 phototherapy

Correct Answer ( B ) Explanation: Systemic sclerosis has a multitude of manifestations. One of the more common signs is Raynaud's phenomenon. This cold temperature-induced condition results in intermittent and recurrent digital vasoconstriction, which causes finger pallor, cyanosis or hyperemia, as well as pain, numbness, ischemia and possibly ulceration. Treatment is geared at avoiding cold temperatures, wearing gloves and thick socks and smoking cessation. Medications can also be used, and include aspirin, prostaglandins, vasodilators and calcium-channel blockers, like nifedipine, especially when there is concomitant pulmonary fibrosis.

Which of the following clinical findings differentiates rheumatoid arthritis from osteoarthritis? Involvement of the proximal interphalangeal (PIP) joints Polyarticular involvement Presence of constitutional symptoms Symmetric joint involvement

Correct Answer ( C ) Explanation: Osteoarthritis (OA) and rheumatoid arthritis (RA) share a number of features but constitutional symptoms are only seen in RA. RA is a chronic inflammatory disease. Patients often present with fever, weakness and musculoskeletal pain lasting for weeks to months. Arthritis involves symmetric joints of the hands, wrists and elbows. Morning stiffness is common and usually last for about an hour. Inflammation may affect other organs leading to hepatitis, scleritis, myocarditis, pericarditis and pleuritis. Atlantoaxial subluxation may also occur and cervical spine precautions should be maintained during intubation.

A 79-year-old woman complains of increasing fatigue and general weakness. She has never fallen in her life before, but, in the past six months, she has had three falls. When considering your differential diagnosis, which of the following age-related body composition changes would you explain to the patient? Bone demineralization Myofibrosis Neuroregeneration Presbycusis

Correct Answer ( B ) Explanation: The aging process can be summarized by describing the "Senescent Phenotype", which includes the four physiological processes of body composition change, neurodegeneration, homeostatic dysregulation and altered energy dynamics. Concerning body composition, a person's weight will change over life, increasing from birth to about 65 years of age, at which point it decreases, mainly due to an overall decrease in protein and fat. However, the ratio of fat to protein increases due to fatty infiltration. One main target of this process is muscle tissue: non-collagenous protein gets replaced with fat. But, the muscle protein that does remain becomes more fibrotic, with an overall increase in fibro-collagenous tissue, a process known as fibrosis. Increased muscle fat and fibrosis, and decreased functional muscle protein, leads to decreased muscle function which manifests as weakness. Bone demineralization (A) does occur with aging, however, it's main effect is an increase in fractures, not a decrease in strength. We are born with all the neurons we'll ever have; i.e., we do not regenerate nerves (C). Presbycusis (D) is the age-related decrease in hearing.

You perform an arthrocentesis on a patient with knee pain. Synovial fluid analysis reveals a WBC 5,000 cells/µL with 70% PMNs. Which of the following is the most likely diagnosis? Hemorrhagic effusion Inflammatory arthritis Noninflammatory arthritis Septic arthritis

Correct Answer ( B ) Explanation: The analysis of synovial fluid is essential for identifying crystalline and suppurative causes of acute arthritis. The WBC count dictates the inflammatory class of the fluid. However, an analysis is often difficult because there is significant overlap between entities. A very high fluid WBC or PMN pleocytosis indicates infection, but a modest elevation does not exclude it. Therefore, cell counts should be used as a guide rather than definitive diagnosis. WBCs between 2,000 and 75,000 are often categorized as inflammatory. However, a WBC over 50,000 has a likelihood ratio of 7.7 for septic arthritis. Hemorrhagic effusions (A) are caused by trauma to the knee or coagulopathy. The fluid is grossly bloody and may contain fat droplets (lipohemarthrosis). Noninflammatory arthritis (C) such as osteoarthritis will have <3,000 WBCs on arthrocentesis. The likelihood ratio for septic arthritis (D) increases as the joint WBC count rises. A WBC count <25,000 has a likelihood ratio of 0.32, >25,000 has a likelihood ratio of 2.9, >50,000 has a likelihood ratio of 7.7.

A football player presents with acute anterior knee pain and significant knee effusion. He felt a "pop" while he was tackled. Which of the following physical examination tests is best used to diagnose a tear of the anterior cruciate ligament? Anterior drawer test Lachman test McMurray test Patellar grind test

Correct Answer ( B ) Explanation: The anterior cruciate ligament (ACL) is the primary stabilizer of the knee. It resists anterior translation of the tibia on the femur and is the most frequently injured major ligament of the knee. A tear results from deceleration, hyperextension or marked internal rotation of the tibia on the femur. The majority of ACL tears occur during sporting activities such as skiing, football, soccer, and basketball. Clinically, there is usually rapid development of hemarthroses, causing significant swelling. Almost half of individuals report feeling or hearing a pop, which is the most reliable factor. There is usually instability of the knee or a "wobbly" feeling. The Lachman test is the most sensitive test (approximately 98% sensitivity) showing increased anterior tibial displacement and a soft end point. MRI is rarely necessary to make a diagnosis. Rest, ice, crutches, immobilization and NSAIDs are common initial treatment options. Some competitive young patients may choose reconstruction as the best possible chance to return to successful play. The anterior drawer test (A) carries a 50% false-negative rate. As such, it is much less appropriate then the Lachman test for diagnosing ACL injury. The McMurray test (C) is used to diagnose meniscal, not ACL, injury. The patellar grind test (D) evaluates the articular surface of the patella, and is used in the evaluation of patellofemoral syndrome, not ACL tears.

A 60-year-old man presents to the office with a history of osteoporosis. Which of the following statements is correct as it relates to men with osteoporosis? Anorexia is a common finding Back pain with vertebral compression is the most common presenting complaint Gonadotropin releasing hormone analogues for prostate cancer lessens the chances of developing osteoporosis Secondary causes includes hyperthyroidism and coronary artery disease

Correct Answer ( B ) Explanation: The clinical presentation of osteoporosis in men is often different from that in women, especially in relation to the time to diagnosis after symptoms begin. Back pain with vertebral compression is the most common presenting complaint. Bone density measurements are less frequently obtained in men, but they are performed after symptoms occur, a far different situation than in postmenopausal women. Lack of androgens results in a skeletal deficit. Peak bone mass is clearly reduced in androgen-insufficient young male patients whether the condition results from idiopathic hypogonadotropic hypogonadism, Klinefelter's syndrome, or constitutional delayed puberty.

Which of the following infectious agents is associated with reactive arthritis? Adenovirus Chlamydia trachomatis Staphylococcus aureus Streptococcus pyogenes

Correct Answer ( B ) Explanation: The development of reactive arthritis is associated with a number of bacterial agents including Chlamydia trachomatis. Reactive arthritis is characterized by oligoarthritis, urethritis and conjunctivitis/uveitis - "can't see, can't pee, can't climb a tree." The syndrome occurs in susceptible hosts after infection with certain agents including C. trachomatis. The arthritis typically develops 2-6 weeks after an episode of urethritis or dysentery. The most commonly involved joints are the ankles and knees and the syndrome typically involves bilateral joints. Conjunctivitis may be present early and progress to uveitis later in the disease process. Management focuses on decreased inflammation with non-steroidal anti-inflammatory medications. C. trachomatis-triggered reactive arthritis improves faster with antibiotics. Adenovirus (A) is a common virus that can cause conjunctivitis.

Which of the following is classically seen in flexor tenosynovitis? Extended position of the involved digit Fusiform swelling of the digit Tenderness over the extensor sheath Vesicular eruption over the flexor surface

Correct Answer ( B ) Explanation: The flexor tendons of the fingers are covered by a double layer of synovium to promote gliding of the tendon underneath. Infections in the synovial spaces in the hand tend to spread along the course of the flexor tendon sheaths and may extend proximally to the hand. Infections are usually due to penetrating trauma involving the sheath but occasionally from hematogenous spread. Four cardinal signs of acute flexor tenosynovitis are usually present to help distinguish tenosynovitis from other hand infections. These criteria are referred to as the Kanavel's signs. Flexor tenosynovitis is a surgical emergency. Consultation with a hand surgeon is warranted along with intravenous antibiotics The affected digit is held in a flexed (A), not extended, posture. The tenderness is over the flexor (C) sheath, not extensor. Vesicles (D) are not commonly associated with flexor tenosynovitis. A localized herpes simplex infection may cause vesicles to form on a digit.

A 34-year-old man presents to the emergency department complaining of extreme left lower leg pain. Three hours earlier, his leg was crushed between his truck and trailer. On physical exam, the pain is exacerbated by passively dorsiflexing the ankle and the leg feels hard and firm. There is diminished sensation in the left foot. Which of the following is the most likely diagnosis? Acute arterial occlusion Compartment syndrome Deep venous thrombosis Superficial thrombophlebitis

Correct Answer ( B ) Explanation: The man in this case most likely has acute compartment syndrome. Compartment syndrome occurs when tissue pressure within a closed compartment compromises perfusion and results in muscle and nerve ischemia. In the extremities, compartment syndrome may occur acutely, usually following trauma, or chronically, seen usually in athletes. Acute compartment syndrome is much more common than chronic compartment syndrome. Fractures are the most common cause of acute compartment syndrome. Other potential causes include crush injuries, thermal burns, highly constrictive bandages, penetrating injuries, bleeding disorders, animal envenomations, and extravasation of intravascular fluids. The anterior compartment of the lower leg is the most common site. One of the most common and earliest symptoms of ACS is pain out of proportion to the apparent injury. Physical manifestations may include pain with passive muscle stretching, firm "wood-like" feeling of the compartment, pallor from compromised vasculature, diminished sensation, or muscle weakness. Motor deficits are late findings. The diagnosis is based on clinical findings and serial measurements of compartment pressures. Immediate surgical consultation should be obtained if acute compartment syndrome is suspected, as this is a surgical emergency. Initial management involves removing any dressings, splints, or casts and keeping the limb level with the torso. Definitive treatment is a fasciotomy. Complications of untreated acute compartment syndrome include muscle contractures, sensory deficits, paralysis, and infection.

A 20-year old man presents after jamming his index finger during a basketball game. When he tries to straighten his finger, the distal tip remains flexed as shown above. Which of the following describes the pathophysiology of this injury? Defect of the central slip causing volar migration of the lateral bands Disruption of the extensor tendon mechanism at the distal interphalangeal phalangeal (DIP) joint Disruption of the ulnar collateral ligament Inflammation of the abductor pollicis longus and extensor pollicis brevis tendons

Correct Answer ( B ) Explanation: The patient has a mallet finger. A mallet finger is characterized by a closed disruption of the distal extensor apparatus, often occurring when there is sudden forceful flexion of an extended finger, such as when struck by a ball. Because the disrupted extensor tendon no longer connects with the distal phalanx, the DIP joint is held in flexion due to unopposed action of the flexor digitorum profundus. A mallet finger injury is the most common hand injury seen in athletes. On examination, the distal tip of the finger is flexed at the DIP joint. Pain, swelling, and tenderness may be present over the DIP joint. Passive extension is usually intact, but patients are unable to actively extend the distal phalanx. Radiographs may show an associated dorsal avulsion fracture. The treatment involves immobilizing the DIP joint in slight hyperextension for 6-8 weeks to allow tendon healing. The proximal interphalangeal (PIP) joint and metacarpophalangeal (MCP) joints should be allowed to move freely.

A 45-year-old man presents to the ED with pain and swelling of his right elbow. He does not recall any specific trauma. On exam, the patient has full passive range of motion with no pain. The elbow is slightly warm to touch. Which of the following is the most appropriate management for this patient? Aspirate fluid for cell count and culture Ice and NSAIDs Incision and drainage Posterior mold splint

Correct Answer ( B ) Explanation: The patient has olecranon bursitis. This is a painful inflammation of the bursa and is often secondary to local trauma, infection, or related systemic illness. Many times, patients cannot recall a specific traumatic event. Occupational risks related to excessive frictional forces include those who frequently place their elbows on a hard desk or a carpenter who spends long periods on his knees. Commonly involved sites include the olecranon, the greater trochanter of the femur, shoulder, and knee. Unless there is concomitant infection, treatment is conservative with ice and NSAIDs. There should also be temporary cessation of the activity that led to bursitis, or protective equipment should be worn, such as kneepads, for people doing prolonged work on their knees.

A 20-year-old man who is employed in construction presents with pain to his thumb and wrist for 4 months. He states that the pain worsens while he is working. Examination reveals pain along the radial surface of the wrist with forced ulnar abduction. What management is indicated? MRI of the wrist NSAIDs and thumb spica splint Surgical release of the dorsal extensor compartment Wrist X-ray with ulnar deviation view

Correct Answer ( B ) Explanation: The patient presents with de Quervain's disease, or tenosynovitis, and should be managed conservatively with splinting and NSAIDs. De Quervain's disease is a repetitive strain injury that is often work related. In this disorder, the abductor pollicus longus (APL) and extensor pollicus brevis (EPB) tendons found in the dorsal extensor compartment of the wrist are inflamed and enlarged. Patients report pain on the radial surface of the wrist, which is increased with movement of the thumb or ulnar deviation at the wrist. Patients may have decreased grip strength as well. Finkelstein's test is considered to be pathognomonic for de Quervain's disease. It involves ulnar deviation of the wrist with the thumb in full flexion. This movement creates severe pain over the styloid of the radius. First line treatment is with rest, splinting (thumb spica splint) and NSAIDs.

A 53-year-old man presents with numbness to his right hand for 3-4 months. He states that he has numbness with waking up in the morning, which gets better when he shakes his hands. The patient indicates numbness to his first, second, and third digits on the right hand. Which of the following tests is most sensitive for this diagnosis? Finkelstein's test Median nerve compression test Phalen's test Tinel's sign

Correct Answer ( B ) Explanation: The patient presents with signs and symptoms consistent with carpal tunnel syndrome (CTS). CTS is the most common entrapment neuropathy of the wrist and is caused by compression of the median nerve. Associated symptoms include numbness and weakness in the distribution of the median nerve. Patients typically present with numbness, pain and parasthesias of the first three digits that are worse at night and with activity but improve with "shaking their hands out." Median nerve compression testing (Durkan's test) consists of direct pressure application to the median nerve at the carpal tunnel. It is positive if this maneuver reproduces the symptoms and is 87% sensitive and 90% specific for the diagnosis. Conservative treatment with wrist splinting and cortisone injections often relieves symptoms. Referral for orthopedic management is indicated in patients with recurrent or refractory symptoms. Phalen's (C) and Tinel's (D) test are classically taught in the diagnosis of CTS but suffer from low sensitivity (76% and 42-85% respectively).

The scaphoid articulates with which of the following bones? Hamate Radius Triquetrum Ulna

Correct Answer ( B ) Explanation: The scaphoid bone is one of the eight carpal bones, which all together with the distal radius and ulna, make up the bones of the wrist. The carpal bones are arranged in two rows of four bones. The proximal row consists of the scaphoid, lunate, triquetrum, and pisiform, while the distal row consists of the trapezium, trapezoid, capitate, and hamate. The wrist has many complex articulations which allow for complex movements, including the radiocarpal joint, the midcarpal joint, and the distal radioulnar joint, which allow wrist flexion, extension, radial deviation (abduction), ulnar deviation (adduction), circumduction, pronation, and supination. The scaphoid and lunate bones articulate proximally with the radius to form the radiocarpal joint. The radius also articulates with the ulna at the distal radioulnar joint. The ulna does not directly articulate with the carpal bones, but rather has an interface with the triangular fibrocartilage complex, known as the articular disk, which binds together the distal ends of the radius, ulna, lunate, and triquetrum. Scaphoid fractures can be radiographically difficult to detect. In 15% of cases, radiographs taken immediately after injury fail to show a fracture line. Therefore, if a scaphoid fracture is suspected, the wrist should be immobilized in a thumb spica splint and follow-up for repeat clinical evaluation and radiographic imaging in 7-10 days.

A 72-year-old man presents with pain in the right knee. It has increased over the last 24 hours with redness and swelling. Joint aspiration reveals negative birefringent crystals. He reports an allergy to NSAIDs and is a brittle diabetic. Colchicine is prescribed. Which side effect limits its usefulness due to a narrow therapeutic window? AHeadache BNausea and vomiting CRash DSore throat

Correct Answer ( B ) Explanation: This patient has gout, identified by the negative birefringent monosodium urate crystals on joint aspiration. Episodes of gout develop acutely with monoarticular joint pain and swelling. The joint is typically erythematous and warm with an effusion. The most common site for gout is the first metatarsal-phalangeal joint. On physical examination, it is difficult to distinguish gout from a septic joint and arthrocentesis is necessary to rule this out. Joint fluid is sent for crystal analysis to make the diagnosis. In crystal-induced arthritis the WBC count is typically 2,000 to 50,000. The measurement of serum uric acid levels is not helpful in the diagnosis of gout as levels vary widely during acute attacks. First line treatment of gout is with NSAIDs. Oral corticosteroids are a useful adjunct as well. Colchicine is helpful in reducing the inflammatory reaction in the joint by inhibiting microtubule synthesis. The most common serious adverse effects of colchicine use are gastrointestinal experienced by more than 10% of patients. This includes nausea, vomiting, and diarrhea. It is important to recognize that low-dose aspirin is potentially harmful as it can increase uric acid levels.

Which of the following patients requires radiologic imaging for her back pain? A 26-year-old woman with unilateral costovertebral angle tenderness, fever, and dysuria A 30-year-old woman with midline lumbar pain and tenderness, fever, and difficulty urinating A 35-year-old woman with bilateral lumbar level back pain after lifting furniture that is worse with movement and alleviated with rest A 40-year-old woman with lumbar-level pain and tenderness that radiates to her left anterior thigh, with a normal neurologic exam

Correct Answer ( B ) Explanation: This patient has symptoms suspicious for a spinal epidural abscess. The classic triad is severe back pain, fever, and neurological deficits. In her case, urinary retention is her deficit, which by itself is worrisome for cauda equina syndrome. The diagnostic test of choice is MRI. Epidural abscess may require evacuation by a spine surgeon. Patients are often on IV antibiotics for many weeks. However, to avoid sterilization of the abscess and facilitate isolation of the causative organism, it is recommended that antibiotic therapy be withheld in the emergency department for suspected (or even radiographically confirmed) cases. Patients at risk for spinal epidural abscess include intravenous drug users, immunocompromised patients, alcohol abusers, diabetics, patients with malignancy, and those with recent spine procedures.

A 44-year-old woman presents with pain to her second digit for 2 days. Physical examination reveals a small fluctuant area at the eponychium. What management is indicated? Acyclovir x 7 days Incision and drainage Oral antibiotics and warm soaks X-ray of the finger

Correct Answer ( B ) Explanation: This patient presents with a paronychia or a localized abscess of the nail fold that should be incised and drained. Patients will present with swelling and tenderness of the soft tissue at the lateral nail fold. Staphylococcus aureus is the most common causative agent in adults but in children, anaerobes may be more present (secondary to finger sucking). Management is with incision and drainage. The finger should be soaked in warm water and the edge of the skin at the nail should be gently lifted away from the nail using a scalpel blade. Subsequently, the cavity should be irrigated and packing gauze should be placed under the eponychium for 24 hours.

A 37-year-old man presents with worsening back pain for 1 week. He states that he has had back pain for years but over the last week, his pain has increased and he has weakness in his legs. Examination reveals 4/5 strength in the left leg and 3/5 strength in the right leg with decreased sensation. He has numbness to his buttocks and a post-void residual of 150 ml. What imaging study should be obtained? CT of the lumbar spine MRI of the lumbar spine Non-contrast head CT X-ray of the lumbar spine

Correct Answer ( B ) Explanation: This patient presents with signs and symptoms concerning for cauda equina syndrome and should emergently have an MRI done to determine the cause of compression. The syndrome results from compression of multiple lumbar and sacral nerve roots usually caused by a central disk herniation. It can also be caused by trauma, malignancy, epidural hematoma, or abscess. In cauda equina syndrome patients have a sudden onset of back pain and radiculopathies of multiple levels involving both legs. Changes in bowel or bladder function also commonly occur with urinary retention being the most consistent examination finding. As neurogenic bladder develops, patients will develop overflow incontinence. Urinary retention has a high sensitivity (90%) and negative predictive value (99.99%). Thus a low postvoid residual makes the disease highly unlikely, although it can not be used to definitively rule out the diagnosis. Other common findings are saddle anesthesia (75%) and decreased rectal tone (60-80%). MRI is the best modality for determining the cause as it has excellent tissue resolution particularly of the intervertebral disks and spinal cord. MRI also has the ability to detect more subtle soft tissue pathology as seen in epidural abscess. In patients who cannot get an MRI, CT myelogram is a reasonable alternative.

An 18-year-old marathon runner presents with diffuse muscle aches and weakness. The symptoms began after he ran his first marathon 3 days ago. His urine is dark and has gross blood on dipstick with 5 RBCs/hpf on microscopy. Which of the following tests is most likely diagnostic for this disorder? Non-contrast CT of the abdomen and pelvis Serum creatine kinase Serum myoglobin Serum potassium

Correct Answer ( B ) Explanation: This patient presents with symptoms consistent with rhabdomyolysis, which can be diagnosed by an elevated creatine kinase (CK). Rhabdomyolysis describes the breakdown of skeletal muscle with release of intracellular contents into the blood stream. These contents include creatine kinase, aspartate transaminase, lactate dehydrogenase, myoglobin, aldolase and electrolytes. Muscle breakdown leads to swelling, edema, increased compartment pressure and ischemia. The entry of intracellular components into the bloodstream can lead to renal impairment, insufficiency and irreversible damage. There are four major etiologies of rhabdomyolysis: 1) impaired muscle ATP production, 2) disruption of nutrient delivery to skeletal muscle, 3) increased metabolic demands of skeletal muscle and 4) direct muscle damage. Patients typically complain of swelling and tenderness of muscle groups as well as cramping and pain. Tea-colored urine often prompts physician evaluation. The disease is commonly suspected by the presence of blood on dipstick with the absence or only minimal amount of RBCs on microscopy. The positive dipstick is the result of myoglobinuria. Elevated CK levels are diagnostic of rhabdomyolysis. The enzyme rises over 12 hours and stays elevated for 5 days. Treatment of rhabdomyolysis focuses on identifying compartment syndrome, correcting electrolyte disorders and fluid resuscitation to correct renal impairment if present.

A 23-year-old woman, who is 6-weeks postpartum, presents with progressive right thumb and wrist pain for the past 3 weeks. The pain worsens with movement. Physical exam reveals tenderness over the right radial styloid. The pain is reproduced when the wrist is rotated ulnarly while the woman's fingers are folded over the thumb. Which of the following is the most likely diagnosis? Carpal tunnel syndrome De Quervain tendinopathy Dupuytren contracture Osteoarthritis of the trapeziometacarpal joint

Correct Answer ( B ) Explanation: This woman most likely has de Quervain tendinopathy. De Quervain tendinopathy is a tendinopathy of the abductor pollicis longus and extensor pollicis brevis tendons. De quervain tendinopathy is a frequent cause of atraumatic wrist pain in adults. De Quervain tendinopathy is most commonly seen in women ages 30-50 years. Risk factors include repetitive activities that involve abducting and extending the thumb. Postpartum mothers are thought to be at increased risk due to repeatedly lifting and holding the baby. Patients with de Quervain tendinopathy typically complain of pain on the radial side of the wrist. Patients may also notice decreased grip strength. Physical exam may reveal tenderness over the radial styloid. Finkelstein test can be used to reproduce the pain. The test involves having the patient wrap their fingers around the thumb and ulnarly deviating the wrist. Crepitus is rarely noted. Radiographs are normal for de Quervain tendinopathy. First-line treatment involves short-term nonsteroidal anti-inflammatory drugs (NSAIDs) and splinting. Glucocorticoid inject may be used for patients who do not improve with conservative therapy. De Quervain tendinopathy is typically self-limited in the majority of patients. Dupuytren contracture (B) is a fibrosing disorder that causes progressive thickening and shortening of the palmar fascia. Dupuytren contracture results in contractures of the metacarpophalangeal or proximal interphalangeal joints. The fourth and fifth fingers are most commonly affected. Patients may present with difficulty extending the affected finger and nodules may be palpated.

A 12-year-old baseball player presents with knee pain. It occurs mainly during running the bases. Tenderness and edema are prominent about the right tibial tuberosity, and resisted knee extension is painful. The remainder of the knee examination is normal. Radiographic examination of this patient would most likely reveal which of the following abnormalities? Bipartite patella Heterotopic ossification Increased density of the femoral condyles Pars interarticularis defect

Correct Answer ( B ) Explanation: Tibial tuberosity osteochondritis is called Osgood-Schlatter disease (OSD). Repetitive quadricep contraction can lead to avulsions at the secondary ossification center between the tibial tuberosity and the patellar tendon. Incidence is greatest in adolescent boys who are active in sports. Symptoms include anterior proximal tibial pain, which is worse with running, jumping and prolonged sitting. Examination typically reveals unilateral tibial tuberosity tenderness and edema with restricted and tender kneeling. Lateral radiographs typically show spicules of heterotopic ossification anterior to the tibial tuberosity. Intermittent ice, NSAIDs and rest are common treatment options. Immobilization may be required for recalcitrant cases. Surgery is commonly unnecessary, and prognosis is typically good. Bipartite patella (A) may present with superolateral patellar pain, but not commonly with tibial tuberosity pain. Increased density of the femoral condyles (C) is not present in OSD. Increased density of the femoral head is common in Legg-Calve-Perthes disease, the childhood disorder of hip avascular osteonecrosis. A defect in the pars interarticularis (D), the bony junction of the superior and inferior articular processes of the vertebral posterior columns, can lead to spondylolisthesis and back pain, not knee pain.

An 18-year-old man is brought into the emergency room for evaluation after being the restrained passenger in a motor vehicle collision. He is alert and complains of severe right lower extremity pain. He was unable to bear weight on the extremity at the scene. On exam, there is presence of a transverse groove at the medial joint line The extremity is pale and cold. Popliteal pulses are not palpable in the right lower extremity and 2+ in the left lower extremity. Which of the following is the most appropriate next step in the management of this patient? Emergent arteriography and vascular repair Emergent open surgical reduction Immediate fasciotomy Immediate manual reduction

Correct Answer ( B ) Explanation: Tibiofemoral dislocation is a limb-threatening emergency caused by multiple ligamentous tears due to hyperextension, posterior force to the anterior tibia, or force to the femur or fibula. Motor vehicle collisions are the most common cause of tibiofemoral dislocations. Knee instability in multiple directions should raise suspicion for tibiofemoral dislocation, even if no gross abnormalities are present, as many will spontaneously reduce prior to presentation. Examination of a suspected tibiofemoral dislocation should include a thorough neurovascular check, including an ankle-brachial index and a motor and sensory exam. The presence of a transverse groove at the medial joint line is called a "dimple sign" and is an indication of a posterolateral knee dislocation, which cannot be manually reduced. Closed reduction is not possible because the medial femoral condyle is buttonholed through the medial capsule and the medial collateral ligament is invaginated into the joint. Posterolateral knee dislocations are a surgical emergency and need immediate open reduction.

Which of the following is not a relative contraindication to an arthrocentesis? Anticoagulant usage Bacteremia History of arthrocentesis Infection of the overlying skin Prosthetic joints

Correct Answer ( C ) Explanation: A history of a previous arthrocentesis (C) is not considered a contraindication to joint aspiration. Known bacteremia (B) is considered a relative contraindication as are bleeding diatheses. Anticoagulation (A) use is considered a relative contraindication to arthrocentesis. Infection in the tissues overlying the site to be punctured (D) is generally considered a relative contraindication to arthrocentesis. Prosthetic joints (E) are at high risk for infection and therefore arthrocentesis should be avoided whenever possible. However this is not an absolute contraindication and should be performed if clinically indicated.

A 45-year-old woman presents to your office with right knee pain. She was playing volleyball yesterday when she collided with another player and was unable to continue playing. The knee was swollen this morning. She is able to walk but it is painful, and she also has pain when she attempts to bend her knee. On examination you have the patient stand flat footed on her right leg while you provide your shoulder for balance. You have her then flex her right knee to 20 degrees and rotate the femur on the tibia medially and laterally. This motion causes significant pain for the patient. You also note medial joint line tenderness. Which one of the following is the most likely cause of her knee pain? Anterior cruciate ligament tear Collateral ligament tear Medial meniscus tear Tibial plateau fracture

Correct Answer ( C ) Explanation: A medial meniscus tear is the most likely diagnosis in a patient older than 40 years of age who was bearing weight when the injury occurred, was unable to continue the activity, and has a positive Thessaly test. This test is performed by having the patient stand on one leg and flex the knee to 20°, then internally and externally rotate the knee. Meniscus tears typically occur as a result of twisting or change of position of the weight-bearing knee in varying degrees of flexion or extension. The presence of swelling due to an effusion is commonly seen immediately after the injury. Pain from meniscus injuries is commonly intermittent and usually the result of synovitis or abnormal motion of the unstable meniscus fragment. The pain is usually localized to the joint line. Magnetic resonance imaging is the criterion standard imaging study for imaging meniscus pathology and all intra-articular disorders. Most meniscal tears do not heal without intervention. If conservative treatment does not allow the patient to resume desired activities, his or her occupation, or a sport, surgical treatment is considered. Surgical treatment of symptomatic meniscal tears is recommended because untreated tears may increase in size and may abrade articular cartilage. Tibial plateau fractures (D) are caused by a varus or valgus force combined with a weight bearing force on the knee. The classically described situation in which this occurs is from a car striking a pedestrian's fixed knee known as a "bumper fracture". This patient is able to bear weight, so a fracture is not likely.

Which of the following laboratory findings is most commonly associated with scleroderma? Elevated erythrocyte sedimentation rate Positive anti-Smith antibodies Positive antinuclear antibodies Positive citrullinated peptide antibodies

Correct Answer ( C ) Explanation: A patient with scleroderma would most likely have positive antinuclear antibodies. Scleroderma is an uncommon, chronic disorder characterized by widespread vascular dysfunction and progressive fibrosis of the skin and internal organs. Scleroderma is more common in women than men. Scleroderma can be divided into localized scleroderma and diffuse scleroderma, or systemic sclerosis. Limited scleroderma is the more common variant. The clinical and pathologic manifestations seen in scleroderma are the result of severe fibroproliferative vascular lesions of small arteries and arterioles and excessive deposition of collagen. Patients with scleroderma typically present with swollen, puffy fingers, skin thickening and tightening, Raynaud phenomenon, or perioral skin tightening. Other complaints due to specific organ involvement may include gastroesophageal reflux, dyspnea, persistent dry cough, arthralgia, hypertension, chronic renal insufficiency, sicca syndrome, and paresthesias. On physical exam, the affected skin appears tight, shiny, and hairless. Telangiectasias, calcinosis, flexion contractures, and dry rales may also be noted. Laboratory studies should include a complete blood count, serum creatinine, creatinine kinase, urinalysis, antinuclear antibody (ANA) test, and other serologic tests. ANA test is positive in the majority of patients. Anti-topoisomerase antibody (Anti-Scl-70) and anti-RNA-polymerase III are highly specific. Pulmonary function testing should be performed in patients with systemic sclerosis. Treatment is based on managing complications and symptomatic relief. Raynaud phenomenon is commonly treated with calcium channel blockers. Esophageal reflux can be managed with proton-pump inhibitors. Renal dysfunction and hypertension are can be managed with angiotensin-converting enzyme (ACE) inhibitors. Congestive heart failure and chronic kidney disease are common causes of mortality. Positive anti-Smith antibodies (C) is suggestive of systemic lupus erythematosus. Anti-Smith and other lupus-associated antibodies (e.g. anti-double-stranded DNA) are normal in patients with scleroderma. Positive citrullinated peptide antibodies (D) are specific for rheumatoid arthritis and are uncommon in patients with scleroderma.

A 54-year-old woman has pain and swelling of the right knee. Examination of the synovial fluid reveals a leukocyte count of 5,000, and rhomboid-shaped crystals. Which of the following is the most likely diagnosis? Gonococcal arthritis Gout Pseudogout Rheumatoid arthritis

Correct Answer ( C ) Explanation: Acute monoarthritis can be the initial manifestation of many joint disorders. The first step in diagnosis is to verify that the source of pain is the joint, not the surrounding soft tissues. A careful history and physical examination are important because diagnostic studies frequently are only supportive. Examination of joint fluid often is essential in making a definitive diagnosis. The synovial fluids should be examined under polarized microscopy. Microscopic examination of synovial fluid in a patient suffering an acute attack of pseudogout shows large numbers of polymorphonuclear leukocytes. Calcium pyrophosphate dihydrate crystals are frequently found extracellularly and in polymorphonuclear leukocytes. When viewed with polarized light, the crystals appear as short, blunt rods, rhomboids, and cuboids. Plain radiographs of the affected joint will show cartilage calcification in the affected joint. Calcium pyrophosphate deposition is associated with inflammation of the knees, wrists, shoulders, ankles, elbows and or hands. There is no definitive treatment, however intra-articular corticosteroids, NSAIDs and colchicine can be effective for both the acute and chronic forms.

A 75-year-old woman presents with 5/10 knee pain that began suddenly yesterday afternoon. Her medical history is significant for diabetes mellitus, hypothyroidism, congestive heart failure and peripheral arterial disease. She denies any recent trauma. Examination reveals a swollen knee that is erythematous, warm and tender to range. Distal examination reveals pallor and decreased toe, but not ankle, proprioception. Gram staining of the fluid aspirate was negative for bacteria. Which of the following is the most likely diagnosis? Lyme arthropathy Neuropathic arthritis Pseudogout Septic arthritis

Correct Answer ( C ) Explanation: Arthritis has several causes other than degeneration (OA) and autoimmune dysfunction (RA). Deposition of crystals into the synovium is one of those causes. Pseudogout involves deposition of calcium pyrophosphate dihydrate crystals. Onset of pseudogout is less painful and dramatic as compared to gout. 50% of cases affect the knees, while other cases involve the elbows, wrists, hips and shoulders. It occurs more commonly in the elderly versus middle-aged adults. Aspiration with fluid-analysis is critical in securing a diagnosis. NSAIDs or aspiration alone may effectively treat a pseudogout flare. Long-term care of pseudogout includes weight management, exercise and ambulation aids. Neuropathic arthritis (Charcot joint) (B) is nonpainful joint destruction caused by altered proprioception. The underlying cause is usually peripheral neuropathy, with diabetes mellitus being the most common cause. The joints are deformed and dysfunctional, but usually not painful when ranged

An 18-year-old hurdler gets tripped-up and falls, contacting the running-track through his hyperextended left wrist. He presents to the ED with dorsolateral wrist pain, erythema, edema and anatomic snuffbox tenderness. Orthopedic consultation is obtained. Initial AP, lateral and oblique radiographs are negative for fracture. He is splinted and discharged. His discharge paperwork would most likely contain which of the following directions? Obtain MRI in 2 weeks if pain persists Obtain primary-care medical clearance examination for upcoming orthopedic surgery Obtain repeat radiographs in 2 weeks Obtain rheumatological consultation for wrist inflammation

Correct Answer ( C ) Explanation: Falling On an Out-Stretched Hand (FOOSH injury; "outstretched" refers to wrist hyperextension) is a common cause of scaphoid fracture, which represents the most common fractured carpal bone. The most common fracture site on the scaphoid is the wrist. Since the blood supply to the scaphoid enters distally, any >1 mm fracture-displacement is worrisome for being complicated by osteonecrosis and nonunion. A scaphoid fracture should be suspected any time there is snuff box tenderness. Wrist and scaphoid X-rays may be negative in the setting of acute fracture. Therefore, patients with snuff box tenderness or suspicion for scaphoid fracture should be placed in a thumb spica splint with repeat radiographs in 2-3 weeks.

A 19-year-old man presents with pain in his index finger. Which of the following is suggestive of flexor tenosynovitis? Delayed capillary refill Holding the finger fully extended Pain on passive extension Swelling localized to the volar aspect of the finger

Correct Answer ( C ) Explanation: Flexor tenosynovitis is an infection of the flexor tendon sheath of the finger. Most commonly, penetrating trauma to the actual sheath causes this infection although direct spread from other areas of the hand may occur. Staph aureus and streptococci are the most common causative organisms. Kanavel's signs are the four cardinal signs of acute flexor tenosynovitis include: 1) tenderness along the flexor tendon, 2) symmetric swelling of the finger ("sausage finger"), 3) pain on passive extension and 4) flexed posture of the finger. These patients need antibiotic therapy and admission to the hospital for possible operative drainage. More often, the finger will be flexed and on passive extension by the clinician the pain is exacerbated. Swelling is not localized to the volar aspect of the finger (D) but is more symmetric and uniform along the entire finger creating the sausage-like appearance.

Which of the following antibiotics is associated with spontaneous tendon rupture? Amoxicillin Doxycycline Levofloxacin Sulfamethoxazole

Correct Answer ( C ) Explanation: Fluoroquinolone drugs, including levofloxacin, have been associated with spontaneous tendon ruptures. Fluroquinolones are a commonly prescribed antibiotic class. The class includes ciprofloxacin, moxifloxacin and levofloxacin. The fluoroquinolone class of drugs has a number of side effects, the most serious of which are prolongation of the QTc and spontaneous tendon rupture. Tendon rupture appears to be more common in older patients. The overall risk is between 0.1 - 0.4%. These drugs are discouraged for use in pregnant women and children secondary to their effect on cartilage.

A 21-year-old track-and-field athlete trips during her last race, costing her the win, and resulting in severe foot pain. She presents with erythema and edema. During examination, stabilization of the calcaneus and rotation of the forefoot results in a clicking sensation and severe dorsal foot discomfort. Toe flexion and extension is maintained in a normal, nonpainful range. You most likely suspect a dislocation of which of the following joints? Fibulotalar Metatarsophalangeal Tarsometatarsal Tibiotalar

Correct Answer ( C ) Explanation: Fracture-dislocation of the tarsometatarsal joints is commonly called a Lisfranc injury. These joints exist between the three cuneiforms and the cuboid proximally and the five metatarsals distally, with the key joint being the "locking" interaction between the middle cuneiform and the second metatarsal base. Common mechanisms of injury include trauma and tripping. Pain is located on the dorsum of the midfoot, as compared to perimalleolar ligamentous pain. As such, Lisfranc injuries are easily misdiagnosed as ankle sprains. A key exam finding is pain with forefoot rotation against a stabilized hindfoot (calcaneus). This maneuver is not painful in ankle sprains or ankle mortise injury, but severely painful with Lisfranc injuries. Diagnosis can be upheld when an AP radiograph reveals lateral shift of the second metatarsal off the middle cuneiform. Nondisplaced injuries are treated with non-weight bearing casting, however, any displacement necessitates surgical intervention.

A laboratory report after an arthrocentesis returns with a finding of calcium pyrophosphate crystals that are rhomboid shaped. What is the most likely diagnosis? ACellulitis BGout CPseudogout DRheumatoid arthritis

Correct Answer ( C ) Explanation: Pseudogout is a disorder caused by deposits of calcium pyrophosphate dihydrate crystals in and outside the fluid of the joints, leading to intermittent attacks of painful joint inflammation. It most commonly affects the large joints (knees and wrists). The clinical presentation of pseudogout is similar to gout. However, distinguishing features of pseudogout include chondrocalcinosis of the affected joint, calcium pyrophosphate crystals that are positively birefringent and rhomboid shaped, and normal serum uric acid levels. Pseudogout is managed with NSAIDs for acute episodes and colchicine for prevention of attacks.

A 73-year-old woman with a past medical history of hypertension, hyperlipidemia, coronary artery disease, and chronic systolic heart failure presents to the clinic complaining of acute onset of pain in her first metatarsophalangeal joint. She denies recent trauma. Her medications include metoprolol succinate, atorvastatin, aspirin, lisinopril, furosemide and potassium. Which of the following medications is most likely contributing to her acute joint pain? Aspirin Atorvastatin Furosemide Metoprolol succinate

Correct Answer ( C ) Explanation: Furosemide is the most likely medication to be contributing to this patient's acute attack of gouty arthritis. Hyperuricemia is a relatively common finding in patients treated with a loop or thiazide diuretic and may, over a period of time, lead to gouty arthritis. Diuretics may increase the relative risk of gout by nearly 80%. Loop and thiazide diuretics decrease urate excretion by increasing net urate reabsorption; this can occur either by enhanced reabsorption or by reduced secretion. A diagnosis of gout is not necessarily an indication for discontinuation of the diuretic. Most patients with diuretic-induced gout are treated with a urate-lowering drug such as allopurinol. Treatment of diuretic-induced asymptomatic hyperuricemia is usually not necessary as the diuretic agent should be held. Some clinicians use allopurinol specifically in diuretic-induced gout to lower urate.

A 38-year-old woman presents to your office with complaints of wrist pain and swelling. Physical exam findings include a flesh-colored, smooth, firm, rounded swelling on the dorsal aspect of her wrist. Which of the following is the most appropriate first step in management? Closed rupture by force Corticosteroid injection Cyst aspiration Referral for surgical intervention

Correct Answer ( C ) Explanation: Ganglion cysts are benign lesions and are one of the most common soft tissue swellings of the wrist and hand. Ganglion cysts are thought to be caused by a degeneration of the mucoid connective tissue, although a definitive etiology has yet to be established. They are most commonly seen at the dorsum of the wrist at the scapholunate joint, but may occur at any joint or tendon sheath. Patients may present with an obvious swelling or may have joint pain as the primary complaint without an obvious etiology. Initial treatment is with nonsurgical measures including observation or needle aspiration. Many patients experience spontaneous resolution of the ganglion cyst without intervention and recurrence is common regardless of intervention used.

A 52-year-old obese man presents to your office with a complaint of right foot pain. He admits that his alcohol consumption has been increasing over the past 5 years and he frequently eats "surf and turf" at his favorite restaurant. He takes daily aspirin and simvastatin. Physical exam reveals redness, warmth, inflammation and exquisite tenderness to palpation of his right great toe. Which of the following provides the most definitive diagnosis? Blood test Clinical diagnosis Synovial fluid analysis Ultrasound

Correct Answer ( C ) Explanation: Gout is characterized by recurrent attacks of inflammatory arthritis due to deposition of monosodium urate crystals. Risk factors for gout include male sex, increasing age, alcohol consumption, obesity, hyperlipidemia, dietary factors, and use of medications that alter urate balance. Synovial fluid analysis in patients with acute gout will show intracellular monosodium urate crystals and provides the most definitive diagnosis of gout. First-line treatment of acute gout involves the use of NSAIDs or colchicine. Prevention includes reduction of risk factors and use of urate-lowering therapy such as allopurinol.

An 8-year-old boy was playing in his father's garden-shed and stepped on a nail. At that time, his father cleansed and dressed the wound appropriately. However, seven days later, the boy presents with difficulty walking. Examination shows a lateral sole puncture wound that is associated with surrounding warmth, erythema and edema. He also has an antalgic gait. Distal neurovascular examination is normal. An AP radiograph shows a round cuboid lucency. Which of the following is the most appropriate intervention at this time? Bone scan Oral antibiotics Surgical debridement Topical antibiotic ointment

Correct Answer ( C ) Explanation: In children, acute osteomyelitis commonly occurs with hematogenous spread or direct contamination, as in stepping on a nail. Pain, edema, erythema, warmth and generalized malaise are common symptoms of acute osteomyelitis. It is also important to consider cellulitis and septic arthritis in any case of suspected osteomyelitis. Laboratory investigation includes CBC, CRP and ESR, all of which may be normal early in the disease course. Blood cultures, which reveal the causative organism in up to 50% of cases, should also be obtained. Radiographs are also recommended. Early films may only show soft tissue swelling, but are useful in ruling-out fracture or neoplasm. Follow-up radiographs around a week after the injury usually shows periosteal elevation and bone lucency, two signs of bony destruction and significant pathology. In addition to blood cultures, aspiration is recommended if one can identify involvement of a specific bone and neoplasm has been ruled-out. Intravenous antibiotics are started immediately. Surgical debridement, especially in the case of nail puncture wounds, is strongly recommended, as failure to completely eradicate infection with Pseudomonas aeruginosa, the most common organism in nail puncture wounds, can lead to chronic refractory osteomyelitis, additional joint destruction, limb-length discrepancies, limb dysfunction and pathological fracture.

Which of the following diagnostic studies has the highest sensitivity to evaluate for acute radiculopathy? Cerebrospinal fluid analysis Electromyography Magnetic resonance imaging X-ray

Correct Answer ( C ) Explanation: Magnetic resonance imaging (MRI) should be performed when evaluating for acute radiculopathy because it is the most sensitive diagnostic tool for a herniated nucleus pulposus or other intraspinal pathology. Radiculopathy is a disease of a nerve root caused by compression, irritation, or swelling. It most commonly is a result of a herniated disc but can also be due to malignancy, infection, or vascular disorders. An MRI is the diagnostic test of choice because it is the least invasive and least radiation-inducing procedure to visualize the cause of radiculopathy. If a patient can't undergo an MRI because of intolerance or a contraindication, a CT scan or CT myelogram with contrast can be performed instead.

Which of the following maneuvers tests the meniscus of the knee? Anterior drawer test Lachman test McMurray test Posterior drawer test

Correct Answer ( C ) Explanation: Meniscal injuries occur frequently in patients with sudden rotary or extension-flexion motions. The menisci have no sensory nerve fibers, and the pain that results after these injuries is from irritation of the ligaments near the joint line. Several symptoms suggest the presence of a meniscal tear including: joint line pain, joint effusion, locking, and giving way of the knee. The McMurray test is performed with the patient supine and the hip and knee flexed. To check the medial meniscus, the examiner palpates the posteromedial joint line with one hand while the other hand grasps the foot. The leg is externally rotated to trap the medial meniscus, and the knee is slowly extended. Conversely, the lateral meniscus is examined with the clinician palpating the posterolateral joint line while internally rotating the leg. A painful click, popping, or thud felt in early extension is considered abnormal. It is 53% sensitive for a meniscal injury. The Anterior Drawer (A) and Lachman tests (B) are used to assess anterior cruciate ligament (ACL) instability.

A 68-year-old woman with a history of diabetes mellitus and hypertension presents to the emergency department for left knee pain. She denies history of trauma. On exam, there is no overlying erythema, wounds, or lesions on the skin. Range of motion is limited due to pain. A plain radiograph of the knee is notable for linear calcifications in the articular cartilage. Joint aspiration of both joints reveals positively birefringent crystals. Which of the following is the most appropriate treatment? Allopurinol Colchicine Naproxen Prednisone

Correct Answer ( C ) Explanation: Naproxen is the most appropriate treatment for this woman with calcium pyrophosphate deposition disease, or pseudogout. Pseudogout is characterized by the deposition of calcium-containing salts resulting in an acute pain in one or more joints. Pseudogout is most commonly seen in older adults and is seen equally in both men and women. Pseudogout is frequently associated with metabolic disorders such as hemochromatosis, hyperparathyroidism, diabetes mellitus, hypothyroidism, Wilson disease, and gout. Patients with pseudogout typically present with severe pain in one or more joints. The knee is the most commonly involved joint. Other joints typically infected include wrists, ankles, and elbows. Laboratory findings are usually nonspecific, such as leukocytosis, and elevated sedimentation rate and serum C-reactive protein. Arthrocentesis is the gold-standard diagnostic study and is critical for ruling out infectious arthritis. Positively birefringent rhomboid-shaped crystals are the hallmark synovial fluid finding for pseudogout. Synovial fluid leukocyte concentration in acute pseudogout is usually 15,000 to 30,000 per mL, with the majority being neutrophils. Punctate and parallel linear radiodensities in the cartilage are the defining feature of pseudogout on plain film radiography. Treatment for pseudogout typically involves intra-articular steroid injection, if feasible, nonsteroidal anti-inflammatory drugs (NSAIDs), colchicine, and systemic corticosteroids. Nonselective NSAIDs are typically first-line management unless contraindicated. If NSAIDs are contraindicated, colchicine is typically recommended, if it can be initiated within 24 hours of attack onset. Systemic corticosteroids are used when NSAIDs and colchicine are contraindicated or ineffective. Allopurinol (A) is not used in the setting of acute, crystal-induced arthropathy. Allopurinol, a xanthine oxidase inhibitor, is commonly used in the management of chronic gout. Allopurinol should be used with caution in patients with renal or hepatic impairment. Colchicine (B) is recommended in patients who cannot use NSAIDs. Colchicine is thought to reduce inflammation by inhibiting microtubule formation and thereby preventing neutrophil migration and activity. Colchicine should be used with caution in patients with renal or hepatic dysfunction. The most common side effects of colchicine are diarrhea, abdominal cramping, nausea, and vomiting. Prednisone (D) or other corticosteroids are used in patients in whom NSAIDs and colchicine are contraindicated. Corticosteroid use increases the risk of rebound flares. Corticosteroids can also increase blood glucose and should be used with caution in diabetics, like the woman in this case.

A 70-year-old woman presents to clinic complaining of a dull, aching pain in her right knee that has been gradually worsening over the past two years. This pain was previously controlled with ibuprofen, but now her pain has started to limit her daily activities. What is the next best step in her treatment? Acetaminophen Glucosamine and chondroitin Intra-articular injection of cortisone with lidocaine Total knee arthroplasty

Correct Answer ( C ) Explanation: Osteoarthritis is a common malady that affects over 26.9 million Americans, and is one of the most common causes of long-term disability in the United States. There are varied risk factors for developing osteoarthritis including age, mechanical stress, and metabolic disorders such as ochronosis. Initial pharmacologic therapy consists of oral analgesics, such as acetaminophen or NSAIDs, but neither are recommended for long-term use. Steroid injections have been found to provide more long-term relief than oral therapies, and may be repeated up to every three months. More frequent injections of steroids can further exacerbate joint degeneration. The correct technique involves inserting the needle between the patella and femoral condyles with either a medial or lateral approach. The use of a local anesthetic (e.g. lidocaine) along with cortisone aids patient tolerance of the pain associated with intra-articular injection, and provides temporary pain relief while the steroid takes effect. For this patient who has failed oral therapy, an intra-articular injection of cortisone and lidocaine into the knee joint is recommended.

A runner presents with recurrent anterior knee pain. She reports a "popping" sensation followed by severe pain. These episodes are brief because the pain resolves once she moves her "kneecap back into position". In the initial evaluation of this patient, a radiologic order would most likely include which of the following views? Frogleg view Grashey view Sunrise view Swimmer's view

Correct Answer ( C ) Explanation: Patellofemoral instability is the transient displacement (usually laterally) of the patella, either partially (subluxation) or completely (dislocation), causing acute or chronic patellar pain. It is associated with a positive apprehension sign when displacing the patella laterally. Diagnosis necessitates AP, lateral, tunnel and axial views. The axial view, also commonly called the sunrise view, allows the clinician a direct visualization of how the patella sits in the femoral trochlear groove. Normally, the "train is on the tracks", in other words, the patella sits equidistantly between the femoral condyles. In patients with malalignment or instability, the "train is off the tracks", showing the patella riding, most commonly, over the lateral femoral condyle, outside of the trochlear groove. Acute cases are treated with rest, icing, splinting and compression, but may also require manual reduction. Chronic recurrent cases require aggressive physical therapy and intermittent bracing. The frogleg lateral view (A) is used in evaluating the hip, not the knee. The Grashey view (B) is used in evaluating the glenohumeral joint, not the knee. A swimmer's view (D) is used to evaluate the cervicothoracic junction, not the knee.

A 45-year-old woman presents to her primary care provider complaining of pain and paresthesias in the first three digits of her left hand. The symptoms worsen at night. She has a history of hypothyroidism. Which of the following tests is most likely to be positive in this patient? Finkelstein test McMurray test Phalen test Thompson test

Correct Answer ( C ) Explanation: Phalen's test can be used to evaluate for carpal tunnel syndrome. Carpal tunnel syndrome (CTS) is a collection of signs and symptoms caused by compression of the median nerve within the carpal tunnel. CTS is the most common compressive focal mononeuropathy. Increased intra-carpal canal pressure is a key cause of clinical CTS. Risk factors for the development of CTS include obesity, female sex, pregnancy, diabetes, hypothyroidism, rheumatoid arthritis, aromatase inhibitor use, and occupational factors (e.g. repetitive hand and wrist use, vibrating tool use, sustained wrist or palm pressure). Patients with CTS classically present with pain or paresthesias in the median nerve distribution. Symptoms are often worse at night and provoked by activities that involve flexing and extending the wrist. Patients with severe cases may notice weakness or clumsiness when using the affected hand. The Phalen and Tinel tests are physical exam tests used to aid in the diagnosis. The Phalen test involves flexing the wrist and pressing the posterior aspect of the hands together. The Tinel test is performed by tapping the wrist over the median nerve. The diagnosis of CTS is a clinical diagnosis. Nerve conduction studies and needle electromyography can be used to exclude other conditions and determine the severity of the median nerve injury. Mild to moderate disease is initially managed with nonsurgical approaches such as splinting, oral and injection glucocorticoids, and physical therapy. Surgical decompression is often recommended in patients who present with severe disease. The majority of women who develop CTS during pregnancy will improve after delivery.

What is the recommended treatment for polymyositis? Antibiotics NSAIDs Prednisone Radiation therapy

Correct Answer ( C ) Explanation: Polymyositis is an inflammatory myopathy that is classified by patient age at onset or by coexisting diseases, such as myositis associated with neoplasia or myositis associated with collagen vascular diseases (e.g., systemic scleroderma, systemic lupus erythematous). They have a bimodal distribution and are seen most often between age 10 to 15 and 45 to 60 years. Myositis is most common after age 50. The cause of inflammatory myopathies is unknown, but evidence suggests a genetic predisposition (associated with certain HLA markers) combined with an environmental insult, such as viruses, thereby initiating an autoimmune process. Patients usually experience progressive, symmetric, proximal muscle weakness with fatigue, malaise, and morning stiffness. Muscles often affected are those of the shoulder, neck, and pelvic girdle. Pulmonary (interstitial pneumonitis or fibrosis), cardiac (cardiomyopathy, congestive heart failure, arrhythmias), pharyngeal (dysphagia), and musculoskeletal (myalgias, arthralgias) symptoms might occur, although most patients do not experience synovitis. CK as well as aldolase, ALT, AST, and lactate dehydrogenase (LDH) levels might be elevated. ESR is elevated only half the time. Muscle biopsy can also be helpful in diagnosis. Prednisone, 1 mg/kg/day for up to several months, is the drug of choice; the earlier started in the disease process, the more effective it is. If prednisone is not sufficient, methotrexate, azathioprine, or another immunosuppressant is added.

An 82-year-old woman with osteoporosis falls against her kitchen table. She presents with acute right shoulder pain, proximal right upper extremity edema and near-absent active shoulder joint range of motion. Which of the following is the most appropriate next step in management? Obtain a scapular-Y radiograph Passive range of motion measurement Perform a distal neurovascular examination Perform a Hawkins testing

Correct Answer ( C ) Explanation: Proximal humerus fractures are most commonly seen in elderly patients. The most common type of fracture occurs at the surgical neck (the epiphyseal-diaphyseal junction, just inferior to both tuberosities). As with any fracture, it is imperative to assess distal neurovascular supply with motor, sensory, and pulse examination. Complications from shoulder injuries include injury to the brachial plexus and vascular such as the axillary artery.

Which of the following disorders is characterized by intermittent color change of extremities? Acrocyanosis Buerger's disease Raynaud's disease Reactive arthritis

Correct Answer ( C ) Explanation: Raynaud's disease is a vasospastic disorder characterized by triphasic color change. Raynaud's disease is defined as vasospasm in small arteries and is characterized by intermittent triphasic color changes: pallor, cyanosis, and then rubor. It is the most common vasospastic disorder and affects women more than men (5:1). The main criteria for diagnosis are: 1) episodes precipitated by cold, 2) bilateral symptoms, 3) no cause of secondary Raynaud's phenomenon, 4) symptoms occurring for 2 or more years and 5) gangrene is absent. Patient's experience attacks lasting 15-60 minutes. Pallor results in a chalk white appearance of the affected digits. Rewarming leads to restoration of normal color and sensation. Acrocyanosis (A) is an uncommon vasospastic disorder characterized by persistent, symmetrical cyanosis of the fingers and hands.

A 13-year-old boy presents with right knee pain. The pain increased gradually two days ago and has been persistent since that time. He has played soccer for several years but denies injury or trauma. He has been recently healthy except for a diarrheal illness two weeks prior, but he has no further diarrhea and no longer has a fever. His examination reveals a small right knee effusion without erythema, warmth, or significant tenderness. He has a mild limp, but examination is otherwise benign. What is the most likely etiology of his knee pain? Leukemia Osgood-Schlatter Disease Reactive arthritis Septic arthritis

Correct Answer ( C ) Explanation: Reactive arthritis refers to arthritis that develops secondary to cross-reactive antibodies from a recent infection. The most commonly associated infections include enteric infections, such as Campylobacter jejuni or Shigella dysenteriae and genitourinary infections such as Chlamydia trachomatis. Reactive arthritis occurs one to four weeks following the inciting infection. The arthritis is an asymmetric oligoarthritis that typically affects the lower extremities. Enthesitis may also occur. Associated symptoms classically include conjunctivitis and urethritis ("can't see, can't pee, can't climb a tree"). Mucosal lesions and rashes may also be present. The initial treatment of reactive arthritis is non-steroidal anti-inflammatory medications. Those with inadequate pain relief may require intra-articular or systemic glucocorticoids. Osgood-Schlatter Disease (B) is an overuse syndrome that results in apophysitis of the tibial tuberosity. It is bilateral in 25-50% of patients. Osgood-Schlatter Disease presents with the gradual onset of anterior knee pain that is worsened with activity and improved with rest. Examination would reveal localized tenderness and swelling of the anterior tibial tuberosity with reproduction of pain upon squatting or extending the knee against resistance. Arthritis is not a finding of Osgood-Schlatter Disease.

Which of the following is a risk factor for an Achilles tendon rupture? Age older than 60 years Diabetes Fluoroquinolone usage Steroid taper

Correct Answer ( C ) Explanation: Rupture of the Achilles tendon is relatively common. It is most common in men between the ages 30 and 50 years who participate in recreational sports—the weekend warrior. However, it can occur in serious athletes. Exogenous risk factors include chronic corticosteroid usage and fluoroquinolone usage.

Which of the following is necessary to confirm the diagnosis of scoliosis? Adam's forward bend test Magnetic resonance imaging Radiography Scoliometer

Correct Answer ( C ) Explanation: Scoliosis is a lateral curvature of the spine. Idiopathic scoliosis is the most common type of scoliosis and within this category, the most common type is adolescent idiopathic scoliosis (AIS). AIS is defined as a Cobb angle greater than or equal to 10 degrees, age of onset 10 years or greater and no identifiable etiology. The Cobb angle is determined on radiography and is the measurement used to monitor the progression of scoliosis. Risk factors for the development of AIS include age less than 12 years, onset prior to menarche, curves with a Cobb angle greater than or equal to 20 degrees, female sex, and double or thoracic curves. The curvature of scoliosis can progress during periods of rapid growth, such as adolescence. This can cause significant deformity and other clinical manifestations such as cardiopulmonary compromise, therefore it is important to identify and monitor patients with this condition. A number of screening measures are used to determine the likelihood of scoliosis, but radiography is needed for the diagnosis of scoliosis as determined by the Cobb angle. Physical examination for scoliosis includes the Adams forward bend test (A). The patient is observed from the back while bending forward at the waist until the spine is parallel to the ground. Lumbar or thoracic prominence on one side indicates possible scoliosis and further workup should be initiated.

A 46-year-old woman presents complaining of dry mouth, fatigue, and lymphadenopathy. On review of systems, she also mentions being bothered by dry eyes. Serologic testing is positive for antinuclear antibodies. What is her most likely diagnosis? Sarcoidosis Scleroderma Sjogren syndrome Systemic lupus erythematosus

Correct Answer ( C ) Explanation: Sjogren syndrome is an autoimmune disease that causes inflammation and dysfunction of the exocrine glands. The two main glands that are affected by this disease are the lacrimal and salivary glands, which leads to xerophthalmia and xerostomia. Sjogren syndrome may also present with numerous extraglandular manifestations, such as hepatomegaly and splenomegaly, but the two most common are Raynaud's phenomenon and small-vessel vasculitis. Additionally, patients with Sjogren syndrome may present with systemic features such as fatigue, fever, weight loss, and lymphadenopathy. Diagnosis is made based on the presence of xerophthalmia and xerostomia as well as evidence of at least one autoantibody (i.e. antinuclear antibodies, rheumatoid factor, anti-Ro antibodies, and anti-La antibodies). Treatment may be symptomatic for symptoms of exocrine deficiencies or systemic for extraglandular involvement. Often, symptomatic management consists of artificial tears or saliva. Systemic treatment may consist of antimalarial agents or immunosuppressive agents such as chloroquine or methotrexate, respectively. A patient with dry mouth, dry eyes, systemic symptoms, and an autoantibody is most likely to have Sjogren syndrome.

A 29-year-old woman presents with a chief complaint of dry eyes. Which of the following would indicate the need for testing for a systemic illness? Anticholinergic use Clear discharge Dry mouth Long term contact lens use

Correct Answer ( C ) Explanation: Sjogren's syndrome is a systemic autoimmune disease in which immune cells attack and destroy the exocrine glands that produce tears and saliva leading to dry mouth and eyes (keratoconjunctivitis sicca). Work up for Sjogren's includes lab work to identify the auto-antibodies (ANA, RF, anti SS-A, anti SS-B (formally anti-Ro and anti-La)), a Schirmer test (for tear production) and a biopsy of the salivary glands. Patients may also present with dryness of other mucous membranes including the vagina and the nasal passageways. Complications of Sjogren's include dyspareunia, corneal abrasions, dental caries, malabsorption syndromes to due to pancreatic failure, lymphoma, polyneuropathy, lung and kidney disease and other rheumatologial complaints. Treatment options include artificial tears and saliva, topical ophthalmic steroids, systemic steroids or cytoxic agents or a combination of both.

Which of the following is the most commonly fractured carpal bone seen in hand fractures Capitate Hamate Scaphoid Triquetrum

Correct Answer ( C ) Explanation: The scaphoid is located in the proximal carpal row along with the lunate, triquetrum and pisiform bones. It is the most commonly fractured carpal bone. Mechanism of injury is generally a fall on an outstretched hand with the wrist in extension and radial deviation. Patients present with pain and swelling on the radial side of the wrist, limited range of motion and tenderness to palpation of the anatomic snuffbox. Initial management includes accurate diagnosis of the fracture, pain control, reduction and splinting (thumb spica splint) of the injury and referral to a hand surgeon for follow up.

A 52-year-old woman with a 10-year history of rheumatoid arthritis presents to the ED with blurred vision. For the past 4 weeks, she has noticed significant eye dryness, gritty sensation and burning. The only thing that has helped with this discomfort is excessive rubbing of eyeball. You readily appreciate a swollen, erythematous left eye with purulent drainage and injected conjunctiva. As you are filling out paperwork for ophthalmologic antibiotics and ophthalmology referral, she asks what she can do about her annoying dry mouth. Which of the following medications do you most likely prescribe? Cyclosporine Nortriptyline Pilocarpine Ranitidine

Correct Answer ( C ) Explanation: Sjogren's syndrome is an autoimmune inflammatory condition that mainly affects the tear and salivary glands. It can occur alone, or in conjunction with another rheumatologic condition, such as rheumatoid arthritis or systemic lupus erythematosus. The common symptoms, called sicca symptoms, include dryness of the eyes, mouth (xerostomia), throat and skin. Complaints include eye burning and gritty sensation, mouth dryness with difficulty swallowing and acid reflux, and nasal, throat and skin dryness. Other symptoms include salivary and tear gland tenderness and edema. Overall, the dryness can lead to eye infections, corneal damage, dental decay and gingivitis and oral infections like thrush. In addition, patient's with Sjogren's can have arthralgias, arthritis, myalgias, rash and Raynaud's phenomenon. Treatment begins with gum chewing, artificial tears and saliva, drinking more water, humidifiers, nasal saline spray and regular dental and ophthalmology care. Prescription medications, such as pilocarpine or cevimuline, can be used to simulate the production of saliva. Pilocarpine's muscarinic-receptor-agonist action offers xerostomia patients a parasympathomimetic saliva-producing effect.

A 55-year-old man presents to your office with gradual onset atraumatic low back pain, leg pain, and numbness. The pain is most often bilateral, involving the buttocks and thighs and spreads distally towards the feet. The leg pain begins and worsens with walking and standing and is relieved by sitting or lying down with hips and knees drawn up in a sitting posture. Which of the following is the most likely diagnosis? Ankylosing spondylitis Scoliosis Spinal stenosis Spondylolisthesis

Correct Answer ( C ) Explanation: Spinal stenosis is the narrowing of the spinal canal and neural foramina. The more common acquired spinal stenosis is caused by degenerative changes in the intervertebral disks, ligaments, and facet joints surrounding the lumbar canal. These degenerative changes can be caused by disk or joint disease, back surgery, and repetitive trauma. Stenosis initially becomes symptomatic at 40 to 50 years and older. Congenital narrowing of the spinal canal causes symptoms earlier in life and is uncommon. Spinal stenosis usually occurs at cervical and lumbar segments. Patients with cervical stenosis present with radiating arm pain, numbness, paresthesia, and motor weakness. The common symptoms with lumbar stenosis are gradual onset low back pain, leg pain, and numbness. The pain is most often bilateral, involving the buttocks and thighs and spreading distally toward the feet. The classic presentation is radiating leg pain (burning or cramping) that begins or worsens with walking and standing and is relieved by sitting or lying down with hips and knees drawn up in a sitting posture (neurogenic claudication). Bending forward diminishes pain. Physical examination is frequently normal but may include loss of lumbar lordosis, impairment of spinal mobility, asymmetric knee or ankle reflexes, and muscle weakness. Results of straight-leg raising are characteristically negative. Advanced imaging studies are obtained to establish and confirm the diagnosis of spinal stenosis when surgery is considered. Plain spine radiographs often reveal degenerative changes. MRI is currently the preferred modality, followed by CT scan.

An 18-year-old girl presents to the ED with left ankle pain. Earlier in the day she was playing softball and slid into second base and "twisted her ankle." On exam, you note moderate swelling, tenderness, and pain with passive range of motion of the ankle. You do note some abnormal motion when stressing the joint. Which of the following is the most likely diagnosis? First-degree sprain First-degree strain Second-degree sprain Second-degree strain

Correct Answer ( C ) Explanation: Sprains are classified as ligamentous injuries resulting from an abnormal motion of a joint. In such cases, there is injury to the ligamentous fibers of a supporting joint. Sprains are graded according to the severity of pathologic findings; however, clinically the grades are often indistinct. A second-degree sprain is a partial tear of a ligament (more than first-degree). Clinically, there will be moderate hemorrhage and swelling, tenderness, painful motion, abnormal motion, and loss of function. Although there may be some laxity with stressing of the joint, an absence of end points will be seen only with complete ligament rupture (i.e., third-degree sprains).

A 14-year-old boy presents to clinic with a one month history of right shoulder pain. The patient is a competitive tennis player. Symptoms began during a match when the patient was serving. He experienced a pop in the shoulder and had immediate pain. Magnetic resonance imaging of the shoulder shows a superior labral tear from anterior to posterior (SLAP lesion). Which of the following is the most likely physical exam finding? Positive cross arm test Positive Neer test Positive O'Brien test Positive sulcus test

Correct Answer ( C ) Explanation: The O'Brien test (also known as the active compression test) is performed with the patient standing. The patient holds his shoulder in 90 degrees of forward flexion with the elbow extended. The arm is then adducted 10 degrees. The patient is instructed to internally rotate his arm, pointing his thumb to the floor. The patient should hold his arm in this position against resistance while the examiner pushes the arm towards the floor. The examiner then applies the same force with the patient's arm externally rotated. The test is considered positive if the patient has pain or popping in the internally rotated position and the pain is eliminated in the externally rotated position. The test is thought to be sensitive for a SLAP (superior labral tear from anterior to posterior) lesion if the patient reports deep, diffuse glenohumeral joint pain. SLAP lesions can be secondary to an acute injury or chronic overuse, such as in overhead athletes. Patients may complain of clicking or mechanical symptoms, especially when the arm is the in cocked position of throwing or serving. The sulcus test (D) is performed with the patient's arm in a relaxed neutral position. The examiner pulls the patient's hand downward at the elbow joint. The examiner looks for depression (sulcus) between the lateral acromion and the head of the humerus, suggesting inferior glenohumeral instability. The cross arm test (A) is used to asses for acromioclavicular joint pain. In this test, the patient raises his or her arm to 90 degrees and then actively adducts the arm. The test is positive if this elicits pain over the acromioclavicular joint. To perform the Neer test (B), the patient should fully pronate the affected arm. The examiner then stabilizes the patient's scapula with one hand while using the other hand to lift the patient's arm into maximal forward flexion. Pain or facial grimacing during this maneuver suggests impingement of the supraspinatus tendon.

Which of the following best classifies this fracture? Salter-Harris Type I Salter-Harris Type II Salter-Harris Type III Salter-Harris Type IV

Correct Answer ( C ) Explanation: The Salter-Harris classification of fractures describe pediatric fractures through the growth plate. These fractures are classified according to the involvement of the physis, metaphysis, or epiphysis. The classification of the injuries is important, because it affects the treatment and prognosis for the patient. The radiograph above demonstrates a Salter-Harris Type 3 fracture. This is defined by a fracture involving the physis (growth plate) and epiphysis. The fracture passes through the hypertrophic layer of the physis and extends to split the epiphysis. Because it crosses the physis and extends into the articular surface of the bone, Salter-Harris Type III fractures put the patient at risk for chronic disability. Rarely does significant deformity occur and the overall prognosis is favorable. Definitive treatment for Type III fractures is surgical.

A 25-year-old carpenter presents to your primary care clinic with acute, atraumatic low back pain. This is his first episode of back pain. He denies weakness, numbness, fever or bowel and bladder incontinence. Which of the following pain descriptors, gathered from a detailed history and examination, would most prompt you to order imaging on this initial visit? Axial pain Flexion-based pain Night pain Pain began after lifting a heavy object

Correct Answer ( C ) Explanation: The evaluation of back pain begins with assessing for "red flags," such as night pain and weight loss (suspicious for tumor), fever, chills, and sweats (consider infection), acute bony tenderness (consider fracture), morning stiffness lasting > 30 minutes in young adults (consider seronegative spondyloarthropathy, and any neurologic deficit or bowel or bladder involvement (consider nerve root compromise). Acute lumbar sprain is usually the result of repeated lifting or twisting, typically occurring in young men with laborious vocation. Axial lumbar pain is typical of sprain and internal disk disruption. Radicular pain is typical of disk bulge or herniation with nerve root encroachment. Flexion-based axial lumbar pain is typical of disk pathology, whereas extension-based axial lumbar pain suggests facet joints, central stenosis or foraminal stenosis are the main pain generators. Imaging plays a limited role in acute sprain, however, it should be considered for acute lumbar pain associated with any neurologic abnormality, trauma, concerning past medical or family history, or any case involved with fever, sweats, chills, weight loss, night pain or pain at rest.

Rupture of the flexor digitorum superficialis tendon will result in the loss of what function? Flexion at the distal interphalangeal joint Flexion at the metacarpophalangeal joint Flexion at the proximal interphalangeal joint Pincer function

Correct Answer ( C ) Explanation: The flexor digitorum superficialis (FDS) originates from the medial epicondyle of the humerus and travels distally through the anterior compartment of the forearm. At the wrist, four tendons come off the muscle and travel through the carpal tunnel. The tendons insert at the proximal aspect of the middle phalanx of each of the four fingers. The flexor digitorum profundus tendon passes deep to the split of the distal flexor digitorum superficialis tendon and inserts at the base of the distal phalanx. The primary function of the FDS is flexion at the proximal interphalangeal joint. On examination, the patient should be asked to flex at the PIP joint against resistance while the other three fingers are held in extension. Injuries to flexor tendons should be referred to a hand specialist for definitive management. If one is not immediately available, open wounds should be irrigated and approximated, the hand should be splinted in partial flexion at the MCP, PIP, and DIP joints and the patient should be referred for urgent outpatient management.

Which of the following stable patients with back pain should have an emergent MRI performed? 23-year-old man with a positive straight leg raise and otherwise normal neurologic examination 30-year-old woman with back pain and dysuria 34-year-old man with a history of intravenous drug abuse with back pain and constipation 45-year-old woman with back pain after a car accident with no midline tenderness and normal neurologic examination

Correct Answer ( C ) Explanation: The majority of patients who present with back pain do not require emergent imaging. However, those who may have a serious cause of their back pain including cauda equina syndrome, epidural abscess, vertebral osteomyelitis and other causes of cord impingement should be considered for emergent imaging of the spine. Patients that present with back pain should be screened for "red flags" in their presentations that support one of these critical diagnoses. These red flags include age (both extremes), history of cancer (especially those known to metastasize to bone), fever, weakness, saddle anesthesia, change in bowel or bladder function (incontinence or retention/constipation), trauma, history of immunocompromise (including chronic steroid use) and intravenous drug abuse. Patients with these components to their presentations have a higher risk of dangerous diagnoses and should be considered for early imaging with either CT (better for bony abnormalities) or MRI (better for spinal cord visualization).

Which of the following fractures is most commonly non-operative in adults? Galeazzi Monteggia Radial head fracture Tibial plateau fracture

Correct Answer ( C ) Explanation: The majority of patients with a radial head fracture will not require operative management. Radial head fractures are typically caused by indirect trauma (e.g. fall on an outstretched hand). There are 4 types of radial head fractures ranging from the more common type I (undisplaced fracture) and type II (marginal fracture with minimal displacement) to the rarer type III (comminuted fracture) and type IV (fracture with dislocation). Type I and II injuries are usually treated with a sling and range of motion exercises. It is important to fully test the patient's range of motion. If the range of motion is limited by pain, the joint should be injected with an anesthetic agent (lidocaine, bupivicaine) and tested again. Continued limited range of motion in spite of pain control suggests the presence of entrapped fragments and orthopedic surgery may be required.

A 49-year-old woman presents with a lump on the dorsum of her wrist which has been present for several weeks. She denies trauma. On examination, there is a soft, nontender, mobile mass on the dorsum of the wrist over the scapholunate joint. Which of the following is the most likely diagnosis? Enchondroma Felon Ganglion cyst Glomus tumor

Correct Answer ( C ) Explanation: The patient has a ganglion cyst, the most common soft tissue tumor of the hand. Ganglion cysts are filled with synovial fluid and develop either from a joint or the synovial lining of a tendon. They occur most commonly in the wrist and the flexor tendon sheaths of the fingers. They develop gradually and are often asymptomatic, though patients may complain of mild pain or paresthesias if the ganglion compresses a nerve. The lesions are benign, but treatment may be necessary for large or painful cysts. Ganglion cysts can be aspirated, but often recur. Definitive treatment is elective operative excision. Enchondromas (A) are the most common primary bone tumor in the hand, but would present as a firm deformity over a bone, not a soft mobile lump. Felon (B) is infection of the pulp of the distal fingertip. Glomus tumors (D) are benign vascular lesions. The most common presentation is a painful, bluish discoloration in the subungual area.

A 58-year-old woman who works on an assembly line complains of bilateral wrist pain for the last several months. She describes pain, numbness, and paresthesias in her thumb, index, and long fingers. Which of the following tests is most likely to be positive? Adson's test Finkelstein's test Phalen's test Tinel's sign

Correct Answer ( C ) Explanation: The patient has carpal tunnel syndrome. Symptoms of carpal tunnel syndrome include gradual onset of numbness, paresthesias, and pain in the thumb, index, and long fingers. Symptoms are often worse at night and after strenuous activity. Carpal tunnel syndrome is commonly caused by repetitive strain. It can also be seen after distal radius and carpal bone fractures and dislocations, or as a result of systemic conditions (e.g rheumatoid arthritis, hypothyroidism, pregnancy, and diabetes). Phalen's test has a sensitivity of 76% and specificity of 80% for carpal tunnel syndrome. The test is performed by having the patient fully flex the wrists and push them together with the hands facing downward. A positive test is elicited if the patient develops paresthesias or numbness in the median nerve distribution within 60 seconds. Tinel's sign is less sensitive that Phalen's sign for carpal tunnel syndrome. A positive test is demonstration of pain or paresthesias in the median nerve distribution when the median nerve is tapped on at the wrist. Cervical radiculopathy and thoracic outlet syndrome should also be considered as symptoms can mimic carpal tunnel syndrome. Nerve conduction studies are used to confirm the diagnosis of carpal tunnel syndrome, with reported sensitivity of 85 to 90%. Nonoperative management for carpal tunnel syndrome includes splinting the wrist in a neutral position, antiinflammatory medications, and cortisone injections into the carpal tunnel. For persistent or severe symptoms, surgical release of the flexor retinaculum is performed. Adson's test (A) is used to assess for thoracic outlet pathology. Finkelstein's test (B) is pathognomonic for de Quervain's tenosynovitis. Tinel's sign (D) is less sensitive for carpal tunnel syndrome than Phalen's test.

A patient presents to the Emergency Department with wrist and hand weakness. He is holding his wrist in flexion and his hand is hanging limply. He is unable to extend his wrist against resistance. Which of the following nerves is most likely injured? Axillary nerve Median nerve Radial nerve Ulnar nerve

Correct Answer ( C ) Explanation: The patient has wrist drop, which is caused by injury to the radial nerve. The radial nerve innervates the dorsal extrinsic muscles in the forearm, which function in wrist and metacarpophalangeal (MCP) joint extension, as well as abduction and extension of the thumb. Radial nerve motor function can be assessed by having the patient extend the wrist against resistance. Patients with an intact radial nerve should be able to make the "thumbs up" sign. Sensation is tested in the first dorsal web space. Radial nerve injury has many causes, including trauma to the brachial plexus or the humerus (the radial nerve runs along the lateral border of the humerus). Additionally, prolonged use of crutches, which applies pressure along the nerve, can result in a radial nerve palsy. Similarly, "Saturday night palsy" refers to an intoxicated individual falling asleep with their arm slung over a chair, compressing the radial nerve. The axillary nerve (A) innervates the deltoid, teres minor, and the long head of the triceps. Injury to the axillary nerve leads to shoulder and elbow motor deficits. The median nerve (B) innervates the flexor muscles of the wrist, the thenar muscles, and the lumbricals to the 2nd and 3rd digits. Median nerve motor function can be assessed by testing thumb opposition and pincer grasp. Patients with an intact median nerve should be able to make the "A-OK" sign. The ulnar nerve (D) innervates the hypothenar muscles, most of the intrinsic muscles of the hand, and the adductor pollicis. Ulnar nerve motor function can be tested with thumb opposition and pincer grasp. Damage to the distal ulnar nerve can result in claw hand, in which the little and ring fingers are held in flexion at the interphalangeal joint.

A 16-year-old boy presents complaining of right ankle pain after injuring himself while playing basketball. He states that he "twisted his ankle" when he landed on another players foot trying to get a rebound. He has not been able to walk on his right foot since this occurred two hours ago and cannot do so in the ED. On examination, his pulses are intact. Strength and sensation are normal. The ankle is swollen laterally. He is tender over the lateral malleolus and at the base of the fifth metatarsal. No tenderness or swelling is noted over the proximal tibia and fibula. Which of the following is true regarding emergency department radiographic work-up of this patient? AA complete ankle series is the only indicated study BA complete foot series is the only indicated study CBoth a complete ankle and a complete foot series are indicated DNo radiographic imaging is indicated at this time

Correct Answer ( C ) Explanation: The patient should undergo X-rays of both the ankle and the foot. The Ottawa Ankle Rules (OAR) are a clinical decision making tool used to help determine the need for radiographic imaging following blunt ankle injury. They apply to acute injuries that have occurred within the previous 48 hours, however are not useful in the subacute or chronic phase. The OAR were not designed to be general guidelines for foot injuries; the foot rules only apply to the midfoot and do not apply to the hindfoot or forefoot.

What is the most appropriate first line treatment for plantar fasciitis? Corticosteroid injection Extracorpeal shock wave ultrasound NSAIDs Surgical therapy

Correct Answer ( C ) Explanation: The plantar fascia is a tough layer of the sole that is functionally significant during foot strike and the early stance phase of walking. Plantar fasciitis is an overuse injury of insidious onset that usually begins with pain on first weight bearing in the morning or after prolonged sitting. This progresses to persistent pain during gait. Pain and tenderness are localized to the medial aspect of the heel. Plain radiography is not diagnostic but shows a calcaneal spur in 50% of patients with plantar fasciitis. NSAIDs are considered first line treatment, along with rest, ice, and shoe inserts

A 48-year-old man presents to the office with right shoulder pain that has been worse over the last 3 months. He has positive Hawkins and Neer signs. There is visible atrophy at the superior and posterior aspects of the shoulder. Which of the following is the most likely diagnosis? Cervical nerve impingement with radiculopathy Glenohumeral osteoarthritis Rotator cuff impingement Suprascapular nerve entrapment

Correct Answer ( C ) Explanation: The rotator cuff is made up of the supraspinatus (abduction), infraspinatus (external rotation), teres minor (external rotation), and subscapularis (internal rotation) muscles. Rotator cuff impingement and tearing usually begin in the supraspinatus tendon as it passes under the acromion. Patients typically have pain with abduction above the head and internal rotation (reaching up the back). It is associated with a positive Hawkins (with the patient's elbow and shoulder flexed to 90 degrees the examiner passively internally rotates the shoulder by stabilizing the elbow and pushing down on the wrist) and Neer (the examiner performs maximal passive forward flexion with internal rotation while stabilizing the patient's scapula with the other hand) tests. Cervical nerve impingement with radiculopathy (A) could have caused muscular atrophy, but would not explain signs of rotator cuff impingement. Cervical radiculopathy is usually identified by neck stiffness, deltoid weakness, absent biceps reflex and possible sensory loss. Glenohumeral osteoarthritis (B) is a common cause of shoulder pain with any movement but would not explain rotator cuff impingement. Glenohumeral osteoarthritis also would not cause focal muscle atrophy, and is typically visible on radiograph. Suprascapular nerve entrapment (D) can cause atrophy of the supraspinatus and infraspinatus muscles, but would not have signs of impingement.

A 26-year-old man is training for a marathon and presents to the Emergency Department with right knee pain. The pain began during a run yesterday and has become progressively worse. He is unable to ascend stairs without significant pain. Palpation below the medial joint line of the knee elicits pain. You suspect anserine bursitis. The tendon of which of the following muscles is affected with this diagnosis? Biceps femoris Gastrocnemius Gracilis Vastus medialis

Correct Answer ( C ) Explanation: The tendons of the gracilis, sartorius and semitendinosus muscles insert onto the medial tibia below the knee joint and overlie the pes anserinus (Latin for "goose's foot") bursa. Pes anserine bursitis occurs commonly in runners, obese women with osteoarthritis and in other overuse syndromes. It is thought to be due to tight hamstrings, leading to increased friction and irritation of the bursa. Patients complain of anterior medial knee pain below the joint line and tenderness to palpation over the bursa. Symptoms may worsen with overuse or with certain activities such as squatting, running, or ascending or descending stairs. It usually is not symptomatic when walking on flat surfaces. Diagnosis is clinical. Management consists of rest, ice, proper stretching and nonsteroidal anti-inflammatory medications. The condition is self-limiting, and most athletes are able to return to play after a period of conservative therapy.

A 45-year-old man presents with left leg pain. He describes a sudden onset pain in his calf area while playing basketball with his son. On exam, you squeeze his right calf and elicit plantar flexion of the right foot. When you do the same with his left calf there is no plantar flexion of his left foot. You administer ibuprofen and place the left leg in a posterior splint. Which of the following is the most appropriate next step in management? Obtain X-rays of the ankle and foot Prescribe physical therapy Refer to orthopedic surgery Screen for autoimmune diseases

Correct Answer ( C ) Explanation: This patient has an abnormal Thompson test, a common clinical test used to diagnosed Achilles tendon rupture. Achilles tendon rupture typically occurs in the setting of sudden and forced dorsiflexion during athletic activity. Patients often report an audible "pop" and sudden pain in the calf area that improves or subsides quickly, followed by decreased ability to plantar flex the foot. The diagnosis is typically clinical. On physical examination, patients may exhibit a palpable tendon defect in the lower calf area. Some plantar flexion is usually maintained, even in cases of complete tendon rupture, due to the actions of the surrounding muscles. The Thompson test is a classic maneuver to assess the Achilles tendon. The patient lies prone, with the legs flexed at the knee at 90 degrees. The examiner then squeezes the calf muscles and observes for passive plantar flexion of the foot. The injured extremity will have a weakened or no response compared to the uninjured side. Risk factors for Achilles tendon rupture include rheumatological diseases, chronic renal failure, steroid use, and recent treatment with fluoroquinolone antibiotics. Controversy remains regarding the best treatment. While early operative repair is associated with lower risk of rerupture and possibly improved functional outcome, it also has a higher rate of complications. Nonoperative management consists of a series of casts. Ultimately, the decision depends on the patient's age, activity level and preference, and is best made on a non-emergent basis in consultation with an orthopedic surgeon or sports medicine physician. Thus, initial management includes non-steroidal anti-inflammatory analgesics, crutches, non-weightbearing status, immobilization of the ankle in plantar flexion with a splint, and outpatient referral.

A 13-year-old boy presents to clinic for evaluation of flat feet. He does not complain of pain. On physical exam, the patient has bilateral pes planus when standing. He is able to recreate an arch when he stands on his toes and has normal subtalar motion bilaterally. Which of the following is the most appropriate course of action? Arch supports to stimulate arch development Calcaneal osteotomy to improve foot alignment Counsel the family that painless flat feet is a normal finding and does not require treatment. Resection of the tarsal coalition that is causing flat feet

Correct Answer ( C ) Explanation: This patient has flexible flat feet characterized by collapse of medial longitudinal arch and hindfoot valgus when standing. Approximately 20-25% of the population has flat feet. It is a normal physiologic variant and does not typically cause pain or functional limitations. It is important, however, to differentiate flexible flat feet from a rigid flatfoot. A rigid flatfoot is caused by a congenital tarsal coalition, an osseous, fibrous, or cartilaginous bar between the talus and calcaneus or calcaneus and navicular bones. In a rigid flatfoot, the patient has decreased subtalar motion and the collapsed arch does not reconstitute with toe standing. Most cases of rigid flatfoot are asymptomatic but may result in subtalar or midtarsal pain or increased ankle sprains due to decreased foot motion. Patients found to have asymptomatic flexible flat feet do not require any specific treatment other than patient and parental education.

A 59-year-old woman presents to clinic with complaints of right lateral hip pain which has been ongoing for four months. She is unable to sleep on her right side secondary to the pain. The patient has pain and tenderness with palpation of her greater trochanter. There is no erythema and no soft tissue swelling. The patient has tried nonsteroidal anti-inflammatory drugs and physical therapy for the last six weeks but her pain persists. What is the most appropriate next step in management? Amitriptyline Clindamycin Corticosteroid injection Trochanteric bursectomy

Correct Answer ( C ) Explanation: This patient has greater trochanteric pain syndrome (previously known as trochanteric bursitis) which typically presents as lateral hip pain which is worse with lying on the affected side, and tenderness with palpation of the greater trochanter. Greater trochanteric pain syndrome is the most common cause of lateral hip pain in adults. First-line treatment is conservative therapy, including nonsteroidal antiinflammatory drugs (NSAIDs) and physical therapy. If pain does not adequately respond to these interventions, a corticosteroid injection to the trochanteric bursa is recommended. This can be done with or without ultrasound guidance. The injection can provide both diagnostic and therapeutic benefit.

A 64-year-old man complains of pain and paresthesias in his right hand intermittently for several weeks. He works in a factory putting together electronics. On exam, he has decreased sensation of his right 1st through 4th digits and an atrophied thenar eminence. What test in the ED will help diagnose his condition? CT scan of the head Electromyelography (EMG) Percuss the right volar wrist Urine drug screen

Correct Answer ( C ) Explanation: This patient has median mononeuropathy, also known as carpal tunnel syndrome, a compression neuropathy of the median nerve as it traverses under the flexor retinaculum at the wrist. The median nerve provides sensation primarily to the palmar aspect of the 1st, 2nd, 3rd, and radial side of the 4th. When it is compressed, the patient experiences pain, paresthesias, and numbness in that distribution. The Tinel's test is performed by lightly tapping the volar surface of the wrist over the median nerve. This should elicit a sensation of tingling or pins and needles in the distribution of the median nerve. Carpal tunnel syndrome is first treated with wrist splinting and initiation of a more ergonomic work environment. NSAIDs may also be helpful. If symptoms do not improve, the patient should be referred to a hand specialist who may elect to perform a carpal tunnel release procedure. An EMG (B) is used most commonly by neurologists to confirm damage to peripheral nerves. It may be used for carpal tunnel syndrome if the symptoms do not resolve with conservative management. A variety of heavy metals are associated with a peripheral neuropathy (lead, mercury), but these require special serologic testing, not a urine drug screen (D).

A previously healthy 13-year-old girl presents to your office after a positive scoliosis screening at school. X-ray results reveal a Cobb angle of 15 degrees. Which of the following is the most appropriate next step in management? Bracing Reassurance that no follow-up is necessary Surgical referral Watchful waiting and 6 month follow-up

Correct Answer ( D ) Explanation: Scoliosis is a lateral curvature of the spine. Patients with a Cobb angle of 10-19° should be observed and followed clinically every 6-9 months until they reach skeletal maturity Bracing (A) is recommended for patients with a Cobb angle of 30-39°, although some patients and families will choose bracing with a Cobb angle of 20-29°. Reassurance that no follow up is necessary (B) is correct for patients with a Cobb angle less than 10°, as this does not meet the definition for scoliosis. Surgical referral (D) is indicated for patients with a Cobb angle greater than or equal to 40°.

A 45-year-old left-handed man presents to the Emergency Department with an injury to his left hand. He was playing basketball with his son and developed sudden onset of pain in the distal aspect of his long finger when trying to catch the ball. Exam findings at rest are shown above. The patient is unable to actively extend his distal interphalangeal joint. Which of the following is the most likely diagnosis? Boutonniere finger Jersey finger Mallet finger Trigger finger

Correct Answer ( C ) Explanation: This patient presents with a mallet finger injury, caused by disruption of the terminal extensor tendon distal to the distal interphalangeal (DIP) joint. This injury typically occurs due to an impaction blow to the tip of the extended finger, forcing the DIP joint into flexion. It is common sports related injury and typically affects middle-aged men. The long finger of the dominant-hand is most commonly affected. Physical exam reveals a swollen and painful DIP with the fingertip at rest in 45 degrees of flexion, and patients are unable to actively extend the DIP joint. On X-rays, a bony avulsion can be present. Most mallet injuries are treated with volar splinting of the DIP joint in extension for at least six weeks. Patients should be referred to a hand surgeon for follow-up and to ensure that no surgical intervention is needed. A Boutonniere finger (A) or deformity is caused by rupture of the central slip causing a characteristic DIP joint extension and proximal interphalangeal joint flexion. Jersey finger (B) refers to rupture of the flexor digitorum profundus tendon at the insertion at the distal phalanx and leads to inability to actively flex the DIP joint. Trigger finger (D) describes painless locking and snapping of a finger during flexion due to mismatch in the size of the flexor tendons and the surrounding pulley system.

A 23-year-old man presents with swelling and pain of his elbow. On examination, he has swelling over the olecranon. He has full range of motion at the joint. Vital signs are normal. What management is indicated? Arthrocentesis Aspiration Compression dressing and nonsteroidal antiinflammatory drugs Incision and drainage

Correct Answer ( C ) Explanation: This patient presents with a noninfectious olecranon bursitis that should initially be managed with a compression dressing, ice and NSAIDs. The olecranon bursa frequently becomes inflamed as a result of repetitive minor trauma. Patients will typically present with pain, swelling and tenderness over the olecranon. The inflammation is often obvious on physical examination. Although active range of motion may be limited by pain, patients should have full passive range of motion as the joint is not involved. Noninfectious bursitis should have minimal or no warmth and erythema. Noninfectious bursitis should be treated conservatively with compression dressings, ice, NSAIDs and orthopedic follow up as well as avoidance of the inciting trauma.

A 63-year-old man with hypertension and dyslipidemia presents with pain and swelling of his left great toe. He denies trauma or fever. Examination reveals an exquisitely tender and swollen first metatarsophalangeal (MTP) joint on his left foot. Arthrocentesis yields fluid as shown above. What is the first line therapy for this patient? Acetaminophen Allopurinol Ibuprofen Probenecid

Correct Answer ( C ) Explanation: This patient presents with monoarticular arthritis and a synovial fluid sample consistent with gout. Gout is a systemic disorder that manifests with joint inflammation. It is caused by precipitation of uric acid crystals from extracellular fluid. Hyperuricemia results either from underexcretion of end products of purine metabolism from the kidney or overproduction (more rare). During an episode of gout, polymorphonuclear cells ingest the crystals and release cytokines leading to an inflammatory reaction within the synovium. The most commonly affected joint is the first metatarsophalangeal (MTP) joint followed by the knee. It is important to consider that patients with gout are at a higher risk of developing septic arthritis because the joints are chronically damaged. In patients with a history of gout that present with symptoms concerning for septic arthritis (pain, fever, decreased range of motion) arthrocentesis should be performed to rule out the presence of infection. Management of gout should be split into acute gouty attack treatment and long-term prophylaxis. The mainstay of acute therapy is NSAIDs and colchicine. Indomethacin, naproxen and ibuprofen all may be used in acute treatment. Colchicine inhibits microtubule formation reducing the inflammatory response to uric acid crystals. Unfortunately, the drug has a number of side effects that are almost universally experienced at therapeutic doses (nausea, vomiting, diarrhea).

A 6-year-old boy presents with ankle pain after getting kicked while playing soccer. He has tenderness anteriorly along the ankle joint. An X-ray is shown above. The patient has significant pain when walking. Which of the following is the most appropriate management? Analgesics and follow up in 2 weeks CT scan of the ankle Immobilization of the ankle and non-weight bearing Immobilization of the ankle and weight bearing as tolerated

Correct Answer ( C ) Explanation: This patient presents with signs and symptoms concerning for a Salter-Harris Type I fracture and should be immobilized and made non-weight bearing. Children's bones, unlike those of adults, contain cartilaginous centers near the end of the bone that give rise to new bone growth (epiphysis). Because these areas are radiolucent, they are not visualized on radiographs. Injuries to the epiphysis may result in abnormal bone growth if they do not heal. Therefore, when injury to this area is suspected, conservative management with splinting and non-weight bearing (if lower extremity) status is recommended to promote healing and discourage worsening injury. Injuries to these areas are referred to as Salter-Harris Type I injuries (displacement of the epiphysis). The Salter-Harris classification system was designed to aid in the description and management of pediatric fractures.

A 19-year-old softball player presents to clinic with complaints of bilateral knee pain which has been ongoing for approximately three months. She denies a history of trauma to either knee. The onset has been insidious and slowly increasing over time. She reports worsening of her pain when playing catcher and climbing stairs. She locates the pain over her anterior knees. X-rays are obtained that are normal. Which of the following is the most likely diagnosis? Meniscus tear Osgood-Schlatter disease Osteochondritis dissecans Patellofemoral syndrome

Correct Answer ( D ) Explanation: Patellofemoral syndrome or patellofemoral pain presents as anterior knee pain at or around the patella. The cause is not fully understood but may be associated with damage of the cartilage on the undersurface of the patella. The condition is more common in women than men and often presents initially in the adolescent or young adult. Symptoms are exacerbated by forces that increased the patellofemoral contact pressure such as deep knee bending, stair climbing, or prolonged sitting with the knee bent. Physical exam may reveal quadriceps atrophy, patellar maltracking, pain with compression of the patella, or palpable crepitus under the kneecap.

A 45-year-old woman presents with low back pain. She states that she has had back pain for years but it is worse today. She reports pain in the low back that radiates to her left foot. Examination reveals no midline tenderness to palpation and a positive straight leg raise. She has no weakness and a normal sensory exam. What diagnostic testing is indicated in the Emergency Department? CT scan of the lumbar spine MRI of the lumbar spine No immediate imaging X-ray of the lumbar spine

Correct Answer ( C ) Explanation: This patient presents with signs of symptoms of sciatica, which may be due to a herniated disk and does not require emergent imaging. Sciatica describes a lumbar radiculopathy that is common seen in individuals. Patients typically complain of sharp, burning or shooting pain beginning in the back and radiating down the leg past the knee. There may be associated numbness or weakness or both on the affected side. The pain is often exacerbated with bending, straining or sitting and relieved with lying supine. Physical examination often reveals tenderness in the sciatic notch and a positive straight leg raise. A straight leg raise is performed with the patient lying supine. The symptomatic leg is then passively raised with the knee fully extended. Eliciting back pain radiating down the leg into the knee at 30 - 70 degrees of leg elevation is suggestive of an L5-S1 radiculopathy. The straight leg raise has a good sensitivity (91%) but poor specificity (26%). Sciatica symptoms are typically caused by lumbar disk herniation (sensitivity of 95%). Weakness of ankle dorsiflexion, toe extension, ankle plantar flexion and knee extension are common findings in disk herniation. Immediate imaging is not indicated in patients with symptoms consistent with disk herniation. The majority of patients with disk herniation and sciatica will improve with conservative management alone.

A 16-year-old girl presents with pain to the right index finger after slamming it in a car door. Physical examination reveals a 50% subungal hematoma and intact nail folds. An X-ray shows a non-displaced distal tuft fracture. What management is indicated? Oral antibiotics, splinting and follow up Splinting and follow up Trephination, splinting and follow up Trephination, splinting, oral antibiotics and follow up

Correct Answer ( C ) Explanation: This patient should have trephination performed to relieve pain from the subungal hematoma, be splinted in extension and follow up with a hand surgeon. Subungal hematomas are common after crush injuries to the digits. They are often associated with distal phalanx fractures. Pain associated with these hematomas can be severe as pressure increases under the nail. Management focuses on diagnosing associated injuries and providing pain relief. The easiest way to relieve pain in the digit is by relieving the pressure building under the nail. This can be accomplished with trephination of the nail. Typically, an 18-gauge needle or electrocautery device is placed over the center of the hematoma. Gentle pressure is applied until the nail is penetrated. This typically results in a drop or two of blood exiting through the hole that has been created. The patient typically experiences immediate pain relief. Nail removal is typically unnecessary.

A 72-year-old man with metastatic prostate cancer presents with severe increasing lower back pain. His lower extremity strength is 4/5 and symmetrically weak. He has no bowel or bladder dysfunction and no perianal anesthesia. Which of the following is the most appropriate imaging test? Bone scan CT scan without contrast of the lumbar spine MRI of the thoracic and lumbar spines Radiographs of the lumbar spine

Correct Answer ( C ) Explanation: With a history of cancer and increasing low back pain, the most worrisome diagnosis is spinal cord compression. When cancer becomes metastatic to the spine, tumor can extend into the epidural space causing compression of the spinal cord. Additionally, tumors compress the venous plexus leading to inflammation and vasogenic edema. The most common cancers associated with cord compression are breast, prostate and lung. Classic symptoms include pain, motor deficits and bowel or bladder dysfunction. Bowel and bladder symptoms are late symptoms and their absence does not rule out the possibility of cord compression. Pain is the initial symptom in 95% of patients. The gold standard for diagnosis is MRI and should include both the lumbar and thoracic spine since there are often lesions at both levels

A 35-year-old man presents with a firm, well-circumcised mass on the dorsal aspect of his right wrist. Which of the following physical findings is most consistent with a ganglion cyst? Erythema and warmth Numbness along C7 nerve distribution Positive Spurling test Transillumination

Correct Answer ( D ) Explanation: A ganglion cyst is a benign soft tissue tumor that may affect any joint, but is most often seen in the dorsal aspect of the wrist. The exact pathophysiology is unknown, but it is thought that extra-articular fluid concentrates and is then surrounded by a cyst wall and pedicle that connects the cyst to the nearby synovial joint. Often, ganglion cysts are managed best with watchful waiting as half will spontaneously resolve. For patients who desire removal, aspiration of the cyst and surgical removal are options. However, these treatments have proven to be difficult as there is a high recurrence rate, and surgical removal opens up the possibility for complications, such as infection. Most ganglion cysts are painless, though some patients report pain on activity which is thought to be due to the compression of the cyst on nearby nerves. Physical exam, will reveal a one to two centimeter cystic structure that transilluminates and feels similar to a firm rubber ball. Carpal tunnel syndrome is the consequence of the compression of the median nerve, often leading to numbness along C7 nerve distribution (B), which is not a common symptom of ganglion cysts. A positive Spurling test (C) is indicative of cervical radiculopathy, not the presence of a ganglion cyst. The test is performed by turning the patient's head to their affected side while the examiner applies downward pressure. A positive test is if this maneuver reproduces their radicular symptoms.

Which nerve root is affected in a patient with loss of the ankle jerk reflex? C5 L4 L5 S1

Correct Answer ( D ) Explanation: The S1 nerve root is responsible for the ankle jerk reflex (Achilles reflex). Patients with S1 nerve root lesions will experience a diminished ankle jerk reflex, weakness with foot eversion, numbness along the lateral edge of the foot, and radicular pain along the posterior buttocks extending into the posterior thigh and calf. S1 is one of the most common locations for disc herniation. Disc herniation occurs when the collagenous annulus fibrosis tears, allowing the gelatinous nucleus pulposus to protrude. The C5 (A) nerve root is responsible for the biceps reflex and deltoid strength. L4 (B) is responsible for the knee jerk (patellar) reflex, foot inversion strength, sensation in the anterior thigh, and will cause pain in the posterior buttock that wraps around the front of the thigh and into the medial calf and foot. L5 (C) root lesions will present with intact reflexes, great toe dorsiflexion weakness, numbness in the web between the great and second toe, and pain along posterior buttock radiating into the lateral thigh and leg.

Which of the following is a complication of an untreated mallet finger injury? Boutonniere deformity Dupuytren's contracture Jersey finger Swan-neck deformity

Correct Answer ( D ) Explanation: A mallet finger is caused by a forced flexion of the distal interphalangeal joint (DIP) leading to rupture of the extensor tendon at its insertion at the base of the distal phalanx or bony avulsion of the tendon insertion site. Appropriate treatment includes splinting the distal interphalangeal joint in full extension or slight hyperextension while allowing the proximal interphalangeal joint to have full range of motion. In untreated or under treated cases, a swan neck deformity occurs. Other causes include rheumatoid arthritis and connective tissue disorders (Ehlers-Danlos syndrome). A swan neck deformity is characterized by hyperextension of the proximal interphalangeal joint (PIP) and flexion of the distal interphalangeal joint (DIP). Dupuytren's contracture (B) refers to a thickened and fibrous palmar fascia which is due to fibrous proliferation. It occurs commonly in men over age 40 years, but it also is common in those of Northern European descent, diabetics, alcoholics and epileptics. Boutonniere deformity (A) occurs from forced flexion at the proximal interphalangeal joint (PIP), causing a tear of the central portion of the extensor tendon at the proximal interphalangeal joint (PIP). Patients are unable to fully extend at the proximal interphalangeal joint (PIP) with the wrist and metacarpalphalangeal joints fully extended. Jersey finger (C) occurs in cases such as a football player grabs another player's jersey, avulsing the profundus tendon from its bony insertion.

A 34-year-old woman presents to a local urgent care clinic with insidious onset of right foot pain that has been getting worse over the past two weeks. She denies any known trauma to the foot, but mentions she is training for a marathon. On physical exam, she is moderately tender to palpation over the second right metatarsal. An X-ray of the foot is normal. What is the most likely diagnosis? Compartment syndrome Ligament sprain Osteomyelitis Stress fracture

Correct Answer ( D ) Explanation: A stress fracture is an overuse injury that is caused by repetitive stress. It is most commonly seen in patients with a history of increased exercise (e.g. marathon training) or a history of increased load (e.g. carrying a heavy backpack). Stress fractures occur when the bone breaks after being subjected to repeated stresses, none of which would be large enough individually to cause the bone to fracture. Risk factors for developing a stress fracture include repetitive activities, decreased physical fitness, inadequate vitamin D and calcium intake, eating disorders, female gender, increased age, decreased bone density, and low body mass index. Radiographs are usually the first line imaging study because they can usually be performed in-office and are cost-effective. However, radiographs have poor sensitivity and are usually normal for the first two to three weeks after injury onset. Periosteal elevation, cortical thickening, sclerosis, and true fracture lines are findings indicative of a stress fracture. Repetitive activities predispose to site-specific stress fractures. For example, a baseball pitcher has an increased risk of humeral and scapular fractures and a runner has an increased risk for pelvic and tibial fractures. The lower extremities account for almost half of all stress fractures. Stress fractures usually cause insidious onset of localized pain that increases with activity. Tenderness to palpation over the affected area is the most sensitive physical finding. Soft tissue swelling may or may not be present, depending on amount of soft tissue overlying the bone. Conservative treatment with activity modification, pain control, and nutrition supplementation, if needed, is preferred over surgical management. Slowing increasing amount of activity helps prevent stress fractures.

What is the most common extra-articular manifestation of ankylosing spondylitis? Aortic regurgitation Inflammatory bowel disease Psoriasis Uveitis

Correct Answer ( D ) Explanation: Ankylosing spondylitis is a chronic inflammatory disease that primarily affects the spine and pelvis. With a propensity of affecting men in their 20s and 30s, symptoms include low back pain and stiffness that is insidious in onset, worse in the morning and improves with mild activity. In addition to pain and stiffness in the spine and pelvis, patients often have extra-articular manifestations as well, the most common of which is unilateral uveitis. Occurring in up to 40% of patients with ankylosing spondylitis, uveitis presents with acute onset of unilateral eye pain, photophobia, and blurred vision

A 73-year-old man with a history of arthritis presents with complaints of a low-grade fever and severe right knee pain for the past three days, with an inability to bear weight since this morning. On exam, you note exquisite right knee tenderness and a large effusion. There is limited range of motion both actively and passively, and he refuses to ambulate. You perform an arthrocentesis and drain 20 mL of turbid fluid. Laboratory analysis of the joint fluid reveals the following: WBC of 55,000/µL with 95% neutrophils and a glucose level of 60 mg/dl (serum glucose is 140 mg/dl). Gram stain and crystal analysis are not immediately available. Which of the following is the most likely diagnosis? Acute gout Osteoarthritis Rheumatoid arthritis Septic arthritis

Correct Answer ( D ) Explanation: Any patient with an acute monoarticular arthritis should be considered to have septic arthritis until proven otherwise. Patients at increased risk for septic arthritis include the elderly, those with prosthetic joints, IV drug abusers, and the immunocompromised. Septic arthritis often occurs in patients with a history of chronic arthritis, complicating the diagnosis. In healthy adults, the knee is the most commonly affected joint, but in IV drug abusers, common sites include the sacroiliac, sternoclavicular, and intervertebral joints. In children, the knee and hip are most commonly affected. Synovial fluid results in this case are consistent with a bacterial rather than inflammatory etiology (see table below). Fluid culture will help confirm the diagnosis of septic arthritis, but such results take time to complete. Empiric treatment should be initiated.

A 44-year-old previously healthy man presents to the Emergency Department with low back pain that started two days ago. He recently moved to a new apartment and has been lifting boxes and furniture. He denies any bowel or bladder incontinence. On examination, he is afebrile. He has tenderness with palpation of the right lumbar paraspinal muscle which radiates to the buttock. Reflexes and strength are intact. Pain is exacerbated with movement. What is the next best step in management? Five day course of oral corticosteroids Magnetic resonance imaging of the lumbosacral spine Plain radiographs of the lumbosacral spine Prescribe ibuprofen and discharge home

Correct Answer ( D ) Explanation: Patients with uncomplicated musculoskeletal low back pain present with pain of the lumbar paraspinal muscles which can radiate to the buttock or posterior thigh. Pain is worse with movement of the trunk, but improves with rest. Strength, sensation, and reflexes will be intact and there will be no history of bowel or bladder incontinence. Presence of any of these neurologic findings on examination would be considered a red flag and should prompt further workup with imaging. Management of uncomplicated low back pain includes oral nonsteroidal anti-inflammatory medications (e.g. ibuprofen) and discharge home with primary care follow-up. Oral narcotic medications can be used in the short-term for breakthrough pain, but have increased side effects and are not considered first line treatment. Strict bed rest is no longer recommended as patients who remain active have an earlier resolution of symptoms.

A 42-year-old man presents with foot complaints. Last night, he was awakened with severe pain in his right great toe. He denies a history of recent injury. Examination reveals significant edema and erythema of the first metatarsophalangeal joint. The overlying skin is intact but warm. Range of motion, both passive and active, are limited. Which of the following is the most likely diagnosis? Pediculosis Pellagra Pes planus Podagra

Correct Answer ( D ) Explanation: Arthritis has several causes other than degeneration (OA) and autoimmune dysfunction (RA). Deposition of crystals into the synovium is one of those causes. Gout involves deposition of monosodium urate crystals. Acute, nighttime onset of intense monoarticular edema and pain is typical of gouty arthritis. 50% of cases occur in the great toe's metatarsophalangeal joint, which is termed podagra, while other cases commonly occur in the knee and tarsal joints. Joint aspiration and fluid analysis is recommended. Radiographs may be normal, or may only show soft tissue edema. Serum uric acid levels may be normal during an acute attack of gout, however, interval asymptomatic hyperuricemia commonly exists between, and prior to, the onset of acute attacks. Acute cases are treated with colchicine, indomethacin, intraarticular corticosteroids and decompressive-aspiration. Pellagra (B) is vitamin B3 (niacin) deficiency, characterized by diarrhea, dermatitis and dementia. Desquamation, keratosis and erythema of sun-exposed skin are the common cutaneous findings. Pes planus (C), or flat-feet or fallen-arches, is a foot arch deficiency that results in a majority of the sole of the foot coming in contact with the ground. Ankle eversion predominates. It can result from biomechanical defects, trauma, normal aging, obesity and pregnancy. It is usually involved with ankle pain and/or instability, but not digital joint edema and erythema.

A 12-year-old obese boy presents with groin and proximal anterior thigh pain. He has no pain below the knee. Physical examination shows a significant decrease in internal rotation of the affected hip. Radiographs show a widened growth plate. Which of the following is the most likely diagnosis? Acute transient synovitis Legg-Calve-Perthes disease Osgood-Schlatter disease Slipped Capital Femoral Epiphysis

Correct Answer ( D ) Explanation: Atraumatic pediatric hip pain or limp is usually caused by acute transient synovitis, Legg-Calve-Perthes disease (LCP) or Slipped Capital Femoral Epiphysis (SCFE). SCFE is a fracture in the physis (growth plate) of the femoral head which leads to slippage of the overlying epiphysis. It usually occurs during adolescent growth spurts. Predisposing factors include male sex, obesity and increased sports activities. The typical age range is 10-14 years for girls and 11-16 years for boys. Patients present with pain in the anterior proximal knee or thigh that is exacerbated by activity. On exam, there is loss of hip internal rotation, particularly with the hip flexed. Patients typically walk with the involved extremity externally rotated. AP and frog-leg radiographs show "ice cream falling off the cone." All cases warrant urgent orthopedic evaluation for stabilization surgery. Patients should be nonweight bearing and restricted to bed rest until then. Complications of untreated disease include chondrolysis and osteonecrosis.

A 19-year old man presents to the ED after rolling his left ankle playing basketball earlier in the evening. X-rays of the ankle do not reveal a fracture. The exam reveals a positive anterior drawer test with no endpoint on the left. Which of the following is the most likely diagnosis? Achilles' tendon rupture Grade 1 tear of the calcaneofibular ligament Grade 2 tear of the deltoid ligament Grade 3 tear of the anterior talofibular ligament

Correct Answer ( D ) Explanation: Based on the physical exam findings, a grade 3 tear of the anterior talofibular ligament (ATF) is the most likely diagnosis as the anterior drawer test is specific for the ATF and the lack of endpoint indicates a grade 3 or complete rupture of the ligament. The ATF is the most commonly injured ligament in an inversion sprain and is often the ligament that tears first. Inversion ankle sprains are classified by grade with grade 1 being the least severe and grade three being the most severe generally resulting in instability of the ankle and inability to bear weight. In addition to malleolar tenderness, the inability to bear weight if both at the time of injury and in the emergency department is an indication for X-ray via the Ottawa ankle rules.

Which finding on synovial fluid analysis is most consistent with the diagnosis of pseudogout? Elevated synovial leukocyte count Multiple gram-negative cocci Negatively birefringent urate crystals Positively birefringent calcium pyrophosphate dihydrate crystals

Correct Answer ( D ) Explanation: Calcium pyrophosphate crystal deposition disease, also referred to as pseudogout, is a common crystal-induced arthropathy that generally affects the large joints. Pseudogout has a similar clinical presentation to gout, but the etiology is different. Pseudogout may be idiopathic, especially in the elderly. It may also be caused by trauma, hyperparathyroidism, hemochromatosis, and medications that cause hypomagnesemia such as loop diuretics or proton pump inhibitors used in peptic ulcer disease. Patients present with acute onset of severe pain, inflammation and edema in the knees, ankles, elbows or wrists. Pseudogout is generally monoarticular, but may present in multiple joints as well. Diagnosis is by synovial fluid analysis, so arthrocentesis is necessary for patients with monoarticular arthritis. Synovial fluid analysis will show positively birefringent calcium pyrophosphate dihydrate crystals, which confirms the diagnosis. Initial treatment for pseudogout is with nonsteroidal anti-inflammatory drugs (NSAIDs) or colchicine.

A 56-year-old man has pain and tingling in the medial aspect of his ankle and the plantar aspect of his foot. He jogs 3 miles daily and has no history of any injury. The symptoms are aggravated by activity and sometimes keep him awake at night. The only findings on examination are paresthesias when a reflex hammer is used to tap just inferior to the medial malleolus. Which of the following is the most likely diagnosis? Diabetic neuropathy Plantar fasciitis Stress fracture Tarsal tunnel syndrome

Correct Answer ( D ) Explanation: Entrapment of the posterior tibial nerve or its branches as the nerve courses behind the medial malleolus results in a neuritis known as tarsal tunnel syndrome. Causes of compression within the tarsal tunnel include varices of the posterior tibial vein, tenosynovitis of the flexor tendon, structural alteration of the tunnel secondary to trauma, and direct compression of the nerve. Pronation of the foot causes pain and paresthesias in the medial aspect of the ankle and heel. Electromyography and nerve conduction velocity studies may be a useful initial tool in evaluating suspected cases of tarsal tunnel syndrome and in confirming the presence of neuropathy. Medical therapy for tarsal tunnel syndrome may consist of local injection of steroids into the tarsal canal. Physical therapy may be of some value in reducing local soft-tissue edema, thereby easing pressure on the compartment. When conservative therapy fails to alleviate the patient's symptoms, surgical intervention may be warranted

A 45-year-old woman presents to your office with a complaint of pain "all over." She tells you that over the past few years her pain has worsened and she also experiences daily fatigue, difficulty concentrating, and headaches. Physical exam reveals multiple points of tenderness to palpation. Which of the following lab results would be expected? Decreased vitamin B12 Elevated erythrocyte sedimentation rate Elevated thyroid stimulating hormone Normal complete blood count

Correct Answer ( D ) Explanation: Fibromyalgia is a syndrome that causes chronic, widespread pain and tenderness. The etiology of fibromyalgia is unknown and the pathogenesis has not been proven, however it is believed to be related to a dysfunction in central pain processing. In addition to complaints of pain, patients often present with fatigue, cognitive difficulties, and multiple somatic and psychiatric symptoms. Fibromyalgia is a diagnosis of exclusion and patients should be evaluated for other disorders before making the diagnosis. Physical exam findings are generally within normal limits other than tenderness to palpation of multiple soft tissue points. Laboratory test results, including complete blood count, are within normal limits. There is no cure for fibromyalgia and management involves a multi-disciplinary approach including education, lifestyle modifications and medication. Patients should be counseled regarding proper diet, physical exercise, stress reduction, and healthy sleep habits. An initial pharmacologic approach with monotherapy using amitriptyline, duloxetine, pregabalin or milnacipran is recommended. Prognosis for patients with fibromyalgia varies, with better outcomes seen when patients seek help, engage in more physical exercise, pace their activities and in those that have less guarding during physical exam

A young woman presents with ankle pain and edema. While wearing high-heel shoes, she twisted her ankle upon stepping off a curb. She is tender about the lateral malleolus. Skin and neurovascular examination are normal. She has no medial tenderness. Ligament testing is negative. Radiographic examination reveals a non-displaced lateral malleolar fracture below the ankle joint. The tibia is unaffected. Which of the following is the most appropriate definitive treatment for this patient? AClosed reduction BDebridement CNon-weight-bearing orthosis DWeight-bearing cast

Correct Answer ( D ) Explanation: Fracture of the ankle may include injury to the medial malleolus (tibia), the lateral malleolus (fibula), the posterior malleolus (tibia), the talus and the collateral ligaments. Stability of the fracture depends on how many sides are injured. Stable fractures involve only one side of the joint, whereas unstable fractures include both sides of the joint. Stable fractures are treated with 4-6 weeks of a weight-bearing cast or brace

A 51-year-old man presents to emergency room with severe pain in his right great toe. He has a history of chronic gout and has had multiple, similar flares. He recently began taking a new medication to control his cholesterol. Which of the following medications did this patient most likely begin taking? Atorvastatin Ezetimibe Gemfibrozil Niacin

Correct Answer ( D ) Explanation: Niacin is used to treat hyperlipidemia and can exacerbate gout. Niacin or nicotinic acid naturally occurs as Vitamin B3. When niacin is used in doses 100-300 times the recommended daily allowance, a reduction in triglycerides and low-density lipoproteins (LDL) can be observed. Niacin is thought to decrease very low-density lipoprotein synthesis, thereby lowering LDL. Additionally, niacin partially inhibits release of free fatty acids from adipose tissue and increases the rate of lipoprotein lipase activity, which lowers blood levels of triglycerides. The therapeutic use of niacin is limited due to its extensive side effects. Most notably, niacin causes prostaglandin-mediated flushing and pruritus that is intolerable for many patients. Slowly increasing the niacin dose, using slow-release preparations, taking niacin with food, and taking aspirin 30 minutes before the niacin dose may attenuate adverse effects. Niacin is contraindicated in patients with hepatic disease, severe hypotension, or active peptic ulcer disease. Niacin can exacerbate hyperuricemia and hyperglycemia and should be avoided in patients with chronic gout or diabetes. Baseline liver transaminases, uric acid, and glucose should be obtained before initiation, after increasing dose, and every six months at maintenance dose. Other medications that can cause a gout flare include tacrolimus, cyclosporine, loop and thiazide diuretics, and low-dose aspirin. Atorvastatin (A), an HMG-CoA reductase inhibitor, is not known to exacerbate gout. The most common side effects of atorvastatin are headache, gastrointestinal complaints, and myalgia. Ezetimibe (B) is a cholesterol absorption inhibitor that has minimal side effects. Side effects of ezetimibe include headache and diarrhea. Gemfibrozil (C), a fibric acid derivative, has minimal side effects of nausea, vomiting, flatulence, and constipation and is generally well tolerated. Gemfibrozil is contraindicated in patients with a history of gallstones.

A 72-year-old woman with osteoporosis sees you for health maintenance care. She currently takes vitamin D 200 IU/day and calcium 1500 mg/day. Her most recent bone mineral density is 2.5 standard deviations below the norm. Which of the following is the most appropriate treatment regimen? Continue calcium, continue vitamin D and add estrogen Continue calcium, increase vitamin D and add colchicine Increase calcium, continue vitamin D and add calcitriol Increase vitamin D, continue calcium and add alendronate

Correct Answer ( D ) Explanation: Osteoporosis, a decrease in bone mass, can be classified as primary and secondary. Primary osteoporosis is the most common type of osteoporosis. It is more common in women than men. A person reaches peak bone density at about age 30. After that, the rate of bone loss slowly increases, while the rate of bone building decreases. Whether a person develops osteoporosis depends on the density of the bones in early life as well as health, diet, and physical activity at all ages. In women, accelerated bone loss usually begins after monthly menstrual periods stop. This happens when a woman's production of estrogen slows down (usually between the ages of 45 and 55). In men, gradual bone thinning typically starts at about 45 to 50 years of age, when a man's production of testosterone slows down. Osteoporosis usually does not have an effect on people until they are 60 or older. Women are usually affected at an earlier age than men, because they start out with lower bone mass. Secondary osteoporosis has the same symptoms as primary osteoporosis. But it occurs as a result of having certain medical conditions, such as hyperparathyroidism, hyperthyroidism, or leukemia. It may also occur as a result of taking medicines known to cause bone breakdown, such as oral or high-dose inhaled corticosteroids (if used for more than 6 months), too high a dose of thyroid replacement, or aromatase inhibitors (used to treat breast cancer). Secondary osteoporosis can occur at any age. Dual-energy X-ray absorptiometry (DEXA) scan measures loss of bone mineral density (BMD) and is the most accurate and precise method to diagnose osteoporosis. Treatment begins with prevention, and includes early adulthood adequate calcium and vitamin D intake, regular weight-bearing exercise and tobacco and alcohol avoidance. The recommended calcium intake for those aged 19-50 years is 1000 mg/day, and those aged 51 years and older, 1200mg/day. Recommendations for vitamin D intake are 400 IU/day for young adults, and 800-1200 IU/day for the elderly. Pharmaceuticals are recommended when bone mineral density scores are below 1.5 to 2 standard deviations below the young-adult-norm, based on the presence (≤1.5) or absence (≤2) of fracture risk factors. Prescription options include estrogen replacement therapy, alendronate (or other bisphosphonate), calcitonin and raloxifene (a selective estrogen receptor modulator). This patient is only taking 200 IU/day, but should be taking 800-1200 IU/day.

A 30-year-old woman complains of anterior knee pain that gets worse with prolonged sitting, going up and down stairs and with deep squats. She has no known history of knee injury. She exercises three times a week on her exercise bicycle. She is average weight and played basketball as a high school student. Which of the following is the most likely diagnosis? Chronic anterior cruciate ligament tear Osgood-Schlatter disease Osteochondritis dissecans Patellofemoral syndrome

Correct Answer ( D ) Explanation: Patella-related pain is the single most common cause of knee pain. Patellofemoral pain syndrome is a multifactorial syndrome characterized by aching anterior knee pain that worsens with activities that stress the patellofemoral joint (climbing stairs, kneeling). Patients complain of diffuse, aching anterior knee pain that is exacerbated by loaded flexion activities such as stair climbing, jumping, or prolonged sitting (theater sign). On exam there may be patellar crepitation. The diagnosis is clinical and knee radiographs provide limited information (articular cartilage loss). Treatment includes activity modification and an exercise program consisting of quadriceps strengthening (medial quadriceps) and hamstring flexibility. NSAIDs are recommended for symptomatic care. Osteochondritis dissecans (C) is due to repetitive small stress to the subchondral bone that leads to osteonecrosis (most commonly the medial femoral condyle). Most cases begin in childhood although individuals may not become symptomatic until late adolescence or early adulthood. Early diagnosis is critical, as the injury has a better potential to heal while the bones are still growing. Osgood-Schlatter disease (B) is caused by rupture of the growth plate at the tibial tuberosity, which causes stress on the patellar tendon. It most commonly occurs in rapidly growing adolescents (10-15-years-old) and is five times more common among those active in sports and up to three times more common in boys.

A 16-year-old girl presents to clinic with 3 weeks of worsening right knee pain. She attends dance class since age 5 and now dances 20 hours a week. She denies any inciting injury and continues to dance on the injured leg. On exam, she has pain at the inferior and medial pole of the right patella with no swelling or erythema of the knee. Which of the following is the most likely diagnosis? Anterior cruciate ligament rupture Medial meniscus tear Patella dislocation Patellofemoral pain syndrome

Correct Answer ( D ) Explanation: Patellofemoral pain syndrome occurs in about a quarter of those involved in athletics and more commonly in women and those between the ages of 10 and 35 years. Symptoms include pain with going up and down stairs and prolonged sitting or squatting. Individuals may have a sensation of the knee buckling or giving way. Swelling, popping or grinding sensations may be present. Treatments include non-steroidal anti-inflammatory medications, ice, quadriceps strengthening, stretching, patella bracing and orthotics.

One of your patients has just been diagnosed with a mixed connective tissue disorder, but she does not know which one. However, she remembers hearing the rheumatologist talk about "too much collagen". Which one of the following autoimmune diseases does this patient most likely have? Polymyositis Sjogren's syndrome Systemic lupus erythematosus Systemic sclerosis

Correct Answer ( D ) Explanation: Progressive systemic sclerosis (SS), sometimes referred to as scleroderma, is a chronic progressive connective tissue disease, marked by excessive skin and organ-epithelial collagen production and deposition. In other words, multisystem fibrosis is present and the body becomes "scarred". It is an autoimmune disease of unknown origin, but is associated more with certain factors, such as exposure to silica, organic solvents, aliphatic hydrocarbons, epoxies, pesticides, vibratory tools, certain cosmetics, cocaine and drugs such as bleomycin and carbidopa. It is a rare disease. Signs and symptoms include fatigue, weakness, inflammatory myopathy, myalgias, arthralgias, dependent edema, joint contracture, pruritic thick skin, dysphagia, vomiting, abdominal cramping, diarrhea, dyspnea, nonproductive cough, atypical chest pain and palpitations. The main presenting manifestations are myalgias, arthralgias and Raynaud's phenomenon, a cold-induced, distal digital arteriolar contraction which leads to pallor, ischemia and ulceration. Skin findings predominate on physical exam, and include face, neck, trunk and proximal arm taut thick skin, skin atrophy, areas of hyper- and hypopigmentation, telangiectasias and Raynaud's findings. Lab abnormalities include increased ESR and CRP, thrombocytopenia and microangiopathic hemolytic anemia, hypergammaglobulinemia and increased creatinine phosphokinase. Abnormal renal indices are present if there is kidney involvement. Antinuclear antibodies are positive and anticentromere antibodies are negative in most patients.

A 29-year-old woman is concerned with a rash. On inspection, you appreciate a central facial rash. You notice that the nasolabial folds are not affected. You also detect alopecia and lip crusting. The neck, chest and trunk are void of cutaneous abnormalities. The dorsal digits, and other extensor surfaces, are not affected. Which of the following is the most likely diagnosis? Dermatomyositis Psoriasis Rosacea Systemic lupus erythematosus

Correct Answer ( D ) Explanation: Rashes can be difficult to evaluate, especially when considering dermatologic versus rheumatologic conditions. A crossover occurs in four main conditions: rosacea, systemic lupus erythematosus (SLE), psoriasis and dermatomyositis. SLE is associated with a red, butterfly-shaped, malar rash, which is located on the central face but not in the nasolabial folds. In comparison, rosacea's malar rash affects the central face and nasolabial folds. Furthermore, systemic symptoms do not occur in rosacea. Facial rash can also present with dermatomyositis, however, this central facial rash is not termed malar, but rather heliotrope, and appears as a waxing-waning violet-colored (violaceous) rash that mainly affects the eyelids but also affects the nasolabial folds. To add to this difficulty, SLE is associated with another rash called discoid rash. This is described as raised chronic inflammatory sores with scarring and scaling, which occur on the face, ears and scalp, and can easily be confused with psoriasis. Patients with SLE may have a malar rash, or discoid rash, or both.

A 22-year-old previously healthy man presents complaining of a month of fever, red eyes, and joint pain. Physical exam reveals brownish-red macules with hyperkeratotic crusted centers on the palms of his hands. He was recently treated for chlamydial urethritis. What is the most likely diagnosis? Coxsackievirus infection Henoch-Schönlein purpura Kaposi sarcoma Reactive arthritis

Correct Answer ( D ) Explanation: Reactive arthritis is a form of arthritis that follows enteric or genitourinary pathogens. The most common triggers are Salmonella, Campylobacter, Shigella, Yersinia, and Chlamydia, though other infections can also initiate reactive arthritis. The course is usually self-limiting and resolves after 3-12 months, although 10-20% of patients may experience chronic deforming arthritis. Management is often supportive, consisting of NSAIDs and topical steroids for possible cutaneous involvement. The diagnosis of reactive arthritis is clinical, and based on the presence of arthritis, urethritis, and conjunctivitis in the setting of a recent infection. Accompanying these symptoms are often brownish-red papules or macules that eventually become hyperkeratotic and crusted on the palms and soles. A patient with a recent chlamydia infection who presents with fever, conjunctivitis, joint pain, and skin lesions is most likely to have Reiter syndrome.

A 39-year-old woman presents to your office with complaints of pain and swelling in her hands and feet. She has not been feeling well for the past few months and has been experiencing morning stiffness in her hands. Laboratory testing reveals an elevated erythrocyte sedimentation rate and a positive anti-cyclic citrullinated peptide antibody test. Which of the following is the most likely diagnosis? Fibromyalgia Polymyalgia rheumatica Reactive arthritis Rheumatoid arthritis

Correct Answer ( D ) Explanation: Rheumatoid arthritis (RA) is a chronic, systemic, inflammatory disease that causes joint inflammation and synovial hypertrophy. The etiology of RA is unknown, but symptoms occur as a result of an autoimmune reaction. Risk factors include female gender, first-degree relatives with RA, and cigarette smoking. Symptoms are typically polyarticular with a gradual onset. The classic presentation of RA includes insidious onset of pain, morning stiffness and swelling of multiple joints. Diagnostic criteria for RA includes inflammatory arthritis with three or more joints, duration of at least six weeks, testing to rule out diseases with similar clinical features, and laboratory testing showing positive rheumatoid factor or anti-cyclic citrullinated peptide antibody plus elevated erythrocyte sedimentation rate or C-reactive protein. Treatment should follow an integrated approach with both pharmacologic and nonpharmacologic treatment. Nonpharmacologic treatment includes diet, exercise, stress reduction, counseling, physical therapy and massage. Pharmacologic treatment includes nonsteroidal anti-inflammatory drugs, nonbiologic and biologic disease-modifying antirheumatic drugs, corticosteroids and immunosuppressants.

A 27-year-old woman presents to your clinic complaining of right wrist pain following a fall on her outstretched hand. Physical exam reveals tenderness over her anatomical snuffbox. Radiographs of her right wrist show no acute fracture. What is the most appropriate next step in management for this patient? Reassurance Repeat X-rays in 10-14 days Surgical consult Thumb spica cast

Correct Answer ( D ) Explanation: Scaphoid fractures are at risk of non-union due to the potential interruption of blood supply from the palmar carpal branch of the radial artery to the proximal pole of the scaphoid. The most common mechanism of injury is a fall on an outstretched hand. These fractures often do not show up on X-rays taken immediately after the causative injury. Therefore, any patient with a suspicious mechanism of injury and snuffbox tenderness should be treated as a scaphoid fracture, regardless of radiographic findings. The appropriate management of a scaphoid fracture is immobilization in a thumb spica cast. While further imaging is necessary in confirming a scaphoid fracture, repeat X-rays in 10-14 days (B) with no immobilization is not appropriate. Until a fracture is confirmed, a surgical orthopedic consult (D) is not indicated.

A 56-year-old woman with a history of Raynaud's phenomenon presents complaining of swollen, painful fingers. Physical examination reveals tender ulcerations at the tips of her fingers. What is the most likely diagnosis? Dermatomyositis Lupus erythematosus Nodular vasculitis Scleroderma

Correct Answer ( D ) Explanation: Scleroderma is a multi-system disorder that causes inflammatory, vascular, and sclerotic changes to the lungs, heart, GI tract, and skin. There are two subsets: limited systemic scleroderma (lSSc) and diffuse systemic scleroderma (dSSc). lSSc is more common than dSSc, but both affect women more often than men, and both progress from cutaneous to internal involvement, resulting in systemic symptoms such as dysphagia, diarrhea, weight loss, and exertional dyspnea. lSSc progresses at a slower rate than dSSc, providing a more favorable prognosis, though symptomatic control remains a challenge for both subsets. Systemic glucocorticoids and immunosuppressive drugs have been shown to have some benefit for early disease, but other systemic treatments have not proven to be effective for long-term management. Diagnosis is often clinical, and based on cutaneous findings such as Raynaud's phenomenon and skin sclerosis. Physical exam will often reveal small painful ulcerations at the fingertips, such as in this patient's case. This finding is called "rat bite necrosis" and is characteristic of scleroderma

A 32-year-old man presents with acute onset of a hot, swollen left knee. The right knee and other peripheral joints are unaffected. Skin lesions are absent. Cardiopulmonary, genitourinary and neurologic examinations show no gross abnormalities. Laboratory examination reveals a leukocytosis. An X-ray shows normal joint space and absent osteophytes. Which of the following is the most likely diagnosis? Lyme disease Osteoarthritis Rheumatoid arthritis Septic arthritis

Correct Answer ( D ) Explanation: Septic, or pyogenic, arthritis usually occurs after direct inoculation or hematogenous spread into a joint. In adults, there may be an underlying arthritis, whether degenerative or rheumatologic, in the joint. Common causative organisms in adults are Staphylococcus aureus and Neiserria gonorrhoeae. Joint edema, warmth, decreased range and protected use are common symptoms. It is important during any examination of suspected septic arthritis to search for a primary source, such as a penetrating wound, local abscess or skin infection. Laboratory examination includes WBC, CRP and ESR. A joint aspirate (arthrocentesis) should be sent for culture, Gram stain and fluid analysis. Blood cultures are also routinely drawn. Broad-spectrum intravenous antibiotics are started immediately. Surgical decompression and drainage are considered if a patient doesn't respond to antibiotics in 24-48 hours, or if the affected joint is the hip. Common antibiotic choices include oxacillin, cefazolin, ceftriaxone and aminoglycosides.

A 10-year-old boy presents to clinic with complaints of right posterior heel pain. There was no preceding injury. The pain has been ongoing for four months. He is a soccer player and notices the pain primarily when wearing his cleats during soccer. On physical exam, the patient has pain with medial-lateral compression of the calcaneus. Which of the following is the most likely diagnosis? Achilles tendon rupture Iselin's disease Plantar fasciitis Sever disease

Correct Answer ( D ) Explanation: Sever disease, also known as calcaneal apophysitis is believed to be a traction apophysitis of the Achilles tendon at the calcaneal apophysis related to overuse. It is most commonly seen in children ages 8-12 years and the pain is often associated with sports that use cleats. Positive physical exam findings include pain with medial-lateral compression of the calcaneus (positive compression test) or pain with direct palpation over the apophysis. Radiographs are not required to make the diagnosis but may show increased sclerosis or fragmentation of the calcaneal apophysis. X-rays can be used to rule out other causes of heel pain if the diagnosis is unclear. Symptomatic management is the mainstay of treatment including the use of heel cups, heel cord stretching exercises, and rest and nonsteroidal anti-inflammatory drugs as needed. Iselin's disease (B) is a traction apophysitis of the tuberosity of the fifth metatarsal at the attachment of the peroneus brevis tendon. Plantar fasciitis (D) is uncommon in children and adolescents and generally presents as pain over the medial calcaneus which can radiate into the arch. The pain is most prominent with the first steps of the day.

Which of the following is characteristic of Sjögren's syndrome? Granuloma formation with necrotizing vasculitis of the small arteries and veins Infection of the upper respiratory tract with group A streptococci Inflammation of the medium and large-sized arteries Lymphocytic infiltration of the exocrine glands

Correct Answer ( D ) Explanation: Sjögren's syndrome is an autoimmune disease characterized by lymphocytic infiltration of the exocrine glands. This infiltration results in xerostomia (dry mouth) and dry eyes. Sjögren's syndrome can present as a primary disorder or as secondary to other autoimmune rheumatic diseases such as rheumatoid arthritis, scleroderma or systemic lupus erythematosus. Treatment is aimed at relief of symptoms and limiting the damage caused by chronic xerostomia and dry eyes. Other autoimmune diseases have very distinct presentations. Granuloma formation with necrotizing vasculitis of small arteries and veins (A) is seen in the histology of Wegener's granulomatosis (Granulomatosis with polyangiitis). Acute rheumatic fever is caused by infection of the upper respiratory tract with group A streptococci (B). Inflammation of medium and large-sized arteries (C) is characteristic of giant cell arteritis. These diseases all differ from Sjögren's syndrome in that there is no involvement with the exocrine glands.

A 13-year-old obese boy presents with complaints of right anterior thigh pain and a limp for 2 weeks. There is no history of trauma. On exam, he is able to bear weight but walks with his right foot externally rotated. He has decreased motion of his right hip and when his hip is flexed, the extremity externally rotates. The patient is afebrile. White blood cell count is 8,000. Which of the following is the most likely diagnosis? Developmental dysplasia of the hip Legg-Calve-Perthes disease Septic arthritis of the hip Slipped capital femoral epiphysis

Correct Answer ( D ) Explanation: Slipped capital femoral epiphysis (SCFE) occurs when the head of the femur (epiphysis) slips relative to the femoral neck. This occurs at the point of weakness in the growth plate (physis). The single greatest risk factor is obesity and is more common in boys than girls. The average age is 12 for girls and 13 for boys. The patient often presents with a history of pain in the groin or anterior thigh although they may also present with referred pain in the knee. Patients usually present with a limp but may be unable to bear weight secondary to pain. On exam, the patients will have decreased range of motion of the hip and obligatory external rotation, where the leg externally rotates with hip flexion. X-rays demonstrate widening of the physis and slipped epiphysis on a lateral view of the hip, confirming the diagnosis. Developmental Dysplasia of the hip (A) is a congenital hip disorder in which there is abnormal development of the acetabulum and proximal femur. Hip dysplasia may present with a limp but this usually is apparent shortly after onset of walking and is generally painless at onset. Legg-Calve-Perthes Disease (B) is a form of avascular necrosis of the hip. Patients with Legg-Calve-Perthes present with a limp; hip or thigh pain may or may not be present. The peak incidence is in a younger patient, ages 4-8, and is more common in boys than girls.

A 62-year-old woman presents complaining of ankle pain following a fall from standing onto the ice last night. She is able to walk across the Emergency Department and back, but states that the pain becomes unbearable toward the end. Her posterior lateral malleolus and her anterior medial malleolus are tender to palpation. Which of the following is the correct indication for ankle radiography in this patient? Age Inability to bear weight after several steps Tenderness over the anterior medial malleolus Tenderness over the posterior lateral malleolus

Correct Answer ( D ) Explanation: The Ottawa Ankle Rule is a clinical decision-making rule that was developed to allow clinicians to effectively select patients who are candidates for ankle radiography after acute ankle trauma. It consists of five criteria: tenderness over the posterior lateral malleolus, tenderness over the posterior medial malleolus, tenderness over the navicular bone, tenderness over the base of the fifth metatarsal, and inability to bear weight for four steps. If any one of the criteria are present, the patient is a candidate for plain films of the ankle.

Which of the following is the most reliable indication of an achilles tendon rupture? Inability to ambulate Inability to plantar flex the foot Pain along the posterior ankle Positive calf squeeze test

Correct Answer ( D ) Explanation: The achilles tendon is formed by the convergence of the gastrocnemius and soleus muscles and attaches on the posterior calcaneus. Achilles tendon ruptures tend to occur when sudden force is placed on the achilles tendon during physical activity that involves pivoting on the foot or sudden acceleration. The majority of ruptures occur in tendons that are already weakened from previous injury or degeneration. Both partial and complete ruptures can occur. Men, typically in their 30s and 40s, are significantly more likely to have an achilles tendon rupture. Patients often present with a history of sudden pain and a "popping" sensation in the posterior ankle during physical activity. On examination, there may be edema and ecchymosis over the achilles tendon and a palpable defect may be appreciated. The calf squeeze test, or Thompson test, is the most reliable indicator of an achilles tendon rupture with a sensitivity of 96%. With the patient prone or kneeling on a chair, the posterior calf is squeezed at its widest point. The absence of plantar flexion indicates a rupture of the tendon. Management includes immobilization with a short leg cast with the ankle in slight plantar flexion, known as the equinus position, and orthopedic consultation

A 45-year-old man presents with tenderness on his right elbow that has worsened over the past 24 hours. On physical examination, his right elbow is erythematous, tender and warm to palpation. Range of motion is significantly reduced in the right elbow compared with the left elbow. Which organism is the most common etiology for this disease? Escherichia coli Group A streptoccocus Pseudomonas aeruginosa Staphylococcus aureus

Correct Answer ( D ) Explanation: The acute onset of monoarticular joint pain, erythema, heat, and immobility should raise suspicion of septic arthritis. Constitutional symptoms such as fever, chills, and rigors are poorly sensitive for septic arthritis. Because the clinical presentation of septic arthritis may overlap with other causes of acute arthritis, arthrocentesis is needed to identify the causative infectious agent. Synovial fluid should be evaluated at the bedside and then sent for white blood cell count with differential, crystal analysis, Gram stain, and culture. Staphylococcus aureus is the organism most commonly found in patients with septic arthritis in the United States and other developed countries. Empiric intravenous antibiotic treatment of septic arthritis should be based on the organism found in the Gram stain of the synovial fluid, or on the suspicion of a pathogen from the patient's clinical presentation. Treatment options include vancomycin for gram-positive cocci, ceftriaxone for gram-negative cocci, and ceftazidime for gram-negative rods. The duration of therapy in patients with nongonococcal septic arthritis is typically three to four weeks. In addition to antibiotic therapy, evacuation of purulent material is necessary.

A 57-year-old man presents to the emergency room after awakening with severe pain in his left big toe. He ate at steakhouse and had several bottles of beer the night before. He has a past medical history of chronic kidney disease. His latest creatinine clearance was 25 mL/min. Joint aspiration reveals negatively birefringent crystals. Which of the following is the most appropriate treatment? Allopurinol Colchicine Indomethacin Prednisone

Correct Answer ( D ) Explanation: The man in this case has gout and should be managed with prednisone. Acute management of gout includes nonsteroidal anti-inflammatory drugs (NSAIDs), colchicine, and corticosteroids. Gout is the most common crystal-induced arthropathy. Gout may be classified as either primary or secondary. Primary gout is caused by underexcretion or overproduction of uric acid. Secondary gout is related to medications or conditions that induce hyperuricemia. Risk factors for primary gout include genetic predisposition, being male, increasing age, hypertension, and diets high in purines, fructose, and alcohol. Gout most commonly presents as acute pain in the metatarsal-phalangeal joint of the great toe (podagra). The base of the great toe is the first manifestation in approximately half of gout cases. In the majority of cases, a single joint is affected. Involvement of other joints, such as the ankle or wrist can occur, but is more likely in recurrent flares. Classic symptoms of an acute gout flare are severe pain, redness, warmth, and swelling of the affected joint. Onset is commonly sudden and nocturnal. Tophi may be observed in patients with a history of chronic gout. Synovial fluid analysis will have characteristic needle-shaped, negatively birefringent crystals under a polarized light microscope. White blood count of the joint fluid ranges between 10,000-100,000 cells with the majority being neutrophils. Serum uric acid levels during an acute flare are unreliable; values can be low, normal, or high. Treatment of an acute flare involves reducing inflammation and pain. Treatment at the first sign of a flare reduces the duration of the attack. NSAIDs are generally the first line treatment options if there are no contraindications, such as renal failure or active peptic ulcer disease. While indomethacin is traditionally used, most NSAIDs can be used. Colchicine is used less commonly today due to narrow therapeutic window and many side effects. Colchicine should be used with caution in renal failure. Corticosteroids, either orally, intravenously, intramuscularly, or intra-articularly, are the treatment of choice when colchicine and NSAIDs cannot be used. Oral steroids should be tapered to avoid a rebound flare.

A 35-year-old man presents to the ED after injuring himself while playing basketball. He states he felt a pop in his right lower extremity while jumping for a rebound. He can ambulate, but with a limp. On exam, squeezing his right calf elicits no plantar flexion of his foot. What is the appropriate disposition for this injury? AAdmission to hospital for serial compartment pressure measurements BElastic wrap, crutches, weight-bearing as tolerated, and orthopedic follow-up within seven days CSplint in dorsi-flexion, keep nonweight-bearing, orthopedic follow-up DSplint in plantar-flexion, keep nonweight-bearing, orthopedic follow-up

Correct Answer ( D ) Explanation: The patient has an Achilles tendon rupture suggested by the inability of his foot to plantar flex with squeezing of the calf (Thompson test). This injury is most common in middle-aged recreational athletes. Patients often note a "popping" sensation followed by acute weakness and the inability to continue activity. A defect may be palpable on examination of the distal Achilles tendon. Management in the ED includes immobilization of the extremity in a short-leg splint, keeping the foot in plantar-flexion. The patient should be nonweight-bearing until seen by an orthopedic surgeon. Definitive repair is surgical.

A 24-year-old man presents with wrist pain after a fall on his outstretched right hand. Examination reveals tenderness at the base of the first metacarpal in the anatomic snuffbox and pain with axial load on the thumb. The patient's X-ray is shown above. What management is indicated? Acetaminophen and primary care follow up Removable soft wrist splint for 2 weeks Sugar tong splint and orthopedics follow up Thumb spica splint and orthopedics follow up

Correct Answer ( D ) Explanation: The patient has an examination concerning for an occult scaphoid fracture and requires immobilization with a thumb spica splint and follow up with an orthopedic surgeon. The scaphoid bone is the most commonly fractured carpal bone. It typically occurs after a fall on an outstretched hand (FOOSH). There are three types of fractures: 1) fractures of the tuberosity and distal pole, 2) fractures of the waist and 3) fractures of the proximal pole. Patients present with pain at the anatomic snuff box or distal radius. Physical examination reveals tenderness and swelling of the anatomic snuff box and may have increased pain with axial compression of the first metacarpal (Watson's scaphoid shift test). Additionally, pain may be increased with thumb to index finger pinch. Diagnostic testing should begin with AP, lateral and oblique plain radiographs of the wrist. A scaphoid view (X-ray with wrist in ulnar deviation) can increase the likelihood of detecting small scaphoid fractures on X-ray. Unfortunately, plain radiographs miss 15% of scaphoid fractures. Patients with missed fractures that are not immobilized are at an increased risk for fracture nonunion and long-term chronic arthritis. Therefore, splint immobilization with a thumb spica is indicated in all patients who have a clinical suspicion for scaphoid fractures regardless of the X-ray findings.

A patient presents with ankle pain after twisting it while walking. Which of the following clinical findings necessitates an ankle X-ray? Age >50 Tenderness to the anterior edge of the lateral malleolus Tenderness to the anterior edge of the medial malleolus Tenderness to the posterior edge of the lateral malleolus

Correct Answer ( D ) Explanation: The patient presents with an injury suspicious for an ankle sprain and the decision to perform an X-ray can be guided by the Ottawa Ankle Rule. Ankle sprains refer to ligamentous injury to the ankle. In patients presenting with ankle pain after trauma, it is important to determine the presence or absence of a fracture as this may guide management. The Ottawa Ankle Rule was developed to reduce the number of X-rays for ankle injuries. Studies have found it to have a sensitivity near 100%. Application reduces the use of X-ray by 36%. Below are the components of the rule. Therefore, tenderness to the posterior edge of the lateral malleolus necessitates an X-ray. There is no specific age cutoff (A) for the Ottawa Ankle Rules and it has been successfully applied in children over the age of 6. Tenderness anterior to the lateral malleolus (B) is not a criterion for an X-ray and this is commonly seen in patients who sprain the anterior talofibular ligament (ATFL). Tenderness to the anterior edge of the medial malleolus (C) is also commonly seen in ankle sprains.

A 40-year-old man who plays weekend baseball as a pitcher presents to the office stating that for the last few months his right shoulder feels like it is "going dead." He states that the pain is progressively worsening and now he is experiencing weakness with overhead activity. He also believes that he is throwing the baseball slower than previously. Which of the following is the most likely diagnosis? Acromioclavicular joint injury Adhesive capsulitis Glenohumeral joint dislocation Rotator cuff tear

Correct Answer ( D ) Explanation: The rotator cuff is made up of the Supraspinatus (abduction), Infraspinatus (external rotation), Teres minor (external rotation), and Subscapularis (internal rotation) muscles. Often remembered by the mnemonic SITS. Rotator cuff impingement and tearing usually begin in the supraspinatus tendon as it passes under the acromion. Patients are usually >50 years of age and will often have significant pain with abduction above the head and internal rotation (reaching up the back). Often times, patients will complain of difficulty brushing their hair or have pain at night when rolling onto their shoulder. The condition can also occur in young patients, particularly baseball pitchers who will often hear a "pop" caused by a tear in the rotator cuff. On exam, the drop arm test and empty can test are positive. If there is weakness on exam and lack of full improvement with rehabilitative exercises and subacromial corticosteroid injection, you should suspect a tear rather than isolated impingement. Tears are diagnosed by MRI and often require surgical repair.

A 17-year-old boy presents to the emergency department after being tackled while playing football. He is unable to bear weight on his left ankle. On physical exam, he is tender to palpation over the anterior tibiofibular ligament. Dorsiflexing while externally rotating the ankle reproduces his pain. Plain radiographs reveals a diastasis between the distal tibia and fibula, but no fractures. Which of the following is the most likely diagnosis? Achilles tendon sprain Anterior talofibular ligament sprain Lateral collateral ligament sprain Syndesmotic ankle sprain

Correct Answer ( D ) Explanation: This boy most likely has a syndesmotic ankle sprain, or high ankle sprain. A syndesmotic sprain involves a ligamentous injury to one or more of the distal tibiofibular syndesmosis. The ankle joint is made up of the tibia, fibula, and the talus. The fibula and tibia are held together by tibiofibular syndesmosis. High ankle sprains are less common than lateral ankle sprains. Risk factor for high ankle sprains are male gender, contact sport participation, and playing on artificial turf. High ankle sprains most commonly occur as the result of external rotation on a planted foot. An antalgic gait or inability to bear weight, ankle edema, and tenderness over the anterior tibiofibular ligament are the most common findings. Physical exam should also include palpation of the proximal fibular, distal tibia and fibula, and talus to evaluate for associated fracture. Special tests including the single-leg hop test, squeeze test, dorsiflexion-external rotation test, and Cotton test, can be performed to further assess the syndesmosis. Plain radiographs are usually indicated in suspected high ankle sprains. Ankle radiographs may demonstrate diastasis between the tibia and fibula depending upon the severity of the sprain. Magnetic resonance imaging is the gold standard imaging modality for visualizing injury to the syndesmosis. Sprains are graded from 1 to 3. Orthopedic referral is indicated for patients with grade 2 or 3 syndesmosis injuries. Grade 1 injuries are managed conservatively with immobilization, crutches, ice, and nonsteroidal anti-inflammatory drugs (NSAIDs) or acetaminophen. Stage 2 and 3 syndesmosis injuries may require operative management due to high risk of chronic pain and disability.

A 45-year-old woman presents with nonradicular, flexion-based lower back pain that began atraumatically three weeks ago. Her medical history is only significant for hypertension and nephrolithiasis. She denies numbness, weakness, fever, bowel or bladder changes or a history of cancer. Physical exam is unremarkable. Which of the following is the next best step? Order a lumbar MRI with contrast Order a neurosurgical consultation for lumbar spine evaluation Recommend bed rest for five days Recommend continued activity within the limits of the pain

Correct Answer ( D ) Explanation: This patient has acute, nonspecific back pain (lumbago). Lower back pain affects 60% to 80% of adults at some point in their lives. Most cases resolve within a few weeks so management is generally conservative. Patients should be instructed to return to work and continue with daily activity based on their pain tolerance. Engaging in activity within the limits of pain aids recovery. Red flags that raise suspicion for a less benign process include night pain and weight loss (suspect neoplasm); fevers, chills, and sweats (spinal infection); acute bony tenderness (fracture); morning stiffness lasting >30 minutes in young adults (seronegative spondyloarthropathy); and any neurologic deficit or bowel or bladder involvement (nerve root compression).

A 21-year-old woman is brought in by ambulance from a soccer game where she was kicked by a teammate as her left leg was planted. Per the ambulance report, the patient was found with the knee bent completely under her, crying in severe pain. She was unable to bear weight on the extremity at the scene. On examination, there is no gross bony deformity of the left leg, knee, or thigh. Peri-patellar ecchymosis and a significant effusion are noted. The knee hyperextends when the leg is lifted by the heel and the knee joint is extremely unstable on valgus and varus stress. Femoral, patellar, posterior tibial, and dorsalis pedis pulses are present. Which of the following is the most likely diagnosis in this patient? Anterior cruciate ligament tear Medial meniscus tear Patellar dislocation Tibiofemoral dislocation

Correct Answer ( D ) Explanation: Tibiofemoral dislocation is a true limb-threatening emergency. It is caused by multiple ligamentous tears due to hyperextension, posterior force to the anterior tibia, or force to the femur or fibula. Tibiofemoral dislocation is most commonly caused by a motor vehicle collision, but can be caused by sports injuries, falls or even spontaneously in very obese patients. Because of the severe ligamentous damage, many tibiofemoral dislocations will spontaneously reduce prior to presentation. Knee instability in multiple directions should raise suspicion for tibiofemoral dislocation, even if no gross abnormalities are present. Hemarthrosis or significant ecchymosis may also be present on examination. Examination should include a thorough neurovascular check, include an ankle-brachial index and a motor and sensory exam. Tibiofemoral dislocations, once recognized, should be immediately manually reduced. Serial neurovascular checks should follow, as damage to neurovascular structures are common.

A patient presents with back pain. Radiographic findings include a herniated nucleus pulposus between L5 and S1. What are you most likely to find on physical examination if the patient has an S1 radiculopathy? Decreased lateral leg sensation Urinary incontinence Weak dorsiflexion of the foot Weak plantarflexion of the foot

Correct Answer ( D ) Explanation: Weak plantarflexion of the foot is a physical examination finding specific for an L5-S1 herniated nucleus pulposus. A herniated nucleus pulposus, also known as a herniated disc, occurs when there is a weakness in the outer annulus fibrosus and the inner nucleus pulposus herniates outward through the small tears. If not corrected, the nucleus pulposus continues to push outward causing impingement on the spinal nerves resulting in radiculopathy, or disease of a nerve. When a disc herniation occurs between the L5 and S1 vertebra, an S1 radiculopathy occurs resulting in pain radiating down the posterior aspect of the leg and foot along with weakness with plantarflexion due to weakness of the gastrocnemius muscle. The ankle reflex is also typically lost. Diagnosis is formulated through physical examination findings as well as confirmation from imaging. Treatment includes pain medications, physical therapy, and surgical interventions, such as a discectomy to remove the herniated nucleus pulposus from the impinged nerve. Decreased lateral leg sensation (A) is a finding of a nerve impingement at the L4-L5 level. An L5 nerve root impingement is one of the most common sites of radiculopathy of the spine. Patients present with low back pain that radiates down the lateral aspect of their leg and foot and can result in foot drop or decreased strength of foot dorsiflexion. Reflexes are typically spared with an L5 radiculopathy. Weak dorsiflexion of the foot (C) is a result of a herniated nucleus pulposus between L4 and L5. At this level, an L5 radiculopathy causes weakness in foot dorsiflexion, inversion, eversion, and toe extension. Weakness of the foot dorsiflexors results in foot drop because the patient can't lift the foot up.

Which of the following is a characteristic of pseudogout? Crystals are not seen with pseudogout Metatarsophalangeal (MTP) joint is most commonly affected Positive birefringent crystals Uric acid crystals

orrect Answer ( C ) Explanation: Calcium pyrophosphate deposition disease (CPDD) is known as pseudogout because of the acute gout-like attacks that CPPD crystals can cause. Chondrocalcinosis refers to radiographically detectable densities in cartilage and joint inflammation caused by these calcium-containing crystals. Calcium pyrophosphate (CP) crystals can be deposited not only on articular cartilage but also in ligaments, tendons, soft tissues, and synovium. Patients with pseudogout present with a monoarticular inflammatory arthropathy similar to gout. The knee is the most commonly affected joint. Arthrocentesis reveals weakly positive birefringent rhomboid-shaped crystals. Treatment of acute pseudogout consists of NSAIDs, colchicine or corticosteroid injections. Colchicine can also be used for chronic cases.


Kaugnay na mga set ng pag-aaral

ACCT 200 - Test 1 (Reading Notes)

View Set

Nurs 220 (Nutrition) CoursePoint Chapter 8

View Set

Govt 2305 Chapter 8 presentation notes Fall 2019 Gottermoller

View Set

Final A&P Comprehensive Chapters 1-15

View Set

World History Unit 7 Answers PHS

View Set